Allowed Publications
Slot System
Featured Buckets
Featured Buckets Admin

Things We Do For No Reason™: Routinely Holding Metformin in the Hospital

Article Type
Changed
Wed, 08/18/2021 - 12:51
Display Headline
Things We Do For No Reason™: Routinely Holding Metformin in the Hospital

Inspired by the ABIM Foundation’s Choosing Wisely® campaign, the “Things We Do for No Reason” (TWDFNR) series reviews practices that have become common parts of hospital care but may provide little value to our patients. Practices reviewed in the TWDFNR series do not represent clear-cut conclusions or clinical practice standards but are meant as a starting place for research and active discussions among hospitalists and patients. We invite you to be part of that discussion.

CLINICAL SCENARIO

A hospitalist admits a 29-year-old man with hypertension, obesity, and type 2 diabetes (type 2 DM) for a posterior neck abscess that failed outpatient oral antibiotic therapy. The patient’s medications include metformin monotherapy. Vital signs taken upon admission include a blood pressure of 136/82 mm Hg, heart rate of 98 beats per minute, respiratory rate 18 of breaths per minute, oxygen saturation of 100% on room air, and temperature of 38.5 oC. Laboratory evaluation revealed a glucose level of 212 mg/dL, with a hemoglobin A1c of 8.0%, lactic acid of 1.4 mmol/L, and normal renal and hepatic function. Based on these findings, the hospitalist holds metformin and starts the patient on sliding-scale insulin therapy.

WHY YOU MIGHT THINK ROUTINELY HOLDING METFORMIN IN THE HOSPITAL IS NECESSARY

Metformin, an oral medication used to treat type 2 DM, is a biguanide that increases peripheral glucose utilization and decreases hepatic gluconeogenesis. However, metformin-associated shunting of metabolism toward anaerobic respiration increases the risk of lactic acidosis.1 Because the kidneys excrete metformin, the risk of developing metformin-associated lactic acidosis (MALA) increases with renal impairment. Disease states common among hospitalized patients, such as hypoperfusion, advanced cirrhosis, alcohol abuse, cardiac failure, muscle ischemia, and severe infection, increase the risk of acute kidney injury (AKI) and elevate blood lactate levels. Therefore, hospitalists regularly hold metformin in the inpatient setting.

Following the introduction of metformin in the United States, the US Food and Drug Administration (FDA) received 47 confirmed reports of nonfatal lactic acidosis associated with the use of metformin, all of which involved cardiac disease (specifically congestive heart failure [CHF]), renal insufficiency, hypoxia, or sepsis.2 Consequently, the FDA listed CHF as a contraindication to metformin use; however, it has since changed the use of metformin in CHF from a contraindication to a warning/precaution for lactic acidosis. The FDA also added a warning against the use of metformin in patients with sepsis or in patients older than 80 years who have abnormal creatinine clearance.

Acute kidney injury, a common inpatient condition, occurs in 20% of hospitalized patients and more than 50% of intensive care patients.3 Moreover, a retrospective observational study showed approximately 50% of all patients hospitalized for COVID-19 had AKI.4 Iodinated contrast, a diagnostic media commonly used in the hospital, may also increase the risk of renal dysfunction. The FDA recommends providers discontinue metformin at or before initiating imaging studies with iodinated contrast5 in patients with an estimated glomerular filtration rate (eGFR) between 30 and 60 mL/min/1.73 m2. The FDA also advises that providers not restart metformin until 48 hours after an intra-arterial (IA) or intravenous (IV) contrast study in patients with an eGFR <60 mL/min/1.73 m2 (equivalent to chronic kidney disease [CKD] stage 3 or worse).5 The American Diabetes Association (ADA) recommends the same eGFR cutoff level in its clinical practice recommendations, as well as withholding metformin 48 hours before patients receive IV contrast.6 Given the risk of AKI in hospitalized patients and concerns of increased MALA, clinicians reflexively hold metformin.

Holding metformin is also consistent with professional guidelines. The 2009 American Association of Clinical Endocrinology and ADA Consensus Statement on Inpatient Glycemic Control recommends cautious use of metformin in the inpatient setting “because of the potential development of a contraindication during the hospitalization.”7 Similarly, the 2012 Endocrine Society guidelines recommend withholding metformin in almost all hospitalized patients.8

WHY ROUTINELY HOLDING METFORMIN IN THE HOSPITAL IS NOT BENEFICIAL

Routinely holding metformin in hospitalized patients is unnecessary and potentially harmful. First, MALA is exceedingly rare, and experts question the causal link. Furthermore, iodinated contrast does not place patients with normal renal function at increased risk of MALA. Finally, holding metformin leads to worsened glycemic control and increased use of insulin, both of which may result in adverse patient outcomes.

The concerns about MALA stem from clinical experiences with phenformin, an older and more potent biguanide. Phenformin shares a similar mechanism of action with metformin but causes more lactic acid production. In 1978, following 306 documented cases of phenformin-associated lactic acidosis, the FDA removed this medication from the market.9 Since the initial 47 cases of MALA were reported to the FDA, repeated studies and systematic reviews have disputed the link between metformin and lactic acidosis, particularly in the absence of significant risk factors or in patients with an eGFR ≥30 mL/min/1.73 m2. In fact, a large observational study showed a reduction in acidosis and mortality in outpatients with stage 3a CKD (eGFR, 45-59 mL/min/1.73 m2) who were taking metformin compared to patients taking insulin or other oral hypoglycemics agents.10 In patients with stage 3b CKD (eGFR, 30-44 mL/min/1.73 m2), this study found no difference in the same outcomes.10

Studies show that metformin does not cause elevated lactate levels in patients with stage 4 CKD (eGFR >15mL/min/1.732) or lower stages of CKD as long as doses are adjusted appropriately to reflect renal function.11 These and other investigations reveal that in the absence of other risk factors, metformin does not cause lactic acidosis (Table).10-15 Based on these findings, the Endocrine Society changed the strength of its recommendation to withhold metformin in hospitalized patients to “weak,” with “very low-quality evidence.” The FDA similarly revised its warnings8 to allow metformin use in all patients with an eGFR ≥30 mL/min/1.73 m2. A large community-based cohort study, which demonstrated no association between hospitalization with acidosis and metformin use in patients with stage 3b CKD or lower stages of CKD, supports this change in treatment threshold.15

JHMVol16No8_Cohen10740818e_t1.JPG

Published evidence also does not support the practice of routinely holding metformin before contrast administration, despite concerns regarding contrast-induced nephropathy. Retrospective chart reviews and a direct comparison in human models have not shown any significant difference in the risk of AKI between the IV and IA contrast.16 Moreover, evidence suggests no interaction between metformin and contrast media in patients with normal renal function.17 In response, the American College of Radiology, Canadian Association of Radiology, Royal College of Radiologists, and Royal Australian and New Zealand College of Radiologists all recommend continuing metformin in patients with normal renal function (eGFR ≥30 mL/min/1.73m2) receiving IV contrast. They advise holding metformin for 48 hours in patients with renal insufficiency (eGFR <30 mL/min/1.73m2) or those undergoing IA catheter studies that might result in renal artery emboli.18

Finally, continuing metformin maintains steady blood glucose control. The practice of replacing metformin with sliding-scale insulin monotherapy for hospitalized patients significantly increases the risk of hyperglycemia and is associated with an increased length of stay.19 Additionally, unlike insulin, metformin does not increase the risk of hypoglycemia. Finally, a recent matched cohort study comparing the use of oral hypoglycemic agents (metformin, thiazolidines, and sulfonylureas) vs insulin monotherapy in patients undergoing emergency abdominal surgery showed that the patients admitted with sepsis and treated with oral agents had a lower 30-day mortality rate and a shorter length of stay.20 Based on the evidence showing that inpatient oral hypoglycemic agents improve quality metrics and mitigate safety events, the ADA advocates resuming oral antihyperglycemic medications (most commonly metformin) 1 to 2 days before discharge.7

WHAT YOU SHOULD DO INSTEAD

Clinicians should continue metformin in all hospitalized patients who are not at significant risk of developing lactic acidosis. Risk factors for MALA include severe sepsis (in the setting of end-organ damage as defined by systemic inflammatory response syndrome criteria), hypoxia requiring oxygen supplementation, hypoperfusion (as from CHF), AKI, CKD (eGFR <30 mL/min/1.73 m2), and advanced cirrhosis. Given the high rates of hypoxia and AKI in admitted patients with COVID-19, clinicians should hold metformin on admission. Continue metformin for patients receiving IV contrast media with an eGFR >30 mL/min/1.73 m2. For patients undergoing IA catheter studies associated with a risk for renal artery emboli, or in patients with renal insufficiency (eGFR <30 mL/min/1.73 m2), temporarily hold metformin for 48 hours. When held, restart metformin as soon as risk factors resolve.

RECOMMENDATIONS

  • Hold metformin in patients with or undergoing the following:
    • High risk for or currently suffering from decompensated heart failure, severe sepsis, or other disease states resulting in hypoxia or tissue hypoperfusion;
    • An eGFR <30 mL/min/1.73 m2 or AKI; resume metformin when the AKI resolves;
    • COVID-19 infection, until the risk of hypoxia has resolved;
    • IV contrast study in the presence of acute renal failure or an eGFR <30 mL/min/1.73 m2; resume metformin 48 hours after contrast administration;
    • Intra-arterial catheter study that might result in renal artery emboli; resume metformin when renal function normalizes.
  • Continue metformin in all hospitalized patients in the absence of the aforementioned disease states or contrast-related indications.

CONCLUSION

Returning to the patient in our clinical scenario, we recommend continuing metformin given the lack of risk factors or disease states associated with increased lactic acidosis. The practice of withholding metformin in hospitalized patients for fear of MALA is based on minimal evidence. Clinicians should, however, hold metformin in patients who have true contraindications, including existing acidosis, hypoperfusion, renal insufficiency, CHF, severe sepsis, hypoxia, advanced cirrhosis, and COVID-19. With regard to iodinated contrast studies, temporarily withhold metformin for 48 hours in patients with an eGFR <30 mL/min/1.73 m2, acute kidney injury, or in patients undergoing an IA catheter study at risk for renal artery emboli. Patients should be restarted on metformin 48 hours after these studies and as renal function normalizes. When withholding metformin during a hospitalization, restart it once risk factors have resolved.

Do you think this is a low-value practice? Is this truly a “Thing We Do for No Reason”? Share what you do in your practice and join in the conversation online by retweeting it on Twitter  (#TWDFNR) and liking it on Facebook. We invite you to propose ideas for other “Things We Do for No Reason” topics by emailing TWDFNR@hospitalmedicine.org

References

1. Kopec KT, Kowalski MJ. Metformin-associated lactic acidosis (MALA): case files of the Einstein Medical Center medical toxicology fellowship. J Med Toxicol. 2013;9(1):61-66. https://doi.org/10.1007/s13181-012-0278-3
2. Misbin RI, Green L, Stadel BV, Gueriguian JL, Gubbi A, Fleming GA. Lactic acidosis in patients with diabetes treated with metformin. N Engl J Med. 1998;338(4):265-266. https://doi.org/10.1056/nejm199801223380415
3. Wang HE, Muntner P, Chertow GM, Warnock DG. Acute kidney injury and mortality in hospitalized patients. Am J Nephrol. 2012;35(4):349-355. https://doi.org/10.1159/000337487
4. Chan L, Chaudhary K, Saha A, et al; Mount Sinai COVID Informatics Center (MSCIC), Li L. AKI in hospitalized patients with COVID-19. J Am Soc Nephrol. 2021;32(1):151-160. https://doi.org/10.1681/asn.2020050615
5. US Food and Drug Administration. FDA drug safety communication: FDA revises warnings regarding use of the diabetes medicine metformin in certain patients with reduced kidney function. Updated November 14, 2017. Accessed June 22, 2021. https://www.fda.gov/drugs/drug-safety-and-availability/fda-drug-safety-communication-fda-revises-warnings-regarding-use-diabetes-medicine-metformin-certain
6. American Diabetes Association. 9. Pharmacologic approaches to glycemic treatment: standards of medical care in diabetes—2019. Diabetes Care. 2019;42 (Suppl 1):S90-S102. https://doi.org/10.2337/dc19-s009
7. Moghissi ES, Korytkowski MT, DiNardo M, et al; American Association of Clinical Endocrinologists; American Diabetes Association. Association of Clinical Endocrinologists and American Diabetes Association consensus statement on inpatient glycemic control. Diabetes Care. 2009;32(6):1119-1131. https://doi.org/10.2337/dc09-9029
8. Umpierrez GE, Hellman R, Korytkowski MT, et al; Endocrine Society. Management of hyperglycemia in hospitalized patients in non-critical care setting: an endocrine society clinical practice guideline. J Clin Endocrinol Metab. 2012;97(1):16-38. https://doi.org/10.1210/jc.2011-2098
9. Misbin RI. Phenformin-associated lactic acidosis: pathogenesis and treatment. Ann Intern Med. 1977;87(5):591-595. https://doi.org/10.7326/0003-4819-87-5-591
10. Ekström N, Schiöler L, Svensson AM, et al. Effectiveness and safety of metformin in 51 675 patients with type 2 diabetes and different levels of renal function: a cohort study from the Swedish National Diabetes Register. BMJ Open. 2012;2(4):e001076. https://doi.org/10.1136/bmjopen-2012-001076
11. Lalau JD, Kajbaf F, Bennis Y, Hurtel-Lemaire AS, Belpaire F, De Broe ME. Metformin treatment in patients with type 2 diabetes and chronic kidney disease stages 3A, 3B, or 4. Diabetes Care. 2018;41(3):547-553. https://doi.org/10.2337/dc17-2231
12. Brown JB, Pedula K, Barzilay J, Herson MK, Latare P. Lactic acidosis rates in type 2 diabetes. Diabetes Care. 1998;21(10):1659-1663. https://doi.org/10.2337/diacare.21.10.1659
13. Lalau JD, Race JM. Lactic acidosis in metformin-treated patients. Prognostic value of arterial lactate levels and plasma metformin concentrations. Drug Saf. 1999;20(4):377-384. https://doi.org/10.2165/00002018-199920040-00006
14. Salpeter SR, Greyber E, Pasternak GA, Salpeter Posthumous EE. Risk of fatal and nonfatal lactic acidosis with metformin use in type 2 diabetes mellitus. Cochrane Database Syst Rev. 2010;(1):CD002967. https://doi.org/10.1002/14651858.cd002967.pub3
15. Lazarus B, Wu A, Shin JI, et al. Association of metformin use with risk of lactic acidosis across the range of kidney function: a community-based cohort study. JAMA Intern Med. 2018;178(7):903-910. https://doi.org/10.1001/jamainternmed.2018.0292
16. McDonald JS, Leake CB, McDonald RJ, et al. Acute kidney injury after intravenous versus intra-arterial contrast material administration in a paired cohort. Invest Radiol. 2016;51(12):804-809. https://doi.org/10.1097/rli.0000000000000298
17. Zeller M, Labalette-Bart M, Juliard JM, et al. Metformin and contrast-induced acute kidney injury in diabetic patients treated with primary percutaneous coronary intervention for ST segment elevation myocardial infarction: a multicenter study. Int J Cardiol. 2016;220:137-142. https://doi.org/10.1016/j.ijcard.2016.06.076
18. Goergen SK, Rumbold G, Compton G, Harris C. Systematic review of current guidelines, and their evidence base, on risk of lactic acidosis after administration of contrast medium for patients receiving metformin. Radiology. 2010;254(1):261-269. https://doi.org/10.1148/radiol.09090690
19. Ambrus DB, O’Connor MJ. Things we do for no reason: sliding-scale insulin as monotherapy for glycemic control in hospitalized patients. J Hosp Med. 2019;14(2):114-116. https://doi.org/10.12788/jhm.3109
20. Haltmeier T, Benjamin E, Beale E, Inaba K, Demetriades D. Insulin-treated patients with diabetes mellitus undergoing emergency abdominal surgery have worse outcomes than patients treated with oral agents. World J Surg. 2016;40(7):1575-1582. https://doi.org/10.1007/s00268-016-3469-2

Article PDF
Author and Disclosure Information

1Department of Medicine, Rutgers Robert Wood Johnson Medical School, New Brunswick, New Jersey; 2Department of Medicine, Harvard Medical School, Boston, Massachusetts; 3Carl J Shapiro Institute for Education and Research, Beth Israel Deaconess Medical Center, Boston, Massachusetts.

Disclosures
The authors have no conflicts to disclose.

Publications
Topics
Sections
Author and Disclosure Information

1Department of Medicine, Rutgers Robert Wood Johnson Medical School, New Brunswick, New Jersey; 2Department of Medicine, Harvard Medical School, Boston, Massachusetts; 3Carl J Shapiro Institute for Education and Research, Beth Israel Deaconess Medical Center, Boston, Massachusetts.

Disclosures
The authors have no conflicts to disclose.

Author and Disclosure Information

1Department of Medicine, Rutgers Robert Wood Johnson Medical School, New Brunswick, New Jersey; 2Department of Medicine, Harvard Medical School, Boston, Massachusetts; 3Carl J Shapiro Institute for Education and Research, Beth Israel Deaconess Medical Center, Boston, Massachusetts.

Disclosures
The authors have no conflicts to disclose.

Article PDF
Article PDF
Related Articles

Inspired by the ABIM Foundation’s Choosing Wisely® campaign, the “Things We Do for No Reason” (TWDFNR) series reviews practices that have become common parts of hospital care but may provide little value to our patients. Practices reviewed in the TWDFNR series do not represent clear-cut conclusions or clinical practice standards but are meant as a starting place for research and active discussions among hospitalists and patients. We invite you to be part of that discussion.

CLINICAL SCENARIO

A hospitalist admits a 29-year-old man with hypertension, obesity, and type 2 diabetes (type 2 DM) for a posterior neck abscess that failed outpatient oral antibiotic therapy. The patient’s medications include metformin monotherapy. Vital signs taken upon admission include a blood pressure of 136/82 mm Hg, heart rate of 98 beats per minute, respiratory rate 18 of breaths per minute, oxygen saturation of 100% on room air, and temperature of 38.5 oC. Laboratory evaluation revealed a glucose level of 212 mg/dL, with a hemoglobin A1c of 8.0%, lactic acid of 1.4 mmol/L, and normal renal and hepatic function. Based on these findings, the hospitalist holds metformin and starts the patient on sliding-scale insulin therapy.

WHY YOU MIGHT THINK ROUTINELY HOLDING METFORMIN IN THE HOSPITAL IS NECESSARY

Metformin, an oral medication used to treat type 2 DM, is a biguanide that increases peripheral glucose utilization and decreases hepatic gluconeogenesis. However, metformin-associated shunting of metabolism toward anaerobic respiration increases the risk of lactic acidosis.1 Because the kidneys excrete metformin, the risk of developing metformin-associated lactic acidosis (MALA) increases with renal impairment. Disease states common among hospitalized patients, such as hypoperfusion, advanced cirrhosis, alcohol abuse, cardiac failure, muscle ischemia, and severe infection, increase the risk of acute kidney injury (AKI) and elevate blood lactate levels. Therefore, hospitalists regularly hold metformin in the inpatient setting.

Following the introduction of metformin in the United States, the US Food and Drug Administration (FDA) received 47 confirmed reports of nonfatal lactic acidosis associated with the use of metformin, all of which involved cardiac disease (specifically congestive heart failure [CHF]), renal insufficiency, hypoxia, or sepsis.2 Consequently, the FDA listed CHF as a contraindication to metformin use; however, it has since changed the use of metformin in CHF from a contraindication to a warning/precaution for lactic acidosis. The FDA also added a warning against the use of metformin in patients with sepsis or in patients older than 80 years who have abnormal creatinine clearance.

Acute kidney injury, a common inpatient condition, occurs in 20% of hospitalized patients and more than 50% of intensive care patients.3 Moreover, a retrospective observational study showed approximately 50% of all patients hospitalized for COVID-19 had AKI.4 Iodinated contrast, a diagnostic media commonly used in the hospital, may also increase the risk of renal dysfunction. The FDA recommends providers discontinue metformin at or before initiating imaging studies with iodinated contrast5 in patients with an estimated glomerular filtration rate (eGFR) between 30 and 60 mL/min/1.73 m2. The FDA also advises that providers not restart metformin until 48 hours after an intra-arterial (IA) or intravenous (IV) contrast study in patients with an eGFR <60 mL/min/1.73 m2 (equivalent to chronic kidney disease [CKD] stage 3 or worse).5 The American Diabetes Association (ADA) recommends the same eGFR cutoff level in its clinical practice recommendations, as well as withholding metformin 48 hours before patients receive IV contrast.6 Given the risk of AKI in hospitalized patients and concerns of increased MALA, clinicians reflexively hold metformin.

Holding metformin is also consistent with professional guidelines. The 2009 American Association of Clinical Endocrinology and ADA Consensus Statement on Inpatient Glycemic Control recommends cautious use of metformin in the inpatient setting “because of the potential development of a contraindication during the hospitalization.”7 Similarly, the 2012 Endocrine Society guidelines recommend withholding metformin in almost all hospitalized patients.8

WHY ROUTINELY HOLDING METFORMIN IN THE HOSPITAL IS NOT BENEFICIAL

Routinely holding metformin in hospitalized patients is unnecessary and potentially harmful. First, MALA is exceedingly rare, and experts question the causal link. Furthermore, iodinated contrast does not place patients with normal renal function at increased risk of MALA. Finally, holding metformin leads to worsened glycemic control and increased use of insulin, both of which may result in adverse patient outcomes.

The concerns about MALA stem from clinical experiences with phenformin, an older and more potent biguanide. Phenformin shares a similar mechanism of action with metformin but causes more lactic acid production. In 1978, following 306 documented cases of phenformin-associated lactic acidosis, the FDA removed this medication from the market.9 Since the initial 47 cases of MALA were reported to the FDA, repeated studies and systematic reviews have disputed the link between metformin and lactic acidosis, particularly in the absence of significant risk factors or in patients with an eGFR ≥30 mL/min/1.73 m2. In fact, a large observational study showed a reduction in acidosis and mortality in outpatients with stage 3a CKD (eGFR, 45-59 mL/min/1.73 m2) who were taking metformin compared to patients taking insulin or other oral hypoglycemics agents.10 In patients with stage 3b CKD (eGFR, 30-44 mL/min/1.73 m2), this study found no difference in the same outcomes.10

Studies show that metformin does not cause elevated lactate levels in patients with stage 4 CKD (eGFR >15mL/min/1.732) or lower stages of CKD as long as doses are adjusted appropriately to reflect renal function.11 These and other investigations reveal that in the absence of other risk factors, metformin does not cause lactic acidosis (Table).10-15 Based on these findings, the Endocrine Society changed the strength of its recommendation to withhold metformin in hospitalized patients to “weak,” with “very low-quality evidence.” The FDA similarly revised its warnings8 to allow metformin use in all patients with an eGFR ≥30 mL/min/1.73 m2. A large community-based cohort study, which demonstrated no association between hospitalization with acidosis and metformin use in patients with stage 3b CKD or lower stages of CKD, supports this change in treatment threshold.15

JHMVol16No8_Cohen10740818e_t1.JPG

Published evidence also does not support the practice of routinely holding metformin before contrast administration, despite concerns regarding contrast-induced nephropathy. Retrospective chart reviews and a direct comparison in human models have not shown any significant difference in the risk of AKI between the IV and IA contrast.16 Moreover, evidence suggests no interaction between metformin and contrast media in patients with normal renal function.17 In response, the American College of Radiology, Canadian Association of Radiology, Royal College of Radiologists, and Royal Australian and New Zealand College of Radiologists all recommend continuing metformin in patients with normal renal function (eGFR ≥30 mL/min/1.73m2) receiving IV contrast. They advise holding metformin for 48 hours in patients with renal insufficiency (eGFR <30 mL/min/1.73m2) or those undergoing IA catheter studies that might result in renal artery emboli.18

Finally, continuing metformin maintains steady blood glucose control. The practice of replacing metformin with sliding-scale insulin monotherapy for hospitalized patients significantly increases the risk of hyperglycemia and is associated with an increased length of stay.19 Additionally, unlike insulin, metformin does not increase the risk of hypoglycemia. Finally, a recent matched cohort study comparing the use of oral hypoglycemic agents (metformin, thiazolidines, and sulfonylureas) vs insulin monotherapy in patients undergoing emergency abdominal surgery showed that the patients admitted with sepsis and treated with oral agents had a lower 30-day mortality rate and a shorter length of stay.20 Based on the evidence showing that inpatient oral hypoglycemic agents improve quality metrics and mitigate safety events, the ADA advocates resuming oral antihyperglycemic medications (most commonly metformin) 1 to 2 days before discharge.7

WHAT YOU SHOULD DO INSTEAD

Clinicians should continue metformin in all hospitalized patients who are not at significant risk of developing lactic acidosis. Risk factors for MALA include severe sepsis (in the setting of end-organ damage as defined by systemic inflammatory response syndrome criteria), hypoxia requiring oxygen supplementation, hypoperfusion (as from CHF), AKI, CKD (eGFR <30 mL/min/1.73 m2), and advanced cirrhosis. Given the high rates of hypoxia and AKI in admitted patients with COVID-19, clinicians should hold metformin on admission. Continue metformin for patients receiving IV contrast media with an eGFR >30 mL/min/1.73 m2. For patients undergoing IA catheter studies associated with a risk for renal artery emboli, or in patients with renal insufficiency (eGFR <30 mL/min/1.73 m2), temporarily hold metformin for 48 hours. When held, restart metformin as soon as risk factors resolve.

RECOMMENDATIONS

  • Hold metformin in patients with or undergoing the following:
    • High risk for or currently suffering from decompensated heart failure, severe sepsis, or other disease states resulting in hypoxia or tissue hypoperfusion;
    • An eGFR <30 mL/min/1.73 m2 or AKI; resume metformin when the AKI resolves;
    • COVID-19 infection, until the risk of hypoxia has resolved;
    • IV contrast study in the presence of acute renal failure or an eGFR <30 mL/min/1.73 m2; resume metformin 48 hours after contrast administration;
    • Intra-arterial catheter study that might result in renal artery emboli; resume metformin when renal function normalizes.
  • Continue metformin in all hospitalized patients in the absence of the aforementioned disease states or contrast-related indications.

CONCLUSION

Returning to the patient in our clinical scenario, we recommend continuing metformin given the lack of risk factors or disease states associated with increased lactic acidosis. The practice of withholding metformin in hospitalized patients for fear of MALA is based on minimal evidence. Clinicians should, however, hold metformin in patients who have true contraindications, including existing acidosis, hypoperfusion, renal insufficiency, CHF, severe sepsis, hypoxia, advanced cirrhosis, and COVID-19. With regard to iodinated contrast studies, temporarily withhold metformin for 48 hours in patients with an eGFR <30 mL/min/1.73 m2, acute kidney injury, or in patients undergoing an IA catheter study at risk for renal artery emboli. Patients should be restarted on metformin 48 hours after these studies and as renal function normalizes. When withholding metformin during a hospitalization, restart it once risk factors have resolved.

Do you think this is a low-value practice? Is this truly a “Thing We Do for No Reason”? Share what you do in your practice and join in the conversation online by retweeting it on Twitter  (#TWDFNR) and liking it on Facebook. We invite you to propose ideas for other “Things We Do for No Reason” topics by emailing TWDFNR@hospitalmedicine.org

Inspired by the ABIM Foundation’s Choosing Wisely® campaign, the “Things We Do for No Reason” (TWDFNR) series reviews practices that have become common parts of hospital care but may provide little value to our patients. Practices reviewed in the TWDFNR series do not represent clear-cut conclusions or clinical practice standards but are meant as a starting place for research and active discussions among hospitalists and patients. We invite you to be part of that discussion.

CLINICAL SCENARIO

A hospitalist admits a 29-year-old man with hypertension, obesity, and type 2 diabetes (type 2 DM) for a posterior neck abscess that failed outpatient oral antibiotic therapy. The patient’s medications include metformin monotherapy. Vital signs taken upon admission include a blood pressure of 136/82 mm Hg, heart rate of 98 beats per minute, respiratory rate 18 of breaths per minute, oxygen saturation of 100% on room air, and temperature of 38.5 oC. Laboratory evaluation revealed a glucose level of 212 mg/dL, with a hemoglobin A1c of 8.0%, lactic acid of 1.4 mmol/L, and normal renal and hepatic function. Based on these findings, the hospitalist holds metformin and starts the patient on sliding-scale insulin therapy.

WHY YOU MIGHT THINK ROUTINELY HOLDING METFORMIN IN THE HOSPITAL IS NECESSARY

Metformin, an oral medication used to treat type 2 DM, is a biguanide that increases peripheral glucose utilization and decreases hepatic gluconeogenesis. However, metformin-associated shunting of metabolism toward anaerobic respiration increases the risk of lactic acidosis.1 Because the kidneys excrete metformin, the risk of developing metformin-associated lactic acidosis (MALA) increases with renal impairment. Disease states common among hospitalized patients, such as hypoperfusion, advanced cirrhosis, alcohol abuse, cardiac failure, muscle ischemia, and severe infection, increase the risk of acute kidney injury (AKI) and elevate blood lactate levels. Therefore, hospitalists regularly hold metformin in the inpatient setting.

Following the introduction of metformin in the United States, the US Food and Drug Administration (FDA) received 47 confirmed reports of nonfatal lactic acidosis associated with the use of metformin, all of which involved cardiac disease (specifically congestive heart failure [CHF]), renal insufficiency, hypoxia, or sepsis.2 Consequently, the FDA listed CHF as a contraindication to metformin use; however, it has since changed the use of metformin in CHF from a contraindication to a warning/precaution for lactic acidosis. The FDA also added a warning against the use of metformin in patients with sepsis or in patients older than 80 years who have abnormal creatinine clearance.

Acute kidney injury, a common inpatient condition, occurs in 20% of hospitalized patients and more than 50% of intensive care patients.3 Moreover, a retrospective observational study showed approximately 50% of all patients hospitalized for COVID-19 had AKI.4 Iodinated contrast, a diagnostic media commonly used in the hospital, may also increase the risk of renal dysfunction. The FDA recommends providers discontinue metformin at or before initiating imaging studies with iodinated contrast5 in patients with an estimated glomerular filtration rate (eGFR) between 30 and 60 mL/min/1.73 m2. The FDA also advises that providers not restart metformin until 48 hours after an intra-arterial (IA) or intravenous (IV) contrast study in patients with an eGFR <60 mL/min/1.73 m2 (equivalent to chronic kidney disease [CKD] stage 3 or worse).5 The American Diabetes Association (ADA) recommends the same eGFR cutoff level in its clinical practice recommendations, as well as withholding metformin 48 hours before patients receive IV contrast.6 Given the risk of AKI in hospitalized patients and concerns of increased MALA, clinicians reflexively hold metformin.

Holding metformin is also consistent with professional guidelines. The 2009 American Association of Clinical Endocrinology and ADA Consensus Statement on Inpatient Glycemic Control recommends cautious use of metformin in the inpatient setting “because of the potential development of a contraindication during the hospitalization.”7 Similarly, the 2012 Endocrine Society guidelines recommend withholding metformin in almost all hospitalized patients.8

WHY ROUTINELY HOLDING METFORMIN IN THE HOSPITAL IS NOT BENEFICIAL

Routinely holding metformin in hospitalized patients is unnecessary and potentially harmful. First, MALA is exceedingly rare, and experts question the causal link. Furthermore, iodinated contrast does not place patients with normal renal function at increased risk of MALA. Finally, holding metformin leads to worsened glycemic control and increased use of insulin, both of which may result in adverse patient outcomes.

The concerns about MALA stem from clinical experiences with phenformin, an older and more potent biguanide. Phenformin shares a similar mechanism of action with metformin but causes more lactic acid production. In 1978, following 306 documented cases of phenformin-associated lactic acidosis, the FDA removed this medication from the market.9 Since the initial 47 cases of MALA were reported to the FDA, repeated studies and systematic reviews have disputed the link between metformin and lactic acidosis, particularly in the absence of significant risk factors or in patients with an eGFR ≥30 mL/min/1.73 m2. In fact, a large observational study showed a reduction in acidosis and mortality in outpatients with stage 3a CKD (eGFR, 45-59 mL/min/1.73 m2) who were taking metformin compared to patients taking insulin or other oral hypoglycemics agents.10 In patients with stage 3b CKD (eGFR, 30-44 mL/min/1.73 m2), this study found no difference in the same outcomes.10

Studies show that metformin does not cause elevated lactate levels in patients with stage 4 CKD (eGFR >15mL/min/1.732) or lower stages of CKD as long as doses are adjusted appropriately to reflect renal function.11 These and other investigations reveal that in the absence of other risk factors, metformin does not cause lactic acidosis (Table).10-15 Based on these findings, the Endocrine Society changed the strength of its recommendation to withhold metformin in hospitalized patients to “weak,” with “very low-quality evidence.” The FDA similarly revised its warnings8 to allow metformin use in all patients with an eGFR ≥30 mL/min/1.73 m2. A large community-based cohort study, which demonstrated no association between hospitalization with acidosis and metformin use in patients with stage 3b CKD or lower stages of CKD, supports this change in treatment threshold.15

JHMVol16No8_Cohen10740818e_t1.JPG

Published evidence also does not support the practice of routinely holding metformin before contrast administration, despite concerns regarding contrast-induced nephropathy. Retrospective chart reviews and a direct comparison in human models have not shown any significant difference in the risk of AKI between the IV and IA contrast.16 Moreover, evidence suggests no interaction between metformin and contrast media in patients with normal renal function.17 In response, the American College of Radiology, Canadian Association of Radiology, Royal College of Radiologists, and Royal Australian and New Zealand College of Radiologists all recommend continuing metformin in patients with normal renal function (eGFR ≥30 mL/min/1.73m2) receiving IV contrast. They advise holding metformin for 48 hours in patients with renal insufficiency (eGFR <30 mL/min/1.73m2) or those undergoing IA catheter studies that might result in renal artery emboli.18

Finally, continuing metformin maintains steady blood glucose control. The practice of replacing metformin with sliding-scale insulin monotherapy for hospitalized patients significantly increases the risk of hyperglycemia and is associated with an increased length of stay.19 Additionally, unlike insulin, metformin does not increase the risk of hypoglycemia. Finally, a recent matched cohort study comparing the use of oral hypoglycemic agents (metformin, thiazolidines, and sulfonylureas) vs insulin monotherapy in patients undergoing emergency abdominal surgery showed that the patients admitted with sepsis and treated with oral agents had a lower 30-day mortality rate and a shorter length of stay.20 Based on the evidence showing that inpatient oral hypoglycemic agents improve quality metrics and mitigate safety events, the ADA advocates resuming oral antihyperglycemic medications (most commonly metformin) 1 to 2 days before discharge.7

WHAT YOU SHOULD DO INSTEAD

Clinicians should continue metformin in all hospitalized patients who are not at significant risk of developing lactic acidosis. Risk factors for MALA include severe sepsis (in the setting of end-organ damage as defined by systemic inflammatory response syndrome criteria), hypoxia requiring oxygen supplementation, hypoperfusion (as from CHF), AKI, CKD (eGFR <30 mL/min/1.73 m2), and advanced cirrhosis. Given the high rates of hypoxia and AKI in admitted patients with COVID-19, clinicians should hold metformin on admission. Continue metformin for patients receiving IV contrast media with an eGFR >30 mL/min/1.73 m2. For patients undergoing IA catheter studies associated with a risk for renal artery emboli, or in patients with renal insufficiency (eGFR <30 mL/min/1.73 m2), temporarily hold metformin for 48 hours. When held, restart metformin as soon as risk factors resolve.

RECOMMENDATIONS

  • Hold metformin in patients with or undergoing the following:
    • High risk for or currently suffering from decompensated heart failure, severe sepsis, or other disease states resulting in hypoxia or tissue hypoperfusion;
    • An eGFR <30 mL/min/1.73 m2 or AKI; resume metformin when the AKI resolves;
    • COVID-19 infection, until the risk of hypoxia has resolved;
    • IV contrast study in the presence of acute renal failure or an eGFR <30 mL/min/1.73 m2; resume metformin 48 hours after contrast administration;
    • Intra-arterial catheter study that might result in renal artery emboli; resume metformin when renal function normalizes.
  • Continue metformin in all hospitalized patients in the absence of the aforementioned disease states or contrast-related indications.

CONCLUSION

Returning to the patient in our clinical scenario, we recommend continuing metformin given the lack of risk factors or disease states associated with increased lactic acidosis. The practice of withholding metformin in hospitalized patients for fear of MALA is based on minimal evidence. Clinicians should, however, hold metformin in patients who have true contraindications, including existing acidosis, hypoperfusion, renal insufficiency, CHF, severe sepsis, hypoxia, advanced cirrhosis, and COVID-19. With regard to iodinated contrast studies, temporarily withhold metformin for 48 hours in patients with an eGFR <30 mL/min/1.73 m2, acute kidney injury, or in patients undergoing an IA catheter study at risk for renal artery emboli. Patients should be restarted on metformin 48 hours after these studies and as renal function normalizes. When withholding metformin during a hospitalization, restart it once risk factors have resolved.

Do you think this is a low-value practice? Is this truly a “Thing We Do for No Reason”? Share what you do in your practice and join in the conversation online by retweeting it on Twitter  (#TWDFNR) and liking it on Facebook. We invite you to propose ideas for other “Things We Do for No Reason” topics by emailing TWDFNR@hospitalmedicine.org

References

1. Kopec KT, Kowalski MJ. Metformin-associated lactic acidosis (MALA): case files of the Einstein Medical Center medical toxicology fellowship. J Med Toxicol. 2013;9(1):61-66. https://doi.org/10.1007/s13181-012-0278-3
2. Misbin RI, Green L, Stadel BV, Gueriguian JL, Gubbi A, Fleming GA. Lactic acidosis in patients with diabetes treated with metformin. N Engl J Med. 1998;338(4):265-266. https://doi.org/10.1056/nejm199801223380415
3. Wang HE, Muntner P, Chertow GM, Warnock DG. Acute kidney injury and mortality in hospitalized patients. Am J Nephrol. 2012;35(4):349-355. https://doi.org/10.1159/000337487
4. Chan L, Chaudhary K, Saha A, et al; Mount Sinai COVID Informatics Center (MSCIC), Li L. AKI in hospitalized patients with COVID-19. J Am Soc Nephrol. 2021;32(1):151-160. https://doi.org/10.1681/asn.2020050615
5. US Food and Drug Administration. FDA drug safety communication: FDA revises warnings regarding use of the diabetes medicine metformin in certain patients with reduced kidney function. Updated November 14, 2017. Accessed June 22, 2021. https://www.fda.gov/drugs/drug-safety-and-availability/fda-drug-safety-communication-fda-revises-warnings-regarding-use-diabetes-medicine-metformin-certain
6. American Diabetes Association. 9. Pharmacologic approaches to glycemic treatment: standards of medical care in diabetes—2019. Diabetes Care. 2019;42 (Suppl 1):S90-S102. https://doi.org/10.2337/dc19-s009
7. Moghissi ES, Korytkowski MT, DiNardo M, et al; American Association of Clinical Endocrinologists; American Diabetes Association. Association of Clinical Endocrinologists and American Diabetes Association consensus statement on inpatient glycemic control. Diabetes Care. 2009;32(6):1119-1131. https://doi.org/10.2337/dc09-9029
8. Umpierrez GE, Hellman R, Korytkowski MT, et al; Endocrine Society. Management of hyperglycemia in hospitalized patients in non-critical care setting: an endocrine society clinical practice guideline. J Clin Endocrinol Metab. 2012;97(1):16-38. https://doi.org/10.1210/jc.2011-2098
9. Misbin RI. Phenformin-associated lactic acidosis: pathogenesis and treatment. Ann Intern Med. 1977;87(5):591-595. https://doi.org/10.7326/0003-4819-87-5-591
10. Ekström N, Schiöler L, Svensson AM, et al. Effectiveness and safety of metformin in 51 675 patients with type 2 diabetes and different levels of renal function: a cohort study from the Swedish National Diabetes Register. BMJ Open. 2012;2(4):e001076. https://doi.org/10.1136/bmjopen-2012-001076
11. Lalau JD, Kajbaf F, Bennis Y, Hurtel-Lemaire AS, Belpaire F, De Broe ME. Metformin treatment in patients with type 2 diabetes and chronic kidney disease stages 3A, 3B, or 4. Diabetes Care. 2018;41(3):547-553. https://doi.org/10.2337/dc17-2231
12. Brown JB, Pedula K, Barzilay J, Herson MK, Latare P. Lactic acidosis rates in type 2 diabetes. Diabetes Care. 1998;21(10):1659-1663. https://doi.org/10.2337/diacare.21.10.1659
13. Lalau JD, Race JM. Lactic acidosis in metformin-treated patients. Prognostic value of arterial lactate levels and plasma metformin concentrations. Drug Saf. 1999;20(4):377-384. https://doi.org/10.2165/00002018-199920040-00006
14. Salpeter SR, Greyber E, Pasternak GA, Salpeter Posthumous EE. Risk of fatal and nonfatal lactic acidosis with metformin use in type 2 diabetes mellitus. Cochrane Database Syst Rev. 2010;(1):CD002967. https://doi.org/10.1002/14651858.cd002967.pub3
15. Lazarus B, Wu A, Shin JI, et al. Association of metformin use with risk of lactic acidosis across the range of kidney function: a community-based cohort study. JAMA Intern Med. 2018;178(7):903-910. https://doi.org/10.1001/jamainternmed.2018.0292
16. McDonald JS, Leake CB, McDonald RJ, et al. Acute kidney injury after intravenous versus intra-arterial contrast material administration in a paired cohort. Invest Radiol. 2016;51(12):804-809. https://doi.org/10.1097/rli.0000000000000298
17. Zeller M, Labalette-Bart M, Juliard JM, et al. Metformin and contrast-induced acute kidney injury in diabetic patients treated with primary percutaneous coronary intervention for ST segment elevation myocardial infarction: a multicenter study. Int J Cardiol. 2016;220:137-142. https://doi.org/10.1016/j.ijcard.2016.06.076
18. Goergen SK, Rumbold G, Compton G, Harris C. Systematic review of current guidelines, and their evidence base, on risk of lactic acidosis after administration of contrast medium for patients receiving metformin. Radiology. 2010;254(1):261-269. https://doi.org/10.1148/radiol.09090690
19. Ambrus DB, O’Connor MJ. Things we do for no reason: sliding-scale insulin as monotherapy for glycemic control in hospitalized patients. J Hosp Med. 2019;14(2):114-116. https://doi.org/10.12788/jhm.3109
20. Haltmeier T, Benjamin E, Beale E, Inaba K, Demetriades D. Insulin-treated patients with diabetes mellitus undergoing emergency abdominal surgery have worse outcomes than patients treated with oral agents. World J Surg. 2016;40(7):1575-1582. https://doi.org/10.1007/s00268-016-3469-2

References

1. Kopec KT, Kowalski MJ. Metformin-associated lactic acidosis (MALA): case files of the Einstein Medical Center medical toxicology fellowship. J Med Toxicol. 2013;9(1):61-66. https://doi.org/10.1007/s13181-012-0278-3
2. Misbin RI, Green L, Stadel BV, Gueriguian JL, Gubbi A, Fleming GA. Lactic acidosis in patients with diabetes treated with metformin. N Engl J Med. 1998;338(4):265-266. https://doi.org/10.1056/nejm199801223380415
3. Wang HE, Muntner P, Chertow GM, Warnock DG. Acute kidney injury and mortality in hospitalized patients. Am J Nephrol. 2012;35(4):349-355. https://doi.org/10.1159/000337487
4. Chan L, Chaudhary K, Saha A, et al; Mount Sinai COVID Informatics Center (MSCIC), Li L. AKI in hospitalized patients with COVID-19. J Am Soc Nephrol. 2021;32(1):151-160. https://doi.org/10.1681/asn.2020050615
5. US Food and Drug Administration. FDA drug safety communication: FDA revises warnings regarding use of the diabetes medicine metformin in certain patients with reduced kidney function. Updated November 14, 2017. Accessed June 22, 2021. https://www.fda.gov/drugs/drug-safety-and-availability/fda-drug-safety-communication-fda-revises-warnings-regarding-use-diabetes-medicine-metformin-certain
6. American Diabetes Association. 9. Pharmacologic approaches to glycemic treatment: standards of medical care in diabetes—2019. Diabetes Care. 2019;42 (Suppl 1):S90-S102. https://doi.org/10.2337/dc19-s009
7. Moghissi ES, Korytkowski MT, DiNardo M, et al; American Association of Clinical Endocrinologists; American Diabetes Association. Association of Clinical Endocrinologists and American Diabetes Association consensus statement on inpatient glycemic control. Diabetes Care. 2009;32(6):1119-1131. https://doi.org/10.2337/dc09-9029
8. Umpierrez GE, Hellman R, Korytkowski MT, et al; Endocrine Society. Management of hyperglycemia in hospitalized patients in non-critical care setting: an endocrine society clinical practice guideline. J Clin Endocrinol Metab. 2012;97(1):16-38. https://doi.org/10.1210/jc.2011-2098
9. Misbin RI. Phenformin-associated lactic acidosis: pathogenesis and treatment. Ann Intern Med. 1977;87(5):591-595. https://doi.org/10.7326/0003-4819-87-5-591
10. Ekström N, Schiöler L, Svensson AM, et al. Effectiveness and safety of metformin in 51 675 patients with type 2 diabetes and different levels of renal function: a cohort study from the Swedish National Diabetes Register. BMJ Open. 2012;2(4):e001076. https://doi.org/10.1136/bmjopen-2012-001076
11. Lalau JD, Kajbaf F, Bennis Y, Hurtel-Lemaire AS, Belpaire F, De Broe ME. Metformin treatment in patients with type 2 diabetes and chronic kidney disease stages 3A, 3B, or 4. Diabetes Care. 2018;41(3):547-553. https://doi.org/10.2337/dc17-2231
12. Brown JB, Pedula K, Barzilay J, Herson MK, Latare P. Lactic acidosis rates in type 2 diabetes. Diabetes Care. 1998;21(10):1659-1663. https://doi.org/10.2337/diacare.21.10.1659
13. Lalau JD, Race JM. Lactic acidosis in metformin-treated patients. Prognostic value of arterial lactate levels and plasma metformin concentrations. Drug Saf. 1999;20(4):377-384. https://doi.org/10.2165/00002018-199920040-00006
14. Salpeter SR, Greyber E, Pasternak GA, Salpeter Posthumous EE. Risk of fatal and nonfatal lactic acidosis with metformin use in type 2 diabetes mellitus. Cochrane Database Syst Rev. 2010;(1):CD002967. https://doi.org/10.1002/14651858.cd002967.pub3
15. Lazarus B, Wu A, Shin JI, et al. Association of metformin use with risk of lactic acidosis across the range of kidney function: a community-based cohort study. JAMA Intern Med. 2018;178(7):903-910. https://doi.org/10.1001/jamainternmed.2018.0292
16. McDonald JS, Leake CB, McDonald RJ, et al. Acute kidney injury after intravenous versus intra-arterial contrast material administration in a paired cohort. Invest Radiol. 2016;51(12):804-809. https://doi.org/10.1097/rli.0000000000000298
17. Zeller M, Labalette-Bart M, Juliard JM, et al. Metformin and contrast-induced acute kidney injury in diabetic patients treated with primary percutaneous coronary intervention for ST segment elevation myocardial infarction: a multicenter study. Int J Cardiol. 2016;220:137-142. https://doi.org/10.1016/j.ijcard.2016.06.076
18. Goergen SK, Rumbold G, Compton G, Harris C. Systematic review of current guidelines, and their evidence base, on risk of lactic acidosis after administration of contrast medium for patients receiving metformin. Radiology. 2010;254(1):261-269. https://doi.org/10.1148/radiol.09090690
19. Ambrus DB, O’Connor MJ. Things we do for no reason: sliding-scale insulin as monotherapy for glycemic control in hospitalized patients. J Hosp Med. 2019;14(2):114-116. https://doi.org/10.12788/jhm.3109
20. Haltmeier T, Benjamin E, Beale E, Inaba K, Demetriades D. Insulin-treated patients with diabetes mellitus undergoing emergency abdominal surgery have worse outcomes than patients treated with oral agents. World J Surg. 2016;40(7):1575-1582. https://doi.org/10.1007/s00268-016-3469-2

Publications
Publications
Topics
Article Type
Display Headline
Things We Do For No Reason™: Routinely Holding Metformin in the Hospital
Display Headline
Things We Do For No Reason™: Routinely Holding Metformin in the Hospital
Sections
Article Source

© 2021 Society of Hospital Medicine

Citation Override
J Hosp Med. Published Online First August 18, 2021. DOI: 10.12788/jhm.3644
Disallow All Ads
Correspondence Location
David A Cohen, MD; Email: DavidACohen.MD@rutgers.edu; Telephone: 323-828-5637.
Content Gating
Gated (full article locked unless allowed per User)
Alternative CME
Disqus Comments
Default
Use ProPublica
Hide sidebar & use full width
render the right sidebar.
Conference Recap Checkbox
Not Conference Recap
Clinical Edge
Display the Slideshow in this Article
Gating Strategy
First Page Free
Medscape Article
Display survey writer
Reuters content
Disable Inline Native ads
WebMD Article
Article PDF Media
Image
Teambase ID
18001DC1.SIG
Disable zoom
Off

Things We Do for No Reason™: Obtaining Urine Testing in Older Adults With Delirium Without Signs or Symptoms of Urinary Tract Infection

Article Type
Changed
Tue, 08/31/2021 - 11:36
Display Headline
Things We Do for No Reason™: Obtaining Urine Testing in Older Adults With Delirium Without Signs or Symptoms of Urinary Tract Infection

Inspired by the ABIM Foundation’s Choosing Wisely® campaign, the “Things We Do for No Reason” (TWDFNR) series reviews practices that have become common parts of hospital care but may provide little value to our patients. Practices reviewed in the TWDFNR series do not represent clear-cut conclusions or clinical practice standards but are meant as a starting place for research and active discussions among hospitalists and patients. We invite you to be part of that discussion.

CLINICAL SCENARIO

A 78-year-old female nursing home resident presents to the emergency department for evaluation of a several-hour history of confusion and restlessness. The patient is accompanied by one of her caregivers from the nursing home. Initial evaluation reveals an awake but inattentive, disoriented, and agitated woman who can answer basic questions appropriately. The caregiver denies the patient having had any antecedent concerns, such as pain with urination, abdominal pain, subjective fevers, chills, or night sweats. Vital signs include a temperature of 37.5 °C (99.5 °F), heart rate of 90 beats per minute, blood pressure of 110/60 mm Hg, respiratory rate of 14 breaths per minute, and oxygen saturation of 98% on room air. The patient has a normal lung and abdominal exam without any suprapubic or flank tenderness. There is no Foley catheter in place.

BACKGROUND

Delirium, defined by the World Health Organization’s 10th revision of the International Classification of Diseases as “an etiologically nonspecific organic cerebral syndrome characterized by concurrent disturbances of consciousness and attention, perception, thinking, memory, psychomotor behavior, emotion, and the sleep-wake schedule,” is associated with poor clinical outcomes in older patients.1,2 Mental status changes, which can arise rapidly over the course of hours to days, often fluctuate, with most cases resolving within days of onset.3 In the United States, more than 2.6 million adults aged 65 years and older develop delirium each year, accounting for an estimated $38 to $152 billion in annual healthcare expenditures.4

WHY YOU MIGHT THINK URINE TESTING IS HELPFUL IN OLDER ADULTS WITH DELIRIUM WHO HAVE NO SIGNS OR SYMPTOMS OF URINARY TRACT INFECTION

Some clinicians believe that the evaluation for delirium should include an empiric urinary infectious workup with urinalysis and/or urine cultures, even in the absence of local genitourinary symptoms or other signs of infection. In fact, altered mental status is the most common indication for ordering a urine culture in older adult patients.5

Urinary tract infections (UTIs) account for almost 25% of all reported infections in older patients, with delirium occurring in up to 30% of this patient population.6 As one study demonstrated, given this population’s very high prevalence of asymptomatic bacteriuria (ASB), urine studies sent during a delirium work-up often yield positive findings (defined as ≥105 colony-forming units [CFU]/mL [≥108 CFU/L]) in older patients with no signs or symptoms attributable to UTI.7 The incidence of ASB increases significantly with age, with prevalence estimated to be between 6% and 10% in women older than 60 years and approximately 5% in men older than 65 years.5 Among older patients residing in long-term care facilities, up to 50% of female residents and up to 40% of male residents have ASB.8 These findings, in part, created the common perception of causation between UTI and delirium.

WHY YOU SHOULD NOT OBTAIN URINE TESTING IN OLDER ADULTS WITH DELIRIUM IF THEY HAVE NO SIGNS OR SYMPTOMS OF URINARY TRACT INFECTION

A recent systematic review demonstrated that there is insufficient evidence to associate UTI with acute confusion in older patients.9 The Centers for Disease Control and Prevention’s National Health Safety Network notes that at least one of the following criteria must be present for the diagnosis of UTI in noncatheterized patients: fever (>38 °C), suprapubic tenderness, costovertebral angle tenderness, urinary frequency, urinary urgency, or dysuria.10 Recent studies have identified that ASB—by definition, without dysuria, frequency, bladder discomfort, or fever—is an unlikely cause of delirium.6,11

The 2019 Infectious Diseases Society of America (IDSA) practice guidelines suggest that clinicians not screen for ASB in older functionally or cognitively impaired patients with no local genitourinary symptoms or other signs of infection. The IDSA acknowledges that the potential adverse outcomes of antimicrobial therapy, including Clostridioides difficile infection, increased antimicrobial resistance, or adverse drug effects, outweigh the potential benefit of treatment given the absence of evidence that such treatment improves outcomes for this vulnerable patient population (strong recommendation, very low-quality evidence).12 Per the IDSA guidelines, recommendations are strong when there is “moderate- or high-quality evidence that the desirable consequences outweigh the undesirable consequences for a course of action” and “may also be strong when there is high-quality evidence of harm and benefits are uncertain (ie, low or very low quality),” as in this case scenario. Studies of older institutionalized and hospitalized patients have found that ASB often results in inappropriate antimicrobial use with limited benefit.7,13,14 In addition to noting the lack of benefit from treatment, these studies have found that these patients treated with antimicrobials have worse outcomes when compared to untreated patients with ASB. One study of hospitalized patients treated for ASB concluded that participants given antimicrobial agents experienced longer durations of hospitalization, with no benefits from treatment.13 Moreover, another study identified poor long-term functional recovery in patients treated for ASB.14

Overtreatment also has public health implications given that it may increase the prevalence of multidrug-resistant bacteria in long-term care facilities.15 One recent study of nursing home residents demonstrated an association between bacteriuria, increased antibiotic use, and subsequent isolation of multidrug-resistant gram-negative organisms.16 The increased prevalence of these organisms limits options for oral antibiotic therapies in the outpatient setting, potentially leading to increased healthcare utilization and further harms relating to institutionalization in this vulnerable patient population. In light of the ethical concept of nonmaleficence, recognizing the potential harms of treating ASB without clear benefit is important for clinicians to take into account when considering urinalysis in this patient population.

In addition, obtaining a urine culture in an older patient with no signs or symptoms of UTI may lead to premature closure from a diagnostic perspective, resulting in missed diagnoses during clinical evaluation. A missed alternative diagnosis could then cause additional, ongoing harm to the patient if left untreated. Subsequent harms from delayed treatment can thus compound the direct harms and added costs incurred by inappropriate testing and treatment of patients with delirium.

Since 2013, the American Geriatrics Society (AGS) has recommended against the use of antimicrobials in older patients with no urinary tract symptoms, stating that “Antimicrobial treatment studies for asymptomatic bacteriuria in older adults demonstrate no benefits and show increased adverse antimicrobial effects.”17 The IDSA practice guidelines state the following: “In older patients with functional and/or cognitive impairment with bacteriuria and delirium (acute mental status change, confusion) and without local genitourinary symptoms or other systemic signs of infection (eg, fever or hemodynamic instability), we recommend assessment for other causes and careful observation rather than antimicrobial treatment (strong recommendation, very low-quality evidence).”12

WHEN YOU SHOULD OBTAIN URINALYSIS FOR OLDER ADULTS WITH DELIRIUM

Older patients presenting with confusion in the setting of recognized symptoms of UTI (eg, acute dysuria, urinary urgency or frequency) warrant urinalysis and urine culture. Additionally, urinalysis and urine cultures may be warranted to assess for UTI—even in the absence of a localizing source—in older patients with signs and symptoms of delirium who also exhibit systemic signs of infection (eg, fever, leukocytosis, hemodynamic instability).12

WHAT YOU SHOULD DO INSTEAD

Initial evaluation of an older patient with delirium should include a thorough review of their recent history and baseline mental status with a knowledgeable informant, a careful physical and neurologic examination, and laboratory studies to determine the presence of electrolyte or metabolic derangements as well as infection and organ failure.4 Clinicians should take into account nonmodifiable risk factors for delirium and conduct a careful review of the time course of changes in mental status and modifiable risk factors, including environment, sleep deprivation, medications, immobilization, and sensory impairments.18

To manage delirium in older patients, clinicians should identify reversible causes of the delirium and minimize modifiable exacerbating factors (eg, sensory impairment, sleep deprivation) in the immediate environment of the patient. They should also carefully review medications that may contribute to delirium, using tools such as the AGS Beers Criteria to identify high-risk medications and concerning medication combinations.19 Patients who develop local or systemic signs of infection (ie, fevers, chills, dysuria) should undergo appropriate testing, including urinalysis if there is clinical suspicion for urinary etiology.

RECOMMENDATIONS

  • For older patients presenting with delirium without localized urinary symptoms or systemic signs of a serious infection, forgo routine ordering of urinalysis and urine culture.
  • For older patients presenting with delirium and localized or systemic signs of infection, routine urine studies and antimicrobial therapy may be appropriate.
  • For older patients presenting with delirium without localized symptoms or systemic signs of serious infection, attempt to first identify the cause of the change in mental status by obtaining history from a reliable informant, performing a thorough physical and neurologic examination, and evaluating for metabolic and electrolyte derangements.

CONCLUSION

Returning to the clinical scenario, older patients presenting with signs and symptoms of delirium should undergo further work-up to determine underlying causes for their altered mental status. The patient’s history, ideally obtained from a knowledgeable informant, should offer insight into her baseline mental status and risk factors for delirium. This should be followed by a careful physical and neurologic examination, and evaluation for electrolyte, metabolic, and other derangements. In patients without localized or systemic signs of infection, routine urine testing and treatment of bacteriuria should be avoided.

Do you think this is a low-value practice? Is this truly a “Thing We Do for No Reason”? Share what you do in your practice and join in the conversation online by retweeting it on Twitter (#TWDFNR) and liking it on Facebook. We invite you to pro­pose ideas for other “Things We Do for No Reason" topics by emailing TWDFNR@hospitalmedicine.org

References

1. World Health Organization. 2018 International Classification of Diseases for Mortality and Morbidity Statistics. 11th Rev. Published September 20, 2020. Accessed April 12, 2021. https://icd.who.int/browse10/2019/en#/F04
2. Witlox J, Eurelings LS, de Jonghe JFM, Kalisvaart KJ, Eikelenboom P, van Gool WA. Delirium in elderly patients and the risk of postdischarge mortality, institutionalization and dementia: a meta-analysis. JAMA. 2010;304(4):443-451. https://doi.org/10.1001/jama.2010.1013
3. Young J, Inouye SK. Delirium in older people. BMJ. 2007;334(7598):842-846. https://doi.org/10.1136/bmj.39169.706574.ad
4. Oh ES, Fong TG, Hshieh TT, Inouye SK. Delirium in older persons: advances in diagnosis and treatment. JAMA. 2017;318(12):1161-1174. https://doi.org/10.1001/jama.2017.12067
5. R McKenzie, M Stewart, M. Bellantoni, TE Finucane. Bacteriuria in Individuals who become delirious. Am J Med. 2014;127(4):255-257. https://doi.org/10.1016/j.amjmed.2013.10.016
6. Balogun S, Philbrick JT. Delirium, a symptom of UTI in the elderly: fact or fable? A systematic review. Can Geriatr J. 2013;17(1):22-26. https://doi.org/10.5770/cgj.17.90
7. Nicolle LE, Mayhew WJ, Bryan L. Prospective randomized comparison of therapy and no therapy for asymptomatic bacteriuria in institutionalized elderly women. Am J Med. 1987;83(1):27-33. https://doi.org/10.1016/0002-9343(87)90493-1
8. Zalmanovici Trestioreanu A, Lador A, Sauerbrun-Cutler MT, Leibovici L. Antibiotics for asymptomatic bacteriuria. Cochrane Database Syst Rev. 2015;4:CD009534. https://doi.org/10.1002/14651858.cd009534.pub2
9. Mayne S, Bowden A, Sundvall PD, Gunnarsson R. The scientific evidence for a potential link between confusion and urinary tract infection in the elderly is still confusing – a systematic literature review. BMC Geriatr. 2019;19(1):32. https://doi.org/10.1186/s12877-019-1049-7
10. Centers for Disease Control and Prevention. Urinary tract infection (catheter-associated urinary tract infection [CAUTI] and non-catheter-associated urinary tract infection [UTI]) events. In: National Health Safety Network (NHSN) Patient Safety Component Manual. 2021:7-5. Published January 2021. Accessed April 12, 2021. https://www.cdc.gov/nhsn/pdfs/pscmanual/pcsmanual_current.pdf
11. Gupta K, Grigoryan L, Trautner B. 2017. Urinary tract infection. Ann Intern Med. 2017;167(7):ITC49-ITC64. https://doi.org/10.7326/aitc201710030
12. Nicolle LE, Gupta K, Bradley SF, et al. 2019. Clinical practice guideline for the management of asymptomatic bacteriuria: 2019 update by the Infectious Diseases Society of America. Clin Infect Dis. 2019;68(10):1611-1615. https://doi.org/10.1093/cid/ciz021
13. Petty LA, Vaughn VM, Flanders SA, et al. Risk factors and outcomes associated with treatment of asymptomatic bacteriuria in hospitalized patients. JAMA Intern Med. 2019;179(11):1519-1527. https://doi.org/10.1001/jamainternmed.2019.2871
14. Dasgupta M, Brymer C, Elsayed S. 2017. Treatment of asymptomatic UTI in older delirious medical in-patients: a prospective cohort study. Arch Gerontol Geriatr. 2017;72:127-134. https://doi.org/10.1016/j.archger.2017.05.010
15. Pop-Vicas A, Mitchell SL, Kandel R, Schreiber R, D’Agata EMC. Multidrug-resistant gram-negative bacteria in a long-term care facility: prevalence and risk factors. J Am Geriatr Soc. 2008;56(7):1276-1280. https://doi.org/10.1111/j.1532-5415.2008.01787.x
16. Das R, Towle V, Van Ness PH, Juthani-Mehta M. 2011. Adverse outcomes in nursing home residents with increased episodes of observed bacteriuria. Infect Control Hosp Epidemiol. 2011;32(1):84-86. https://doi.org/10.1086/657664
17. American Board of Internal Medicine. Choosing Wisely. American Geriatrics Society. Antimicrobials to treat bacteriuria in older adults.” Published February 21, 2013. Accessed April 12, 2021. www.choosingwisely.org/clinician-lists/american-geriatrics-society-antimicrobials-to-treat-bacteriuria-in-older-adults/
18. Fong TG, Tulebaev SR, Inouye SK. Delirium in elderly adults: diagnosis, prevention and treatment. Nat Rev Neurol. 2009;5(4):210-220. https://doi.org/10.1038/nrneurol.2009.24
19. 2019 American Geriatrics Society Beers Criteria Update Expert Panel. American Geriatrics Society 2019 update AGS Beers Criteria for potential inappropriate medication use in older adults. J Am Geriatr Soc. 2019;67(4):674-694. https://doi.org/10.1111/jgs.15767

Article PDF
Author and Disclosure Information

1Johns Hopkins University School of Medicine, Baltimore, Maryland; 2Johns Hopkins Bayview Medical Center, Baltimore, Maryland; 3University of Miami, Miller School of Medicine, Miami, Florida; 4Jackson Memorial Hospital, Miami, Florida.

Disclosures
The authors have no conflicts to disclose.

Issue
Journal of Hospital Medicine 16(9)
Publications
Topics
Page Number
557-559. Published Online First July 26, 2021
Sections
Author and Disclosure Information

1Johns Hopkins University School of Medicine, Baltimore, Maryland; 2Johns Hopkins Bayview Medical Center, Baltimore, Maryland; 3University of Miami, Miller School of Medicine, Miami, Florida; 4Jackson Memorial Hospital, Miami, Florida.

Disclosures
The authors have no conflicts to disclose.

Author and Disclosure Information

1Johns Hopkins University School of Medicine, Baltimore, Maryland; 2Johns Hopkins Bayview Medical Center, Baltimore, Maryland; 3University of Miami, Miller School of Medicine, Miami, Florida; 4Jackson Memorial Hospital, Miami, Florida.

Disclosures
The authors have no conflicts to disclose.

Article PDF
Article PDF
Related Articles

Inspired by the ABIM Foundation’s Choosing Wisely® campaign, the “Things We Do for No Reason” (TWDFNR) series reviews practices that have become common parts of hospital care but may provide little value to our patients. Practices reviewed in the TWDFNR series do not represent clear-cut conclusions or clinical practice standards but are meant as a starting place for research and active discussions among hospitalists and patients. We invite you to be part of that discussion.

CLINICAL SCENARIO

A 78-year-old female nursing home resident presents to the emergency department for evaluation of a several-hour history of confusion and restlessness. The patient is accompanied by one of her caregivers from the nursing home. Initial evaluation reveals an awake but inattentive, disoriented, and agitated woman who can answer basic questions appropriately. The caregiver denies the patient having had any antecedent concerns, such as pain with urination, abdominal pain, subjective fevers, chills, or night sweats. Vital signs include a temperature of 37.5 °C (99.5 °F), heart rate of 90 beats per minute, blood pressure of 110/60 mm Hg, respiratory rate of 14 breaths per minute, and oxygen saturation of 98% on room air. The patient has a normal lung and abdominal exam without any suprapubic or flank tenderness. There is no Foley catheter in place.

BACKGROUND

Delirium, defined by the World Health Organization’s 10th revision of the International Classification of Diseases as “an etiologically nonspecific organic cerebral syndrome characterized by concurrent disturbances of consciousness and attention, perception, thinking, memory, psychomotor behavior, emotion, and the sleep-wake schedule,” is associated with poor clinical outcomes in older patients.1,2 Mental status changes, which can arise rapidly over the course of hours to days, often fluctuate, with most cases resolving within days of onset.3 In the United States, more than 2.6 million adults aged 65 years and older develop delirium each year, accounting for an estimated $38 to $152 billion in annual healthcare expenditures.4

WHY YOU MIGHT THINK URINE TESTING IS HELPFUL IN OLDER ADULTS WITH DELIRIUM WHO HAVE NO SIGNS OR SYMPTOMS OF URINARY TRACT INFECTION

Some clinicians believe that the evaluation for delirium should include an empiric urinary infectious workup with urinalysis and/or urine cultures, even in the absence of local genitourinary symptoms or other signs of infection. In fact, altered mental status is the most common indication for ordering a urine culture in older adult patients.5

Urinary tract infections (UTIs) account for almost 25% of all reported infections in older patients, with delirium occurring in up to 30% of this patient population.6 As one study demonstrated, given this population’s very high prevalence of asymptomatic bacteriuria (ASB), urine studies sent during a delirium work-up often yield positive findings (defined as ≥105 colony-forming units [CFU]/mL [≥108 CFU/L]) in older patients with no signs or symptoms attributable to UTI.7 The incidence of ASB increases significantly with age, with prevalence estimated to be between 6% and 10% in women older than 60 years and approximately 5% in men older than 65 years.5 Among older patients residing in long-term care facilities, up to 50% of female residents and up to 40% of male residents have ASB.8 These findings, in part, created the common perception of causation between UTI and delirium.

WHY YOU SHOULD NOT OBTAIN URINE TESTING IN OLDER ADULTS WITH DELIRIUM IF THEY HAVE NO SIGNS OR SYMPTOMS OF URINARY TRACT INFECTION

A recent systematic review demonstrated that there is insufficient evidence to associate UTI with acute confusion in older patients.9 The Centers for Disease Control and Prevention’s National Health Safety Network notes that at least one of the following criteria must be present for the diagnosis of UTI in noncatheterized patients: fever (>38 °C), suprapubic tenderness, costovertebral angle tenderness, urinary frequency, urinary urgency, or dysuria.10 Recent studies have identified that ASB—by definition, without dysuria, frequency, bladder discomfort, or fever—is an unlikely cause of delirium.6,11

The 2019 Infectious Diseases Society of America (IDSA) practice guidelines suggest that clinicians not screen for ASB in older functionally or cognitively impaired patients with no local genitourinary symptoms or other signs of infection. The IDSA acknowledges that the potential adverse outcomes of antimicrobial therapy, including Clostridioides difficile infection, increased antimicrobial resistance, or adverse drug effects, outweigh the potential benefit of treatment given the absence of evidence that such treatment improves outcomes for this vulnerable patient population (strong recommendation, very low-quality evidence).12 Per the IDSA guidelines, recommendations are strong when there is “moderate- or high-quality evidence that the desirable consequences outweigh the undesirable consequences for a course of action” and “may also be strong when there is high-quality evidence of harm and benefits are uncertain (ie, low or very low quality),” as in this case scenario. Studies of older institutionalized and hospitalized patients have found that ASB often results in inappropriate antimicrobial use with limited benefit.7,13,14 In addition to noting the lack of benefit from treatment, these studies have found that these patients treated with antimicrobials have worse outcomes when compared to untreated patients with ASB. One study of hospitalized patients treated for ASB concluded that participants given antimicrobial agents experienced longer durations of hospitalization, with no benefits from treatment.13 Moreover, another study identified poor long-term functional recovery in patients treated for ASB.14

Overtreatment also has public health implications given that it may increase the prevalence of multidrug-resistant bacteria in long-term care facilities.15 One recent study of nursing home residents demonstrated an association between bacteriuria, increased antibiotic use, and subsequent isolation of multidrug-resistant gram-negative organisms.16 The increased prevalence of these organisms limits options for oral antibiotic therapies in the outpatient setting, potentially leading to increased healthcare utilization and further harms relating to institutionalization in this vulnerable patient population. In light of the ethical concept of nonmaleficence, recognizing the potential harms of treating ASB without clear benefit is important for clinicians to take into account when considering urinalysis in this patient population.

In addition, obtaining a urine culture in an older patient with no signs or symptoms of UTI may lead to premature closure from a diagnostic perspective, resulting in missed diagnoses during clinical evaluation. A missed alternative diagnosis could then cause additional, ongoing harm to the patient if left untreated. Subsequent harms from delayed treatment can thus compound the direct harms and added costs incurred by inappropriate testing and treatment of patients with delirium.

Since 2013, the American Geriatrics Society (AGS) has recommended against the use of antimicrobials in older patients with no urinary tract symptoms, stating that “Antimicrobial treatment studies for asymptomatic bacteriuria in older adults demonstrate no benefits and show increased adverse antimicrobial effects.”17 The IDSA practice guidelines state the following: “In older patients with functional and/or cognitive impairment with bacteriuria and delirium (acute mental status change, confusion) and without local genitourinary symptoms or other systemic signs of infection (eg, fever or hemodynamic instability), we recommend assessment for other causes and careful observation rather than antimicrobial treatment (strong recommendation, very low-quality evidence).”12

WHEN YOU SHOULD OBTAIN URINALYSIS FOR OLDER ADULTS WITH DELIRIUM

Older patients presenting with confusion in the setting of recognized symptoms of UTI (eg, acute dysuria, urinary urgency or frequency) warrant urinalysis and urine culture. Additionally, urinalysis and urine cultures may be warranted to assess for UTI—even in the absence of a localizing source—in older patients with signs and symptoms of delirium who also exhibit systemic signs of infection (eg, fever, leukocytosis, hemodynamic instability).12

WHAT YOU SHOULD DO INSTEAD

Initial evaluation of an older patient with delirium should include a thorough review of their recent history and baseline mental status with a knowledgeable informant, a careful physical and neurologic examination, and laboratory studies to determine the presence of electrolyte or metabolic derangements as well as infection and organ failure.4 Clinicians should take into account nonmodifiable risk factors for delirium and conduct a careful review of the time course of changes in mental status and modifiable risk factors, including environment, sleep deprivation, medications, immobilization, and sensory impairments.18

To manage delirium in older patients, clinicians should identify reversible causes of the delirium and minimize modifiable exacerbating factors (eg, sensory impairment, sleep deprivation) in the immediate environment of the patient. They should also carefully review medications that may contribute to delirium, using tools such as the AGS Beers Criteria to identify high-risk medications and concerning medication combinations.19 Patients who develop local or systemic signs of infection (ie, fevers, chills, dysuria) should undergo appropriate testing, including urinalysis if there is clinical suspicion for urinary etiology.

RECOMMENDATIONS

  • For older patients presenting with delirium without localized urinary symptoms or systemic signs of a serious infection, forgo routine ordering of urinalysis and urine culture.
  • For older patients presenting with delirium and localized or systemic signs of infection, routine urine studies and antimicrobial therapy may be appropriate.
  • For older patients presenting with delirium without localized symptoms or systemic signs of serious infection, attempt to first identify the cause of the change in mental status by obtaining history from a reliable informant, performing a thorough physical and neurologic examination, and evaluating for metabolic and electrolyte derangements.

CONCLUSION

Returning to the clinical scenario, older patients presenting with signs and symptoms of delirium should undergo further work-up to determine underlying causes for their altered mental status. The patient’s history, ideally obtained from a knowledgeable informant, should offer insight into her baseline mental status and risk factors for delirium. This should be followed by a careful physical and neurologic examination, and evaluation for electrolyte, metabolic, and other derangements. In patients without localized or systemic signs of infection, routine urine testing and treatment of bacteriuria should be avoided.

Do you think this is a low-value practice? Is this truly a “Thing We Do for No Reason”? Share what you do in your practice and join in the conversation online by retweeting it on Twitter (#TWDFNR) and liking it on Facebook. We invite you to pro­pose ideas for other “Things We Do for No Reason" topics by emailing TWDFNR@hospitalmedicine.org

Inspired by the ABIM Foundation’s Choosing Wisely® campaign, the “Things We Do for No Reason” (TWDFNR) series reviews practices that have become common parts of hospital care but may provide little value to our patients. Practices reviewed in the TWDFNR series do not represent clear-cut conclusions or clinical practice standards but are meant as a starting place for research and active discussions among hospitalists and patients. We invite you to be part of that discussion.

CLINICAL SCENARIO

A 78-year-old female nursing home resident presents to the emergency department for evaluation of a several-hour history of confusion and restlessness. The patient is accompanied by one of her caregivers from the nursing home. Initial evaluation reveals an awake but inattentive, disoriented, and agitated woman who can answer basic questions appropriately. The caregiver denies the patient having had any antecedent concerns, such as pain with urination, abdominal pain, subjective fevers, chills, or night sweats. Vital signs include a temperature of 37.5 °C (99.5 °F), heart rate of 90 beats per minute, blood pressure of 110/60 mm Hg, respiratory rate of 14 breaths per minute, and oxygen saturation of 98% on room air. The patient has a normal lung and abdominal exam without any suprapubic or flank tenderness. There is no Foley catheter in place.

BACKGROUND

Delirium, defined by the World Health Organization’s 10th revision of the International Classification of Diseases as “an etiologically nonspecific organic cerebral syndrome characterized by concurrent disturbances of consciousness and attention, perception, thinking, memory, psychomotor behavior, emotion, and the sleep-wake schedule,” is associated with poor clinical outcomes in older patients.1,2 Mental status changes, which can arise rapidly over the course of hours to days, often fluctuate, with most cases resolving within days of onset.3 In the United States, more than 2.6 million adults aged 65 years and older develop delirium each year, accounting for an estimated $38 to $152 billion in annual healthcare expenditures.4

WHY YOU MIGHT THINK URINE TESTING IS HELPFUL IN OLDER ADULTS WITH DELIRIUM WHO HAVE NO SIGNS OR SYMPTOMS OF URINARY TRACT INFECTION

Some clinicians believe that the evaluation for delirium should include an empiric urinary infectious workup with urinalysis and/or urine cultures, even in the absence of local genitourinary symptoms or other signs of infection. In fact, altered mental status is the most common indication for ordering a urine culture in older adult patients.5

Urinary tract infections (UTIs) account for almost 25% of all reported infections in older patients, with delirium occurring in up to 30% of this patient population.6 As one study demonstrated, given this population’s very high prevalence of asymptomatic bacteriuria (ASB), urine studies sent during a delirium work-up often yield positive findings (defined as ≥105 colony-forming units [CFU]/mL [≥108 CFU/L]) in older patients with no signs or symptoms attributable to UTI.7 The incidence of ASB increases significantly with age, with prevalence estimated to be between 6% and 10% in women older than 60 years and approximately 5% in men older than 65 years.5 Among older patients residing in long-term care facilities, up to 50% of female residents and up to 40% of male residents have ASB.8 These findings, in part, created the common perception of causation between UTI and delirium.

WHY YOU SHOULD NOT OBTAIN URINE TESTING IN OLDER ADULTS WITH DELIRIUM IF THEY HAVE NO SIGNS OR SYMPTOMS OF URINARY TRACT INFECTION

A recent systematic review demonstrated that there is insufficient evidence to associate UTI with acute confusion in older patients.9 The Centers for Disease Control and Prevention’s National Health Safety Network notes that at least one of the following criteria must be present for the diagnosis of UTI in noncatheterized patients: fever (>38 °C), suprapubic tenderness, costovertebral angle tenderness, urinary frequency, urinary urgency, or dysuria.10 Recent studies have identified that ASB—by definition, without dysuria, frequency, bladder discomfort, or fever—is an unlikely cause of delirium.6,11

The 2019 Infectious Diseases Society of America (IDSA) practice guidelines suggest that clinicians not screen for ASB in older functionally or cognitively impaired patients with no local genitourinary symptoms or other signs of infection. The IDSA acknowledges that the potential adverse outcomes of antimicrobial therapy, including Clostridioides difficile infection, increased antimicrobial resistance, or adverse drug effects, outweigh the potential benefit of treatment given the absence of evidence that such treatment improves outcomes for this vulnerable patient population (strong recommendation, very low-quality evidence).12 Per the IDSA guidelines, recommendations are strong when there is “moderate- or high-quality evidence that the desirable consequences outweigh the undesirable consequences for a course of action” and “may also be strong when there is high-quality evidence of harm and benefits are uncertain (ie, low or very low quality),” as in this case scenario. Studies of older institutionalized and hospitalized patients have found that ASB often results in inappropriate antimicrobial use with limited benefit.7,13,14 In addition to noting the lack of benefit from treatment, these studies have found that these patients treated with antimicrobials have worse outcomes when compared to untreated patients with ASB. One study of hospitalized patients treated for ASB concluded that participants given antimicrobial agents experienced longer durations of hospitalization, with no benefits from treatment.13 Moreover, another study identified poor long-term functional recovery in patients treated for ASB.14

Overtreatment also has public health implications given that it may increase the prevalence of multidrug-resistant bacteria in long-term care facilities.15 One recent study of nursing home residents demonstrated an association between bacteriuria, increased antibiotic use, and subsequent isolation of multidrug-resistant gram-negative organisms.16 The increased prevalence of these organisms limits options for oral antibiotic therapies in the outpatient setting, potentially leading to increased healthcare utilization and further harms relating to institutionalization in this vulnerable patient population. In light of the ethical concept of nonmaleficence, recognizing the potential harms of treating ASB without clear benefit is important for clinicians to take into account when considering urinalysis in this patient population.

In addition, obtaining a urine culture in an older patient with no signs or symptoms of UTI may lead to premature closure from a diagnostic perspective, resulting in missed diagnoses during clinical evaluation. A missed alternative diagnosis could then cause additional, ongoing harm to the patient if left untreated. Subsequent harms from delayed treatment can thus compound the direct harms and added costs incurred by inappropriate testing and treatment of patients with delirium.

Since 2013, the American Geriatrics Society (AGS) has recommended against the use of antimicrobials in older patients with no urinary tract symptoms, stating that “Antimicrobial treatment studies for asymptomatic bacteriuria in older adults demonstrate no benefits and show increased adverse antimicrobial effects.”17 The IDSA practice guidelines state the following: “In older patients with functional and/or cognitive impairment with bacteriuria and delirium (acute mental status change, confusion) and without local genitourinary symptoms or other systemic signs of infection (eg, fever or hemodynamic instability), we recommend assessment for other causes and careful observation rather than antimicrobial treatment (strong recommendation, very low-quality evidence).”12

WHEN YOU SHOULD OBTAIN URINALYSIS FOR OLDER ADULTS WITH DELIRIUM

Older patients presenting with confusion in the setting of recognized symptoms of UTI (eg, acute dysuria, urinary urgency or frequency) warrant urinalysis and urine culture. Additionally, urinalysis and urine cultures may be warranted to assess for UTI—even in the absence of a localizing source—in older patients with signs and symptoms of delirium who also exhibit systemic signs of infection (eg, fever, leukocytosis, hemodynamic instability).12

WHAT YOU SHOULD DO INSTEAD

Initial evaluation of an older patient with delirium should include a thorough review of their recent history and baseline mental status with a knowledgeable informant, a careful physical and neurologic examination, and laboratory studies to determine the presence of electrolyte or metabolic derangements as well as infection and organ failure.4 Clinicians should take into account nonmodifiable risk factors for delirium and conduct a careful review of the time course of changes in mental status and modifiable risk factors, including environment, sleep deprivation, medications, immobilization, and sensory impairments.18

To manage delirium in older patients, clinicians should identify reversible causes of the delirium and minimize modifiable exacerbating factors (eg, sensory impairment, sleep deprivation) in the immediate environment of the patient. They should also carefully review medications that may contribute to delirium, using tools such as the AGS Beers Criteria to identify high-risk medications and concerning medication combinations.19 Patients who develop local or systemic signs of infection (ie, fevers, chills, dysuria) should undergo appropriate testing, including urinalysis if there is clinical suspicion for urinary etiology.

RECOMMENDATIONS

  • For older patients presenting with delirium without localized urinary symptoms or systemic signs of a serious infection, forgo routine ordering of urinalysis and urine culture.
  • For older patients presenting with delirium and localized or systemic signs of infection, routine urine studies and antimicrobial therapy may be appropriate.
  • For older patients presenting with delirium without localized symptoms or systemic signs of serious infection, attempt to first identify the cause of the change in mental status by obtaining history from a reliable informant, performing a thorough physical and neurologic examination, and evaluating for metabolic and electrolyte derangements.

CONCLUSION

Returning to the clinical scenario, older patients presenting with signs and symptoms of delirium should undergo further work-up to determine underlying causes for their altered mental status. The patient’s history, ideally obtained from a knowledgeable informant, should offer insight into her baseline mental status and risk factors for delirium. This should be followed by a careful physical and neurologic examination, and evaluation for electrolyte, metabolic, and other derangements. In patients without localized or systemic signs of infection, routine urine testing and treatment of bacteriuria should be avoided.

Do you think this is a low-value practice? Is this truly a “Thing We Do for No Reason”? Share what you do in your practice and join in the conversation online by retweeting it on Twitter (#TWDFNR) and liking it on Facebook. We invite you to pro­pose ideas for other “Things We Do for No Reason" topics by emailing TWDFNR@hospitalmedicine.org

References

1. World Health Organization. 2018 International Classification of Diseases for Mortality and Morbidity Statistics. 11th Rev. Published September 20, 2020. Accessed April 12, 2021. https://icd.who.int/browse10/2019/en#/F04
2. Witlox J, Eurelings LS, de Jonghe JFM, Kalisvaart KJ, Eikelenboom P, van Gool WA. Delirium in elderly patients and the risk of postdischarge mortality, institutionalization and dementia: a meta-analysis. JAMA. 2010;304(4):443-451. https://doi.org/10.1001/jama.2010.1013
3. Young J, Inouye SK. Delirium in older people. BMJ. 2007;334(7598):842-846. https://doi.org/10.1136/bmj.39169.706574.ad
4. Oh ES, Fong TG, Hshieh TT, Inouye SK. Delirium in older persons: advances in diagnosis and treatment. JAMA. 2017;318(12):1161-1174. https://doi.org/10.1001/jama.2017.12067
5. R McKenzie, M Stewart, M. Bellantoni, TE Finucane. Bacteriuria in Individuals who become delirious. Am J Med. 2014;127(4):255-257. https://doi.org/10.1016/j.amjmed.2013.10.016
6. Balogun S, Philbrick JT. Delirium, a symptom of UTI in the elderly: fact or fable? A systematic review. Can Geriatr J. 2013;17(1):22-26. https://doi.org/10.5770/cgj.17.90
7. Nicolle LE, Mayhew WJ, Bryan L. Prospective randomized comparison of therapy and no therapy for asymptomatic bacteriuria in institutionalized elderly women. Am J Med. 1987;83(1):27-33. https://doi.org/10.1016/0002-9343(87)90493-1
8. Zalmanovici Trestioreanu A, Lador A, Sauerbrun-Cutler MT, Leibovici L. Antibiotics for asymptomatic bacteriuria. Cochrane Database Syst Rev. 2015;4:CD009534. https://doi.org/10.1002/14651858.cd009534.pub2
9. Mayne S, Bowden A, Sundvall PD, Gunnarsson R. The scientific evidence for a potential link between confusion and urinary tract infection in the elderly is still confusing – a systematic literature review. BMC Geriatr. 2019;19(1):32. https://doi.org/10.1186/s12877-019-1049-7
10. Centers for Disease Control and Prevention. Urinary tract infection (catheter-associated urinary tract infection [CAUTI] and non-catheter-associated urinary tract infection [UTI]) events. In: National Health Safety Network (NHSN) Patient Safety Component Manual. 2021:7-5. Published January 2021. Accessed April 12, 2021. https://www.cdc.gov/nhsn/pdfs/pscmanual/pcsmanual_current.pdf
11. Gupta K, Grigoryan L, Trautner B. 2017. Urinary tract infection. Ann Intern Med. 2017;167(7):ITC49-ITC64. https://doi.org/10.7326/aitc201710030
12. Nicolle LE, Gupta K, Bradley SF, et al. 2019. Clinical practice guideline for the management of asymptomatic bacteriuria: 2019 update by the Infectious Diseases Society of America. Clin Infect Dis. 2019;68(10):1611-1615. https://doi.org/10.1093/cid/ciz021
13. Petty LA, Vaughn VM, Flanders SA, et al. Risk factors and outcomes associated with treatment of asymptomatic bacteriuria in hospitalized patients. JAMA Intern Med. 2019;179(11):1519-1527. https://doi.org/10.1001/jamainternmed.2019.2871
14. Dasgupta M, Brymer C, Elsayed S. 2017. Treatment of asymptomatic UTI in older delirious medical in-patients: a prospective cohort study. Arch Gerontol Geriatr. 2017;72:127-134. https://doi.org/10.1016/j.archger.2017.05.010
15. Pop-Vicas A, Mitchell SL, Kandel R, Schreiber R, D’Agata EMC. Multidrug-resistant gram-negative bacteria in a long-term care facility: prevalence and risk factors. J Am Geriatr Soc. 2008;56(7):1276-1280. https://doi.org/10.1111/j.1532-5415.2008.01787.x
16. Das R, Towle V, Van Ness PH, Juthani-Mehta M. 2011. Adverse outcomes in nursing home residents with increased episodes of observed bacteriuria. Infect Control Hosp Epidemiol. 2011;32(1):84-86. https://doi.org/10.1086/657664
17. American Board of Internal Medicine. Choosing Wisely. American Geriatrics Society. Antimicrobials to treat bacteriuria in older adults.” Published February 21, 2013. Accessed April 12, 2021. www.choosingwisely.org/clinician-lists/american-geriatrics-society-antimicrobials-to-treat-bacteriuria-in-older-adults/
18. Fong TG, Tulebaev SR, Inouye SK. Delirium in elderly adults: diagnosis, prevention and treatment. Nat Rev Neurol. 2009;5(4):210-220. https://doi.org/10.1038/nrneurol.2009.24
19. 2019 American Geriatrics Society Beers Criteria Update Expert Panel. American Geriatrics Society 2019 update AGS Beers Criteria for potential inappropriate medication use in older adults. J Am Geriatr Soc. 2019;67(4):674-694. https://doi.org/10.1111/jgs.15767

References

1. World Health Organization. 2018 International Classification of Diseases for Mortality and Morbidity Statistics. 11th Rev. Published September 20, 2020. Accessed April 12, 2021. https://icd.who.int/browse10/2019/en#/F04
2. Witlox J, Eurelings LS, de Jonghe JFM, Kalisvaart KJ, Eikelenboom P, van Gool WA. Delirium in elderly patients and the risk of postdischarge mortality, institutionalization and dementia: a meta-analysis. JAMA. 2010;304(4):443-451. https://doi.org/10.1001/jama.2010.1013
3. Young J, Inouye SK. Delirium in older people. BMJ. 2007;334(7598):842-846. https://doi.org/10.1136/bmj.39169.706574.ad
4. Oh ES, Fong TG, Hshieh TT, Inouye SK. Delirium in older persons: advances in diagnosis and treatment. JAMA. 2017;318(12):1161-1174. https://doi.org/10.1001/jama.2017.12067
5. R McKenzie, M Stewart, M. Bellantoni, TE Finucane. Bacteriuria in Individuals who become delirious. Am J Med. 2014;127(4):255-257. https://doi.org/10.1016/j.amjmed.2013.10.016
6. Balogun S, Philbrick JT. Delirium, a symptom of UTI in the elderly: fact or fable? A systematic review. Can Geriatr J. 2013;17(1):22-26. https://doi.org/10.5770/cgj.17.90
7. Nicolle LE, Mayhew WJ, Bryan L. Prospective randomized comparison of therapy and no therapy for asymptomatic bacteriuria in institutionalized elderly women. Am J Med. 1987;83(1):27-33. https://doi.org/10.1016/0002-9343(87)90493-1
8. Zalmanovici Trestioreanu A, Lador A, Sauerbrun-Cutler MT, Leibovici L. Antibiotics for asymptomatic bacteriuria. Cochrane Database Syst Rev. 2015;4:CD009534. https://doi.org/10.1002/14651858.cd009534.pub2
9. Mayne S, Bowden A, Sundvall PD, Gunnarsson R. The scientific evidence for a potential link between confusion and urinary tract infection in the elderly is still confusing – a systematic literature review. BMC Geriatr. 2019;19(1):32. https://doi.org/10.1186/s12877-019-1049-7
10. Centers for Disease Control and Prevention. Urinary tract infection (catheter-associated urinary tract infection [CAUTI] and non-catheter-associated urinary tract infection [UTI]) events. In: National Health Safety Network (NHSN) Patient Safety Component Manual. 2021:7-5. Published January 2021. Accessed April 12, 2021. https://www.cdc.gov/nhsn/pdfs/pscmanual/pcsmanual_current.pdf
11. Gupta K, Grigoryan L, Trautner B. 2017. Urinary tract infection. Ann Intern Med. 2017;167(7):ITC49-ITC64. https://doi.org/10.7326/aitc201710030
12. Nicolle LE, Gupta K, Bradley SF, et al. 2019. Clinical practice guideline for the management of asymptomatic bacteriuria: 2019 update by the Infectious Diseases Society of America. Clin Infect Dis. 2019;68(10):1611-1615. https://doi.org/10.1093/cid/ciz021
13. Petty LA, Vaughn VM, Flanders SA, et al. Risk factors and outcomes associated with treatment of asymptomatic bacteriuria in hospitalized patients. JAMA Intern Med. 2019;179(11):1519-1527. https://doi.org/10.1001/jamainternmed.2019.2871
14. Dasgupta M, Brymer C, Elsayed S. 2017. Treatment of asymptomatic UTI in older delirious medical in-patients: a prospective cohort study. Arch Gerontol Geriatr. 2017;72:127-134. https://doi.org/10.1016/j.archger.2017.05.010
15. Pop-Vicas A, Mitchell SL, Kandel R, Schreiber R, D’Agata EMC. Multidrug-resistant gram-negative bacteria in a long-term care facility: prevalence and risk factors. J Am Geriatr Soc. 2008;56(7):1276-1280. https://doi.org/10.1111/j.1532-5415.2008.01787.x
16. Das R, Towle V, Van Ness PH, Juthani-Mehta M. 2011. Adverse outcomes in nursing home residents with increased episodes of observed bacteriuria. Infect Control Hosp Epidemiol. 2011;32(1):84-86. https://doi.org/10.1086/657664
17. American Board of Internal Medicine. Choosing Wisely. American Geriatrics Society. Antimicrobials to treat bacteriuria in older adults.” Published February 21, 2013. Accessed April 12, 2021. www.choosingwisely.org/clinician-lists/american-geriatrics-society-antimicrobials-to-treat-bacteriuria-in-older-adults/
18. Fong TG, Tulebaev SR, Inouye SK. Delirium in elderly adults: diagnosis, prevention and treatment. Nat Rev Neurol. 2009;5(4):210-220. https://doi.org/10.1038/nrneurol.2009.24
19. 2019 American Geriatrics Society Beers Criteria Update Expert Panel. American Geriatrics Society 2019 update AGS Beers Criteria for potential inappropriate medication use in older adults. J Am Geriatr Soc. 2019;67(4):674-694. https://doi.org/10.1111/jgs.15767

Issue
Journal of Hospital Medicine 16(9)
Issue
Journal of Hospital Medicine 16(9)
Page Number
557-559. Published Online First July 26, 2021
Page Number
557-559. Published Online First July 26, 2021
Publications
Publications
Topics
Article Type
Display Headline
Things We Do for No Reason™: Obtaining Urine Testing in Older Adults With Delirium Without Signs or Symptoms of Urinary Tract Infection
Display Headline
Things We Do for No Reason™: Obtaining Urine Testing in Older Adults With Delirium Without Signs or Symptoms of Urinary Tract Infection
Sections
Article Source

© 2021 Society of Hospital Medicine

Disallow All Ads
Correspondence Location
Paul O’Rourke, MD, MPH; Email: mailto:porourk3@jhu.edu; Telephone: 410-550-2292; Twitter: @ORourkeJr.
Content Gating
Gated (full article locked unless allowed per User)
Alternative CME
Disqus Comments
Default
Use ProPublica
Hide sidebar & use full width
render the right sidebar.
Conference Recap Checkbox
Not Conference Recap
Clinical Edge
Display the Slideshow in this Article
Gating Strategy
First Page Free
Medscape Article
Display survey writer
Reuters content
Disable Inline Native ads
WebMD Article
Article PDF Media

Things We Do For No Reason™: Serum Serologic Helicobacter pylori Testing

Article Type
Changed
Mon, 11/01/2021 - 11:04
Display Headline
Things We Do For No Reason™: Serum Serologic Helicobacter pylori Testing

Inspired by the ABIM Foundation’s Choosing Wisely® campaign, the “Things We Do for No Reason” (TWDFNR) series reviews practices that have become common parts of hospital care but may provide little value to our patients. Practices reviewed in the TWDFNR series do not represent clear-cut conclusions or clinical practice standards but are meant as a starting place for research and active discussions among hospitalists and patients. We invite you to be part of that discussion.

CLINICAL SCENARIO

A hospitalist admits a 25-year-old woman for evaluation of a 2-day history of intractable vomiting. The patient reports a 6-month history of intermittent dyspepsia. Vital signs include a normal temperature, tachycardia with a heart rate of 115 beats per minute, and a blood pressure of 100/60 mm Hg. Laboratory studies, including a complete blood count, electrolyte panel, and serum lipase, are normal; a pregnancy test is negative. Computed tomography (CT) of the patient’s abdomen and pelvis shows no abnormalities. The patient rapidly improves after 2 days with fluid resuscitation and supportive care. A serologic Helicobacter pylori test ordered on admission returns positive, prompting the hospitalist to discharge the patient on a course of bismuth quadruple anti-H pylori therapy.

BACKGROUND

H pylori infection causes upper gastrointestinal symptoms and progressive gastric damage, which can lead to peptic ulcer disease and gastric cancer. When H pylori infection is diagnosed, the current American College of Gastroenterology guidelines recommend eradication of the infection.1 Even with a waning prevalence in the United States, H pylori infects approximately 17% of persons aged 20 to 29 years and 57% of persons >70 years.2 Widely available noninvasive testing options for detecting H pylori include the enzyme-linked immunosorbent assay test for immunoglobulin G antibodies (ie, serology), the stool antigen test, and the urea breath test. Invasive options include upper endoscopy with biopsy. An analysis of diagnostic testing in the United States between 2010 and 2012 showed that approximately 70% of first-time testing was serologic.3

WHY YOU MIGHT THINK SEROLOGIC 
H PYLORI TESTING IS HELPFUL

Providers often select serologic testing for H pylori because of the relative ease of obtaining a blood sample compared to obtaining samples for a stool antigen or urea breath test. Stool antigen and the urea breath tests identify active infections and require a large population of H pylori in the stomach. Concurrent treatment with therapies that suppress H pylori, such as antimicrobials, bismuth, or proton pump inhibitors (PPIs), reduces the sensitivity of those tests.4 One study showed that treatment with bismuth reduced the sensitivity of urea breath and stool antigen tests to 50% and 85%, respectively, and that PPIs reduced the sensitivity of the urea breath test and stool antigen test to 60% and 75%, respectively.4 The use of antibiotics, PPIs, or bismuth, however, does not affect the test characteristics of serology.

Invasive testing with endoscopy and biopsy may also yield false-negative results. For example, providers often appropriately start PPI therapy in hospitalized patients with suspected bleeding peptic ulcers. Without concurrent treatment with a PPI, the gastric histology should show the histologic hallmarks of H pylori (ie, acute-on-chronic inflammation), as well as the organisms. However, PPI suppression of the infection and active bleeding may reduce the sensitivity of endoscopic biopsy.5,6 In one study, PPI use decreased sensitivity of histology to approximately 67% compared to polymerase chain reaction testing of the biopsy.6 Bleeding peptic ulcers do not affect the accuracy of serologic testing.

WHY SEROLOGIC TESTING FOR
H PYLORI IS NOT HELPFUL

There are three main issues with H pylori serology testing: (1) decreased sensitivity of these tests compared to other noninvasive tests, (2) inability of serology tests to distinguish between past and active infection (ie, the test is not specific for active infection), and (3) wide availability and use by commercial laboratories of serologic tests that are not approved by the US Food and Drug Administration (FDA).

A multicenter trial in the United States comparing three different serologic tests for H pylori demonstrated sensitivities ranging from 76% to 84%.7 By comparison, the main stool antigen test for H pylori available in the United States has a sensitivity of 93%.8 A recent meta-analysis showed a pooled sensitivity of 96% for urea breath tests.9 These studies demonstrate that the stool antigen and urea breath tests generally eclipse the sensitivity of the available serologic tests.

To further illustrate the issues associated with serologic testing, one may consider a population of 1,000 people with an H pylori prevalence of 35%, the estimated overall prevalence of H pylori in the United States.10 In this population, a serologic test with an 80% sensitivity would result in 70 false-negative results, whereas a urea breath or stool antigen test with a 95% sensitivity would yield only 18 false-negative results. These numbers change drastically with changing prevalence or pretest probability. In some low-prevalence or low-pretest probability scenarios, serologic tests offer little more than a “coin-flip” chance of detecting active H pylori infection (Figure).

Xu14540721e_f1.JPG

Serologic testing offers the benefit of an immediate result but at the cost of reduced sensitivity and specificity. The superior accuracy of biopsy and urea breath and stool antigen tests is dependent upon on cessation of antimicrobials, bismuth, and PPI therapy—something that may be difficult to achieve in hospitalized patients. In the majority of cases, however, there is little evidence equating immediate diagnosis of H pylori with improved patient outcomes. The preferred strategy to reduce false-negative results is to defer stool antigen or urea breath testing until patients have been off antimicrobials, bismuth, and PPIs for 4 weeks.

Serologic tests for H pylori may remain positive for years, which decreases the specificity of these tests in confirming active or eradicated infection.11 One study evaluated three different serology tests on 82 patients 6 months after confirmed eradication by urea breath test. In this study, only seven or eight patients tested negative by serology (depending on the serology test)—a specificity of 8% to 10% for active infection.12 Another study showed that even after 1 year of confirmed eradication, 65% of patients remained seropositive, which equates to a specificity of 35%.11 These studies illustrate that serologic testing for H pylori has a very poor ability to distinguish between active and past infection.

An additional common misconception is that a positive serologic test in the absence of prior treatment for, or diagnosis of, H pylori indicates an active infection. Children and adults can spontaneously clear and become reinfected with H pylori.13,14 Therefore, serologic testing for ascertaining active H pylori infection is unreliable.

As noted, the wide availability of non-FDA-approved serologic tests offered by commercial laboratories in the United States creates another problem for serologic testing. Most immunoglobulin A (IgA) and all immunoglobulin M (IgM) tests lack FDA approval and typically have low sensitivity and specificity. One study showed that compared to stool antigen, IgA and IgM serologic tests had a sensitivity of 63% and 7%, respectively.15

WHEN MIGHT SEROLOGIC   H PYLORI TESTING BE HELPFUL?

Despite its limitations, serologic testing for H pylori may have a role in some situations. Clinical scenarios associated with a high pretest probability of H pylori infection (eg, chronic peptic ulcer disease without other risk factors) increase the positive predictive value of H pylori infection. In such a situation, a positive serologic test should prompt initiation of treatment, whereas a negative serologic test does not rule out H pylori infection (Figure). In contrast, in the presence of lower pretest probability symptoms (eg, dyspepsia), positive serologic testing has such a high false-positive rate that providers must first confirm the result with a stool antigen or urea breath test before initiating treatment.

WHAT YOU SHOULD DO INSTEAD

For patients with alarm signs and symptoms and an indication for endoscopy (eg, bleeding peptic ulcer, iron deficiency anemia), providers should use endoscopy with biopsy to diagnose H pylori infection.16 For patients with dyspepsia or nonspecific gastrointestinal symptoms (ie, a low pretest probability of H pylori) and no indication for endoscopy, providers should diagnose active infection with stool antigen or urea breath test. If possible, serologic testing should be avoided. Except in very high pretest probability clinical scenarios, positive serologic tests should be confirmed via stool antigen or urea breath test before initiating treatment. The stool antigen or urea breath test should only be ordered after patients have stopped antibiotics, bismuth, and PPIs for 4 weeks.16 For patients requiring antisecretory therapy, providers can substitute histamine-2 receptor antagonists (H2RA) for the PPIs, as H2RAs do not interfere with either the stool antigen or urea breath test.4 Eradication of H pylori infection should be confirmed through biopsy, urea breath test, or stool antigen test 4 weeks after patients have completed treatment.

RECOMMENDATIONS

  • Use stool antigen or urea breath tests to diagnose H pylori infection noninvasively in patients without an indication for endoscopy.
  • Use endoscopic biopsy with histology to diagnose H pylori infection in patients with an indication for endoscopy.
  • Delay stool antigen and urea breath testing until 4 weeks after patients have ceased using medications that interfere with test results (eg, antibiotics, bismuth, PPIs); H2RAs do not interfere with testing.
  • In cases of a bleeding peptic ulcer with a negative biopsy for H pylori, retest with biopsy after the bleeding resolves or retest using stool antigen or urea breath test.
  • Confirm a positive serologic test via stool antigen or urea breath test before initiating treatment except in very high pretest probability clinical scenarios.
  • Test to confirm eradication with biopsy, urea breath, or stool antigen test in all cases of confirmed H pylori infection.
  • Do not order or try to interpret H pylori IgA and IgM tests as they have no role in the diagnosis or management of H pylori infections.

CONCLUSION

In the clinical scenario, the patient clinically improved with fluid resuscitation and supportive care. The history of unexplained dyspepsia is an indication to assess for H pylori infection with either urea breath test or stool antigen test. Given the positive serologic test, the provider should have retested for active infection with a stool antigen or urea breath test prior to initiating treatment.

Do you think this is a low-value practice? Is this truly a “Thing We Do for No Reason”? Share what you do in your practice and join in the conversation online by retweeting it on Twitter (#TWDFNR) and liking it on Facebook. We invite you to propose ideas for other “Things We Do for No Reason” topics by emailing TWDFNR@hospitalmedicine.org

References

1. Chey WD, Wong BC; Practice Parameters Committee of the American College of Gastroenterology. American College of Gastroenterology guideline on the management of Helicobacter pylori infection. Am J Gastroenterol. 2007;102(8):1808-1825. https://doi.org/10.1111/j.1572-0241.2007.01393.x
2. Everhart JE, Kruszon-Moran D, Perez-Perez GI, Tralka TS, McQuillan G. Seroprevalence and ethnic differences in Helicobacter pylori infection among adults in the United States. J Infect Dis. 2000;181(4):1359-1363. https://doi.org/10.1086/315384
3. Theel ES, Johnson RD, Plumhoff E, Hanson CA. Use of the Optum Labs Data Warehouse to assess test ordering patterns for diagnosis of Helicobacter pylori infection in the United States. J Clin Microbiol. 2015;53(4):1358-1360. https://doi.org/10.1128/jcm.03464-14
4. Bravo LE, Realpe JL, Campo C, Correa P. Effects of acid suppression and bismuth medications on the performance of diagnostic tests for Helicobacter pylori infection. Am J Gastroentrol. 1999;94(9):2380-2383. https://doi.org/10.1111/j.1572-0241.1999.01361.x
5. Logan RP, Walker MM, Misiewicz JJ, Gummett PA, Karim QN, Baron JH. Changes in the intragastric distribution of Helicobacter pylori during treatment with omeprazole. Gut. 1995;36(1):12-16. https://doi.org/10.1136/gut.36.1.12
6. Yakoob J, Jafri W, Abbas Z, Abid S, Islam M, Ahmed Z. The diagnostic yield of various tests for Helicobacter pylori infection in patients on acid-reducing drugs. Dig Dis Sci. 2008;53(1):95-100. https://doi.org/10.1007/s10620-007-9828-y
7. Chey WD, Murthy U, Shaw S, et al. A comparison of three fingerstick, whole blood antibody tests for Helicobacter pylori infection: a United States, multicenter trial. Am J Gastroentrol. 1999;94(6):1512-1516. https://doi.org/10.1111/j.1572-0241.1999.1135_x.x
8. Li YH, Guo H, Zhang PB, Zhao XY, Da SP. Clinical value of Helicobacter pylori stool antigen test, ImmunoCard STAT HpSA, for detecting H pylori infection. World J Gastroenterol. 2004;10(6):913-914. https://doi.org/10.3748/wjg.v10.i6.913
9. Ferwana M, Abdulmajeed I, Alhajiahmed A, et al. Accuracy of urea breath test in Helicobacter pylori infection: meta-analysis. World J Gastroenterol. 2015;21(4):1305-1314. https://doi.org/10.3748/wjg.v21.i4.1305
10. Hooi JK, Lai WY, Ng WK, et al. Global prevalence of Helicobacter pylori infection: systematic review and meta-analysis. Gastroenterology. 2017;153(2):420-429. https://doi.org/10.1053/j.gastro.2017.04.022
11. Cutler AF, Prasad VM. Long-term follow-up of Helicobacter pylori serology after successful eradication. Am J Gastroenterol. 1996;91(1):85-88.
12. Bergey B, Marchildon P, Peacock J, Mégraud PF. What is the role of serology in assessing Helicobacter pylori eradication? Aliment Pharmacol Ther. 2003;18(6):635-639. https://doi.org/10.1046/j.1365-2036.2003.01716.x
13. Duque X, Vilchis J, Mera R, et al. Natural history of Helicobacter pylori infection in Mexican schoolchildren: incidence and spontaneous clearance. J Pediatr Gastroenterol Nutr. 2012;55(2):209. https://doi.org/10.1097/mpg.0b013e318248877f
14. Luzza F, Suraci E, Larussa T, Leone I, Imeneo M. High exposure, spontaneous clearance, and low incidence of active Helicobacter pylori infection: the Sorbo San Basile study. Helicobacter. 2014;19(4):296-305. https://doi.org/10.1111/hel.12133
15. She RC, Wilson AR, Litwin CM. Evaluation of Helicobacter pylori immunoglobulin G (IgG), IgA, and IgM serologic testing compared to stool antigen testing. Clin Vaccine Immunol. 2009;16(8):1253-1255. https://doi.org/10.1128/cvi.00149-09
16. El-Serag HB, Kao JY, Kanwal F, et al. Houston consensus conference on testing for Helicobacter pylori infection in the United States. Clin Gastroenterol Hepatol. 2018;16(7):992-1002. Published correction appears in Clin Gastroenterol Hepatol. 2019;17(4):801. https://doi.org/10.1016/j.cgh.2019.01.006

Article PDF
Author and Disclosure Information

1Department of Medicine, Baylor College of Medicine, Houston, Texas; 2Section of Gastroenterology, Michael E. Debakey Veteran Affairs Medical Center, Houston, Texas.

Disclosures
Dr Graham reports receipt of grants from the National Institute of Diabetes and Digestive and Kidney Diseases and RedHill Biopharma; nonfinancial support from Phathom Pharmaceuticals; and personal fees from Otsuka Pharmaceutical Co, Ltd, Otsuka, Japan, outside the submitted work.

Issue
Journal of Hospital Medicine 16(11)
Publications
Topics
Page Number
691-693. Published Online First July 21, 2021
Sections
Author and Disclosure Information

1Department of Medicine, Baylor College of Medicine, Houston, Texas; 2Section of Gastroenterology, Michael E. Debakey Veteran Affairs Medical Center, Houston, Texas.

Disclosures
Dr Graham reports receipt of grants from the National Institute of Diabetes and Digestive and Kidney Diseases and RedHill Biopharma; nonfinancial support from Phathom Pharmaceuticals; and personal fees from Otsuka Pharmaceutical Co, Ltd, Otsuka, Japan, outside the submitted work.

Author and Disclosure Information

1Department of Medicine, Baylor College of Medicine, Houston, Texas; 2Section of Gastroenterology, Michael E. Debakey Veteran Affairs Medical Center, Houston, Texas.

Disclosures
Dr Graham reports receipt of grants from the National Institute of Diabetes and Digestive and Kidney Diseases and RedHill Biopharma; nonfinancial support from Phathom Pharmaceuticals; and personal fees from Otsuka Pharmaceutical Co, Ltd, Otsuka, Japan, outside the submitted work.

Article PDF
Article PDF
Related Articles

Inspired by the ABIM Foundation’s Choosing Wisely® campaign, the “Things We Do for No Reason” (TWDFNR) series reviews practices that have become common parts of hospital care but may provide little value to our patients. Practices reviewed in the TWDFNR series do not represent clear-cut conclusions or clinical practice standards but are meant as a starting place for research and active discussions among hospitalists and patients. We invite you to be part of that discussion.

CLINICAL SCENARIO

A hospitalist admits a 25-year-old woman for evaluation of a 2-day history of intractable vomiting. The patient reports a 6-month history of intermittent dyspepsia. Vital signs include a normal temperature, tachycardia with a heart rate of 115 beats per minute, and a blood pressure of 100/60 mm Hg. Laboratory studies, including a complete blood count, electrolyte panel, and serum lipase, are normal; a pregnancy test is negative. Computed tomography (CT) of the patient’s abdomen and pelvis shows no abnormalities. The patient rapidly improves after 2 days with fluid resuscitation and supportive care. A serologic Helicobacter pylori test ordered on admission returns positive, prompting the hospitalist to discharge the patient on a course of bismuth quadruple anti-H pylori therapy.

BACKGROUND

H pylori infection causes upper gastrointestinal symptoms and progressive gastric damage, which can lead to peptic ulcer disease and gastric cancer. When H pylori infection is diagnosed, the current American College of Gastroenterology guidelines recommend eradication of the infection.1 Even with a waning prevalence in the United States, H pylori infects approximately 17% of persons aged 20 to 29 years and 57% of persons >70 years.2 Widely available noninvasive testing options for detecting H pylori include the enzyme-linked immunosorbent assay test for immunoglobulin G antibodies (ie, serology), the stool antigen test, and the urea breath test. Invasive options include upper endoscopy with biopsy. An analysis of diagnostic testing in the United States between 2010 and 2012 showed that approximately 70% of first-time testing was serologic.3

WHY YOU MIGHT THINK SEROLOGIC 
H PYLORI TESTING IS HELPFUL

Providers often select serologic testing for H pylori because of the relative ease of obtaining a blood sample compared to obtaining samples for a stool antigen or urea breath test. Stool antigen and the urea breath tests identify active infections and require a large population of H pylori in the stomach. Concurrent treatment with therapies that suppress H pylori, such as antimicrobials, bismuth, or proton pump inhibitors (PPIs), reduces the sensitivity of those tests.4 One study showed that treatment with bismuth reduced the sensitivity of urea breath and stool antigen tests to 50% and 85%, respectively, and that PPIs reduced the sensitivity of the urea breath test and stool antigen test to 60% and 75%, respectively.4 The use of antibiotics, PPIs, or bismuth, however, does not affect the test characteristics of serology.

Invasive testing with endoscopy and biopsy may also yield false-negative results. For example, providers often appropriately start PPI therapy in hospitalized patients with suspected bleeding peptic ulcers. Without concurrent treatment with a PPI, the gastric histology should show the histologic hallmarks of H pylori (ie, acute-on-chronic inflammation), as well as the organisms. However, PPI suppression of the infection and active bleeding may reduce the sensitivity of endoscopic biopsy.5,6 In one study, PPI use decreased sensitivity of histology to approximately 67% compared to polymerase chain reaction testing of the biopsy.6 Bleeding peptic ulcers do not affect the accuracy of serologic testing.

WHY SEROLOGIC TESTING FOR
H PYLORI IS NOT HELPFUL

There are three main issues with H pylori serology testing: (1) decreased sensitivity of these tests compared to other noninvasive tests, (2) inability of serology tests to distinguish between past and active infection (ie, the test is not specific for active infection), and (3) wide availability and use by commercial laboratories of serologic tests that are not approved by the US Food and Drug Administration (FDA).

A multicenter trial in the United States comparing three different serologic tests for H pylori demonstrated sensitivities ranging from 76% to 84%.7 By comparison, the main stool antigen test for H pylori available in the United States has a sensitivity of 93%.8 A recent meta-analysis showed a pooled sensitivity of 96% for urea breath tests.9 These studies demonstrate that the stool antigen and urea breath tests generally eclipse the sensitivity of the available serologic tests.

To further illustrate the issues associated with serologic testing, one may consider a population of 1,000 people with an H pylori prevalence of 35%, the estimated overall prevalence of H pylori in the United States.10 In this population, a serologic test with an 80% sensitivity would result in 70 false-negative results, whereas a urea breath or stool antigen test with a 95% sensitivity would yield only 18 false-negative results. These numbers change drastically with changing prevalence or pretest probability. In some low-prevalence or low-pretest probability scenarios, serologic tests offer little more than a “coin-flip” chance of detecting active H pylori infection (Figure).

Xu14540721e_f1.JPG

Serologic testing offers the benefit of an immediate result but at the cost of reduced sensitivity and specificity. The superior accuracy of biopsy and urea breath and stool antigen tests is dependent upon on cessation of antimicrobials, bismuth, and PPI therapy—something that may be difficult to achieve in hospitalized patients. In the majority of cases, however, there is little evidence equating immediate diagnosis of H pylori with improved patient outcomes. The preferred strategy to reduce false-negative results is to defer stool antigen or urea breath testing until patients have been off antimicrobials, bismuth, and PPIs for 4 weeks.

Serologic tests for H pylori may remain positive for years, which decreases the specificity of these tests in confirming active or eradicated infection.11 One study evaluated three different serology tests on 82 patients 6 months after confirmed eradication by urea breath test. In this study, only seven or eight patients tested negative by serology (depending on the serology test)—a specificity of 8% to 10% for active infection.12 Another study showed that even after 1 year of confirmed eradication, 65% of patients remained seropositive, which equates to a specificity of 35%.11 These studies illustrate that serologic testing for H pylori has a very poor ability to distinguish between active and past infection.

An additional common misconception is that a positive serologic test in the absence of prior treatment for, or diagnosis of, H pylori indicates an active infection. Children and adults can spontaneously clear and become reinfected with H pylori.13,14 Therefore, serologic testing for ascertaining active H pylori infection is unreliable.

As noted, the wide availability of non-FDA-approved serologic tests offered by commercial laboratories in the United States creates another problem for serologic testing. Most immunoglobulin A (IgA) and all immunoglobulin M (IgM) tests lack FDA approval and typically have low sensitivity and specificity. One study showed that compared to stool antigen, IgA and IgM serologic tests had a sensitivity of 63% and 7%, respectively.15

WHEN MIGHT SEROLOGIC   H PYLORI TESTING BE HELPFUL?

Despite its limitations, serologic testing for H pylori may have a role in some situations. Clinical scenarios associated with a high pretest probability of H pylori infection (eg, chronic peptic ulcer disease without other risk factors) increase the positive predictive value of H pylori infection. In such a situation, a positive serologic test should prompt initiation of treatment, whereas a negative serologic test does not rule out H pylori infection (Figure). In contrast, in the presence of lower pretest probability symptoms (eg, dyspepsia), positive serologic testing has such a high false-positive rate that providers must first confirm the result with a stool antigen or urea breath test before initiating treatment.

WHAT YOU SHOULD DO INSTEAD

For patients with alarm signs and symptoms and an indication for endoscopy (eg, bleeding peptic ulcer, iron deficiency anemia), providers should use endoscopy with biopsy to diagnose H pylori infection.16 For patients with dyspepsia or nonspecific gastrointestinal symptoms (ie, a low pretest probability of H pylori) and no indication for endoscopy, providers should diagnose active infection with stool antigen or urea breath test. If possible, serologic testing should be avoided. Except in very high pretest probability clinical scenarios, positive serologic tests should be confirmed via stool antigen or urea breath test before initiating treatment. The stool antigen or urea breath test should only be ordered after patients have stopped antibiotics, bismuth, and PPIs for 4 weeks.16 For patients requiring antisecretory therapy, providers can substitute histamine-2 receptor antagonists (H2RA) for the PPIs, as H2RAs do not interfere with either the stool antigen or urea breath test.4 Eradication of H pylori infection should be confirmed through biopsy, urea breath test, or stool antigen test 4 weeks after patients have completed treatment.

RECOMMENDATIONS

  • Use stool antigen or urea breath tests to diagnose H pylori infection noninvasively in patients without an indication for endoscopy.
  • Use endoscopic biopsy with histology to diagnose H pylori infection in patients with an indication for endoscopy.
  • Delay stool antigen and urea breath testing until 4 weeks after patients have ceased using medications that interfere with test results (eg, antibiotics, bismuth, PPIs); H2RAs do not interfere with testing.
  • In cases of a bleeding peptic ulcer with a negative biopsy for H pylori, retest with biopsy after the bleeding resolves or retest using stool antigen or urea breath test.
  • Confirm a positive serologic test via stool antigen or urea breath test before initiating treatment except in very high pretest probability clinical scenarios.
  • Test to confirm eradication with biopsy, urea breath, or stool antigen test in all cases of confirmed H pylori infection.
  • Do not order or try to interpret H pylori IgA and IgM tests as they have no role in the diagnosis or management of H pylori infections.

CONCLUSION

In the clinical scenario, the patient clinically improved with fluid resuscitation and supportive care. The history of unexplained dyspepsia is an indication to assess for H pylori infection with either urea breath test or stool antigen test. Given the positive serologic test, the provider should have retested for active infection with a stool antigen or urea breath test prior to initiating treatment.

Do you think this is a low-value practice? Is this truly a “Thing We Do for No Reason”? Share what you do in your practice and join in the conversation online by retweeting it on Twitter (#TWDFNR) and liking it on Facebook. We invite you to propose ideas for other “Things We Do for No Reason” topics by emailing TWDFNR@hospitalmedicine.org

Inspired by the ABIM Foundation’s Choosing Wisely® campaign, the “Things We Do for No Reason” (TWDFNR) series reviews practices that have become common parts of hospital care but may provide little value to our patients. Practices reviewed in the TWDFNR series do not represent clear-cut conclusions or clinical practice standards but are meant as a starting place for research and active discussions among hospitalists and patients. We invite you to be part of that discussion.

CLINICAL SCENARIO

A hospitalist admits a 25-year-old woman for evaluation of a 2-day history of intractable vomiting. The patient reports a 6-month history of intermittent dyspepsia. Vital signs include a normal temperature, tachycardia with a heart rate of 115 beats per minute, and a blood pressure of 100/60 mm Hg. Laboratory studies, including a complete blood count, electrolyte panel, and serum lipase, are normal; a pregnancy test is negative. Computed tomography (CT) of the patient’s abdomen and pelvis shows no abnormalities. The patient rapidly improves after 2 days with fluid resuscitation and supportive care. A serologic Helicobacter pylori test ordered on admission returns positive, prompting the hospitalist to discharge the patient on a course of bismuth quadruple anti-H pylori therapy.

BACKGROUND

H pylori infection causes upper gastrointestinal symptoms and progressive gastric damage, which can lead to peptic ulcer disease and gastric cancer. When H pylori infection is diagnosed, the current American College of Gastroenterology guidelines recommend eradication of the infection.1 Even with a waning prevalence in the United States, H pylori infects approximately 17% of persons aged 20 to 29 years and 57% of persons >70 years.2 Widely available noninvasive testing options for detecting H pylori include the enzyme-linked immunosorbent assay test for immunoglobulin G antibodies (ie, serology), the stool antigen test, and the urea breath test. Invasive options include upper endoscopy with biopsy. An analysis of diagnostic testing in the United States between 2010 and 2012 showed that approximately 70% of first-time testing was serologic.3

WHY YOU MIGHT THINK SEROLOGIC 
H PYLORI TESTING IS HELPFUL

Providers often select serologic testing for H pylori because of the relative ease of obtaining a blood sample compared to obtaining samples for a stool antigen or urea breath test. Stool antigen and the urea breath tests identify active infections and require a large population of H pylori in the stomach. Concurrent treatment with therapies that suppress H pylori, such as antimicrobials, bismuth, or proton pump inhibitors (PPIs), reduces the sensitivity of those tests.4 One study showed that treatment with bismuth reduced the sensitivity of urea breath and stool antigen tests to 50% and 85%, respectively, and that PPIs reduced the sensitivity of the urea breath test and stool antigen test to 60% and 75%, respectively.4 The use of antibiotics, PPIs, or bismuth, however, does not affect the test characteristics of serology.

Invasive testing with endoscopy and biopsy may also yield false-negative results. For example, providers often appropriately start PPI therapy in hospitalized patients with suspected bleeding peptic ulcers. Without concurrent treatment with a PPI, the gastric histology should show the histologic hallmarks of H pylori (ie, acute-on-chronic inflammation), as well as the organisms. However, PPI suppression of the infection and active bleeding may reduce the sensitivity of endoscopic biopsy.5,6 In one study, PPI use decreased sensitivity of histology to approximately 67% compared to polymerase chain reaction testing of the biopsy.6 Bleeding peptic ulcers do not affect the accuracy of serologic testing.

WHY SEROLOGIC TESTING FOR
H PYLORI IS NOT HELPFUL

There are three main issues with H pylori serology testing: (1) decreased sensitivity of these tests compared to other noninvasive tests, (2) inability of serology tests to distinguish between past and active infection (ie, the test is not specific for active infection), and (3) wide availability and use by commercial laboratories of serologic tests that are not approved by the US Food and Drug Administration (FDA).

A multicenter trial in the United States comparing three different serologic tests for H pylori demonstrated sensitivities ranging from 76% to 84%.7 By comparison, the main stool antigen test for H pylori available in the United States has a sensitivity of 93%.8 A recent meta-analysis showed a pooled sensitivity of 96% for urea breath tests.9 These studies demonstrate that the stool antigen and urea breath tests generally eclipse the sensitivity of the available serologic tests.

To further illustrate the issues associated with serologic testing, one may consider a population of 1,000 people with an H pylori prevalence of 35%, the estimated overall prevalence of H pylori in the United States.10 In this population, a serologic test with an 80% sensitivity would result in 70 false-negative results, whereas a urea breath or stool antigen test with a 95% sensitivity would yield only 18 false-negative results. These numbers change drastically with changing prevalence or pretest probability. In some low-prevalence or low-pretest probability scenarios, serologic tests offer little more than a “coin-flip” chance of detecting active H pylori infection (Figure).

Xu14540721e_f1.JPG

Serologic testing offers the benefit of an immediate result but at the cost of reduced sensitivity and specificity. The superior accuracy of biopsy and urea breath and stool antigen tests is dependent upon on cessation of antimicrobials, bismuth, and PPI therapy—something that may be difficult to achieve in hospitalized patients. In the majority of cases, however, there is little evidence equating immediate diagnosis of H pylori with improved patient outcomes. The preferred strategy to reduce false-negative results is to defer stool antigen or urea breath testing until patients have been off antimicrobials, bismuth, and PPIs for 4 weeks.

Serologic tests for H pylori may remain positive for years, which decreases the specificity of these tests in confirming active or eradicated infection.11 One study evaluated three different serology tests on 82 patients 6 months after confirmed eradication by urea breath test. In this study, only seven or eight patients tested negative by serology (depending on the serology test)—a specificity of 8% to 10% for active infection.12 Another study showed that even after 1 year of confirmed eradication, 65% of patients remained seropositive, which equates to a specificity of 35%.11 These studies illustrate that serologic testing for H pylori has a very poor ability to distinguish between active and past infection.

An additional common misconception is that a positive serologic test in the absence of prior treatment for, or diagnosis of, H pylori indicates an active infection. Children and adults can spontaneously clear and become reinfected with H pylori.13,14 Therefore, serologic testing for ascertaining active H pylori infection is unreliable.

As noted, the wide availability of non-FDA-approved serologic tests offered by commercial laboratories in the United States creates another problem for serologic testing. Most immunoglobulin A (IgA) and all immunoglobulin M (IgM) tests lack FDA approval and typically have low sensitivity and specificity. One study showed that compared to stool antigen, IgA and IgM serologic tests had a sensitivity of 63% and 7%, respectively.15

WHEN MIGHT SEROLOGIC   H PYLORI TESTING BE HELPFUL?

Despite its limitations, serologic testing for H pylori may have a role in some situations. Clinical scenarios associated with a high pretest probability of H pylori infection (eg, chronic peptic ulcer disease without other risk factors) increase the positive predictive value of H pylori infection. In such a situation, a positive serologic test should prompt initiation of treatment, whereas a negative serologic test does not rule out H pylori infection (Figure). In contrast, in the presence of lower pretest probability symptoms (eg, dyspepsia), positive serologic testing has such a high false-positive rate that providers must first confirm the result with a stool antigen or urea breath test before initiating treatment.

WHAT YOU SHOULD DO INSTEAD

For patients with alarm signs and symptoms and an indication for endoscopy (eg, bleeding peptic ulcer, iron deficiency anemia), providers should use endoscopy with biopsy to diagnose H pylori infection.16 For patients with dyspepsia or nonspecific gastrointestinal symptoms (ie, a low pretest probability of H pylori) and no indication for endoscopy, providers should diagnose active infection with stool antigen or urea breath test. If possible, serologic testing should be avoided. Except in very high pretest probability clinical scenarios, positive serologic tests should be confirmed via stool antigen or urea breath test before initiating treatment. The stool antigen or urea breath test should only be ordered after patients have stopped antibiotics, bismuth, and PPIs for 4 weeks.16 For patients requiring antisecretory therapy, providers can substitute histamine-2 receptor antagonists (H2RA) for the PPIs, as H2RAs do not interfere with either the stool antigen or urea breath test.4 Eradication of H pylori infection should be confirmed through biopsy, urea breath test, or stool antigen test 4 weeks after patients have completed treatment.

RECOMMENDATIONS

  • Use stool antigen or urea breath tests to diagnose H pylori infection noninvasively in patients without an indication for endoscopy.
  • Use endoscopic biopsy with histology to diagnose H pylori infection in patients with an indication for endoscopy.
  • Delay stool antigen and urea breath testing until 4 weeks after patients have ceased using medications that interfere with test results (eg, antibiotics, bismuth, PPIs); H2RAs do not interfere with testing.
  • In cases of a bleeding peptic ulcer with a negative biopsy for H pylori, retest with biopsy after the bleeding resolves or retest using stool antigen or urea breath test.
  • Confirm a positive serologic test via stool antigen or urea breath test before initiating treatment except in very high pretest probability clinical scenarios.
  • Test to confirm eradication with biopsy, urea breath, or stool antigen test in all cases of confirmed H pylori infection.
  • Do not order or try to interpret H pylori IgA and IgM tests as they have no role in the diagnosis or management of H pylori infections.

CONCLUSION

In the clinical scenario, the patient clinically improved with fluid resuscitation and supportive care. The history of unexplained dyspepsia is an indication to assess for H pylori infection with either urea breath test or stool antigen test. Given the positive serologic test, the provider should have retested for active infection with a stool antigen or urea breath test prior to initiating treatment.

Do you think this is a low-value practice? Is this truly a “Thing We Do for No Reason”? Share what you do in your practice and join in the conversation online by retweeting it on Twitter (#TWDFNR) and liking it on Facebook. We invite you to propose ideas for other “Things We Do for No Reason” topics by emailing TWDFNR@hospitalmedicine.org

References

1. Chey WD, Wong BC; Practice Parameters Committee of the American College of Gastroenterology. American College of Gastroenterology guideline on the management of Helicobacter pylori infection. Am J Gastroenterol. 2007;102(8):1808-1825. https://doi.org/10.1111/j.1572-0241.2007.01393.x
2. Everhart JE, Kruszon-Moran D, Perez-Perez GI, Tralka TS, McQuillan G. Seroprevalence and ethnic differences in Helicobacter pylori infection among adults in the United States. J Infect Dis. 2000;181(4):1359-1363. https://doi.org/10.1086/315384
3. Theel ES, Johnson RD, Plumhoff E, Hanson CA. Use of the Optum Labs Data Warehouse to assess test ordering patterns for diagnosis of Helicobacter pylori infection in the United States. J Clin Microbiol. 2015;53(4):1358-1360. https://doi.org/10.1128/jcm.03464-14
4. Bravo LE, Realpe JL, Campo C, Correa P. Effects of acid suppression and bismuth medications on the performance of diagnostic tests for Helicobacter pylori infection. Am J Gastroentrol. 1999;94(9):2380-2383. https://doi.org/10.1111/j.1572-0241.1999.01361.x
5. Logan RP, Walker MM, Misiewicz JJ, Gummett PA, Karim QN, Baron JH. Changes in the intragastric distribution of Helicobacter pylori during treatment with omeprazole. Gut. 1995;36(1):12-16. https://doi.org/10.1136/gut.36.1.12
6. Yakoob J, Jafri W, Abbas Z, Abid S, Islam M, Ahmed Z. The diagnostic yield of various tests for Helicobacter pylori infection in patients on acid-reducing drugs. Dig Dis Sci. 2008;53(1):95-100. https://doi.org/10.1007/s10620-007-9828-y
7. Chey WD, Murthy U, Shaw S, et al. A comparison of three fingerstick, whole blood antibody tests for Helicobacter pylori infection: a United States, multicenter trial. Am J Gastroentrol. 1999;94(6):1512-1516. https://doi.org/10.1111/j.1572-0241.1999.1135_x.x
8. Li YH, Guo H, Zhang PB, Zhao XY, Da SP. Clinical value of Helicobacter pylori stool antigen test, ImmunoCard STAT HpSA, for detecting H pylori infection. World J Gastroenterol. 2004;10(6):913-914. https://doi.org/10.3748/wjg.v10.i6.913
9. Ferwana M, Abdulmajeed I, Alhajiahmed A, et al. Accuracy of urea breath test in Helicobacter pylori infection: meta-analysis. World J Gastroenterol. 2015;21(4):1305-1314. https://doi.org/10.3748/wjg.v21.i4.1305
10. Hooi JK, Lai WY, Ng WK, et al. Global prevalence of Helicobacter pylori infection: systematic review and meta-analysis. Gastroenterology. 2017;153(2):420-429. https://doi.org/10.1053/j.gastro.2017.04.022
11. Cutler AF, Prasad VM. Long-term follow-up of Helicobacter pylori serology after successful eradication. Am J Gastroenterol. 1996;91(1):85-88.
12. Bergey B, Marchildon P, Peacock J, Mégraud PF. What is the role of serology in assessing Helicobacter pylori eradication? Aliment Pharmacol Ther. 2003;18(6):635-639. https://doi.org/10.1046/j.1365-2036.2003.01716.x
13. Duque X, Vilchis J, Mera R, et al. Natural history of Helicobacter pylori infection in Mexican schoolchildren: incidence and spontaneous clearance. J Pediatr Gastroenterol Nutr. 2012;55(2):209. https://doi.org/10.1097/mpg.0b013e318248877f
14. Luzza F, Suraci E, Larussa T, Leone I, Imeneo M. High exposure, spontaneous clearance, and low incidence of active Helicobacter pylori infection: the Sorbo San Basile study. Helicobacter. 2014;19(4):296-305. https://doi.org/10.1111/hel.12133
15. She RC, Wilson AR, Litwin CM. Evaluation of Helicobacter pylori immunoglobulin G (IgG), IgA, and IgM serologic testing compared to stool antigen testing. Clin Vaccine Immunol. 2009;16(8):1253-1255. https://doi.org/10.1128/cvi.00149-09
16. El-Serag HB, Kao JY, Kanwal F, et al. Houston consensus conference on testing for Helicobacter pylori infection in the United States. Clin Gastroenterol Hepatol. 2018;16(7):992-1002. Published correction appears in Clin Gastroenterol Hepatol. 2019;17(4):801. https://doi.org/10.1016/j.cgh.2019.01.006

References

1. Chey WD, Wong BC; Practice Parameters Committee of the American College of Gastroenterology. American College of Gastroenterology guideline on the management of Helicobacter pylori infection. Am J Gastroenterol. 2007;102(8):1808-1825. https://doi.org/10.1111/j.1572-0241.2007.01393.x
2. Everhart JE, Kruszon-Moran D, Perez-Perez GI, Tralka TS, McQuillan G. Seroprevalence and ethnic differences in Helicobacter pylori infection among adults in the United States. J Infect Dis. 2000;181(4):1359-1363. https://doi.org/10.1086/315384
3. Theel ES, Johnson RD, Plumhoff E, Hanson CA. Use of the Optum Labs Data Warehouse to assess test ordering patterns for diagnosis of Helicobacter pylori infection in the United States. J Clin Microbiol. 2015;53(4):1358-1360. https://doi.org/10.1128/jcm.03464-14
4. Bravo LE, Realpe JL, Campo C, Correa P. Effects of acid suppression and bismuth medications on the performance of diagnostic tests for Helicobacter pylori infection. Am J Gastroentrol. 1999;94(9):2380-2383. https://doi.org/10.1111/j.1572-0241.1999.01361.x
5. Logan RP, Walker MM, Misiewicz JJ, Gummett PA, Karim QN, Baron JH. Changes in the intragastric distribution of Helicobacter pylori during treatment with omeprazole. Gut. 1995;36(1):12-16. https://doi.org/10.1136/gut.36.1.12
6. Yakoob J, Jafri W, Abbas Z, Abid S, Islam M, Ahmed Z. The diagnostic yield of various tests for Helicobacter pylori infection in patients on acid-reducing drugs. Dig Dis Sci. 2008;53(1):95-100. https://doi.org/10.1007/s10620-007-9828-y
7. Chey WD, Murthy U, Shaw S, et al. A comparison of three fingerstick, whole blood antibody tests for Helicobacter pylori infection: a United States, multicenter trial. Am J Gastroentrol. 1999;94(6):1512-1516. https://doi.org/10.1111/j.1572-0241.1999.1135_x.x
8. Li YH, Guo H, Zhang PB, Zhao XY, Da SP. Clinical value of Helicobacter pylori stool antigen test, ImmunoCard STAT HpSA, for detecting H pylori infection. World J Gastroenterol. 2004;10(6):913-914. https://doi.org/10.3748/wjg.v10.i6.913
9. Ferwana M, Abdulmajeed I, Alhajiahmed A, et al. Accuracy of urea breath test in Helicobacter pylori infection: meta-analysis. World J Gastroenterol. 2015;21(4):1305-1314. https://doi.org/10.3748/wjg.v21.i4.1305
10. Hooi JK, Lai WY, Ng WK, et al. Global prevalence of Helicobacter pylori infection: systematic review and meta-analysis. Gastroenterology. 2017;153(2):420-429. https://doi.org/10.1053/j.gastro.2017.04.022
11. Cutler AF, Prasad VM. Long-term follow-up of Helicobacter pylori serology after successful eradication. Am J Gastroenterol. 1996;91(1):85-88.
12. Bergey B, Marchildon P, Peacock J, Mégraud PF. What is the role of serology in assessing Helicobacter pylori eradication? Aliment Pharmacol Ther. 2003;18(6):635-639. https://doi.org/10.1046/j.1365-2036.2003.01716.x
13. Duque X, Vilchis J, Mera R, et al. Natural history of Helicobacter pylori infection in Mexican schoolchildren: incidence and spontaneous clearance. J Pediatr Gastroenterol Nutr. 2012;55(2):209. https://doi.org/10.1097/mpg.0b013e318248877f
14. Luzza F, Suraci E, Larussa T, Leone I, Imeneo M. High exposure, spontaneous clearance, and low incidence of active Helicobacter pylori infection: the Sorbo San Basile study. Helicobacter. 2014;19(4):296-305. https://doi.org/10.1111/hel.12133
15. She RC, Wilson AR, Litwin CM. Evaluation of Helicobacter pylori immunoglobulin G (IgG), IgA, and IgM serologic testing compared to stool antigen testing. Clin Vaccine Immunol. 2009;16(8):1253-1255. https://doi.org/10.1128/cvi.00149-09
16. El-Serag HB, Kao JY, Kanwal F, et al. Houston consensus conference on testing for Helicobacter pylori infection in the United States. Clin Gastroenterol Hepatol. 2018;16(7):992-1002. Published correction appears in Clin Gastroenterol Hepatol. 2019;17(4):801. https://doi.org/10.1016/j.cgh.2019.01.006

Issue
Journal of Hospital Medicine 16(11)
Issue
Journal of Hospital Medicine 16(11)
Page Number
691-693. Published Online First July 21, 2021
Page Number
691-693. Published Online First July 21, 2021
Publications
Publications
Topics
Article Type
Display Headline
Things We Do For No Reason™: Serum Serologic Helicobacter pylori Testing
Display Headline
Things We Do For No Reason™: Serum Serologic Helicobacter pylori Testing
Sections
Article Source

© 2021 Society of Hospital Medicine

Disallow All Ads
Correspondence Location
Anthony A Xu, MD; Email: Anthony.Xu@bcm.edu; Telephone: 512-786-1309; Twitter: @AnthonyXuMD.
Content Gating
Gated (full article locked unless allowed per User)
Alternative CME
Disqus Comments
Default
Use ProPublica
Hide sidebar & use full width
render the right sidebar.
Conference Recap Checkbox
Not Conference Recap
Clinical Edge
Display the Slideshow in this Article
Gating Strategy
First Page Free
Medscape Article
Display survey writer
Reuters content
Disable Inline Native ads
WebMD Article
Article PDF Media

Things We Do For No Reason™: Rasburicase for Adult Patients With Tumor Lysis Syndrome

Article Type
Changed
Fri, 07/02/2021 - 08:38
Display Headline
Things We Do For No Reason™: Rasburicase for Adult Patients With Tumor Lysis Syndrome

Inspired by the ABIM Foundation’s Choosing Wisely ® campaign, the “Things We Do for No Reason™” (TWDFNR) series reviews practices that have become common parts of hospital care but may provide little value to our patients. Practices reviewed in the TWDFNR series do not represent clear-cut conclusions or clinical practice standards but are meant as a starting place for research and active discussions among hospitalists and patients. We invite you to be part of that discussion.

CLINICAL SCENARIO

A 35-year-old man with a history of diffuse large B-cell lymphoma (DLBCL), who most recently received treatment 12 months earlier, presents to the emergency department with abdominal pain and constipation. A computed tomography scan of the abdomen reveals retroperitoneal and mesenteric lymphadenopathy causing small bowel obstruction. The basic metabolic panel reveals a creatinine of 1.1 mg/dL, calcium of 8.5 mg/dL, phosphorus of 4 mg/dL, potassium of 4.5 mEq/L, and uric acid of 7.3 mg/dL. The admitting team contemplates using allopurinol or rasburicase for tumor lysis syndrome (TLS) prevention in the setting of recurrent DLBCL.

BACKGROUND

Tumor lysis syndrome is characterized by metabolic derangement and end-organ damage in the setting of cytotoxic chemotherapy, chemosensitive malignancy, and/or increased tumor burden.1 Risk stratification for TLS takes into account patient and disease characteristics (Table 1). Other risk factors include tumor bulk, elevated baseline serum lactate dehydrogenase, and certain types of chemotherapy (eg, cisplatin, cytarabine, etoposide, paclitaxel, cytotoxic therapies), immunotherapy, or targeted therapy.2 Elevated serum levels of uric acid, potassium, and phosphorus, as well as preexisting renal dysfunction, predispose patients to clinical TLS.3

niforatos1308-0621e_t1.png

The Cairo-Bishop classification system is most frequently used to diagnose TLS (Table 2).3 Laboratory features include hyperkalemia, hyperphosphatemia, hyperuricemia, and hypocalcemia secondary to lysis of proliferating tumor cells and their nuclei. Clinical features include arrhythmias, seizures, and acute kidney injury (AKI).1 Acute kidney injury, the most common clinical complication of TLS, results from crystallization of markedly elevated plasma uric acid, leading to tubular obstruction.1,4 The development of AKI can predict morbidity (namely, the need for renal replacement therapy [RRT]) and mortality in this patient population.1

niforatos1308-0621e_t2.png

Stratifying a patient’s baseline risk of developing TLS often dictates the prevention and management plan. Therapeutic prophylaxis and management strategies for TLS include aggressive fluid resuscitation, diuresis, plasma uric acid (PUA) levels, monitoring electrolyte levels, and, in certain life-threatening situations, dialysis. Oncologists presume reducing uric acid levels prevents and treats TLS.

Current methods to reduce PUA as a means of preventing or treating TLS include xanthine oxidase inhibitors (eg, allopurinol) or urate oxidase (eg, rasburicase). Before the US Food and Drug Administration’s (FDA) approval of rasburicase to manage TLS, providers combined allopurinol (a purine analog that inhibits the enzyme xanthine oxidase, decreasing uric acid level) with aggressive fluid resuscitation. Approved by the FDA in 2002, rasburicase offers an alternative treatment for hyperuricemia by directly decreasing levels of uric acid instead of merely preventing the increased formation of uric acid. As a urate oxidase, rasburicase converts uric acid to the non-nephrotoxic, water-soluble, and freely excreted allantoin.

WHY YOU MIGHT THINK YOU SHOULD USE URATE OXIDASE IN TUMOR LYSIS SYNDROME FOR THE PREVENTION AND MANAGEMENT OF ACUTE KIDNEY INJURY

Rasburicase is often considered the standard-of-care treatment for hyperuricemia due to its ability to reduce circulating uric acid levels rapidly. The primary goal of uric acid reduction is to prevent the occurrence of AKI.

Based upon bioplausible relevance to clinically meaningful endpoints, researchers selected PUA reduction as the primary outcome in randomized controlled trials (RCTs) and observational studies to justify treatment with rasburicase. In RCTs, compassionate trials, and systematic reviews and meta-analyses, rasburicase demonstrated a more rapid reduction in uric acid levels compared to allopurinol.5 Specifically, in one study by Goldman et al,6 rasburicase decreased baseline uric acid levels in pediatric oncology patients by 86% (statistically significant) 4 hours after administration, compared to allopurinol, which only reduced baseline uric acid by 12%. According to a study by Cairo et al, allopurinol may take up to 1 day to reduce PUA.3

WHY URATE OXIDASE MAY NOT IMPROVE CLINICAL OUTCOMES IN PATIENTS AT RISK FOR OR WITH TUMOR LYSIS SYNDROME

Randomized controlled trials examining the safety, efficacy, and cost-effectiveness of rasburicase in adult patients remain sparse. Both RCTs and systematic reviews and meta-analyses rely on PUA levels as a surrogate endpoint and fail to include clinically meaningful primary endpoints (eg, change in baseline creatinine or need for RRT), raising the question as to whether rasburicase improves patient-centered outcomes.5 Since previous studies in the oncology literature show low or modest correlations between PUA reduction and patient-oriented outcomes, we must question whether PUA reduction serves as a meaningful surrogate endpoint.

Treatment of Tumor Lysis Syndrome

Two meta-analyses focusing on the treatment of TLS by Dinnel et al5 and Lopez-Olivo et al8 each included only three unique RCTs (two of the three RCTs were referenced in both meta-analyses). Moreover, both studies included only one RCT comparing rasburicase directly to allopurinol (a 2010 RCT by Cortes et al9) while the other RCTs compared the impact of different rasburicase dosing regimens. Researchers powered the head-to-head RCT by Cortes et al9 to detect a difference in PUA levels across three different arms: rasburicase, rasburicase plus allopurinol, or allopurinol alone. All three treatment arms resulted in a statistically significant reduction in serum PUA levels (87%, 78%, 66%, respectively; P = .001) without a change in the secondary, underpowered clinical outcomes such as clinical TLS or reduced renal function (defined in this study as increased creatinine, renal failure/impairment, or acute renal failure).

More recently, retrospective analyses of patients with AKI secondary to TLS found no difference in creatinine improvement, renal recovery, or prevention of RRT based on whether the patients received either rasburicase or allopurinol.10,11 While rasburicase is associated with greater PUA reduction compared to allopurinol, according to meaningful RCT and observational data as discussed previously and described further in the following section, this does not translate to clinically important risk reduction.

Prevention of Tumor Lysis Syndrome

Furthermore, there exists little compelling evidence to support the use of rasburicase for preventing AKI secondary to TLS. Even among patients at high-risk for TLS (the only group for whom rasburicase is currently recommended),5 rasburicase does not definitively prevent AKI. Data suggest that despite lowering uric acid levels, rasburicase does not consistently prevent renal injury11 or decrease the total number of subsequent inpatient days.12 The only phase 3 trial that compared the efficacy of rasburicase to allopurinol for the prevention of TLS and included clinically meaningful endpoints (eg, renal failure) found that, while rasburicase reduced uric acid levels faster than allopurinol, it did not decrease rates of clinical TLS.9

The published literature offers limited efficacy data of rasburicase in preventing TLS in low-risk patients; however, the absence of benefit of rasburicase in preventing renal failure in high-risk patients warrants skepticism as to its potential efficacy in low-risk patients.8,10

Costs-Effectiveness and Other Ethical Considerations

Rasburicase is an expensive treatment. The estimated cost of the FDA-recommended dosing is around $37,500.13 Moreover, studies comparing the cost-effectiveness of rasburicase to allopurinol focus primarily on patients at high-risk for TLS, which overestimates the cost-effectiveness of rasburicase in patients at low-to-intermediate risk for TLS.14,15 Unfortunately, some providers inappropriately prescribe rasburicase regularly to patients at low or intermediate risk for TLS. Based on observational studies of rasburicase in various clinical scenarios, including inpatient and emergency department settings, inappropriate use of rasburicase (eg, in the setting of hyperuricemia without evidence of a high-risk TLS tumor, no prior trial of allopurinol, preserved renal function, no laboratory evaluation) ranges from 32% to 70%.14,15

Finally, while <1% of patients experience rasburicase-induced anaphylaxis, 20% to 50% of patients develop gastrointestinal symptoms and viral-syndrome-like symptoms.16 Meanwhile, major side effects from allopurinol that occur with 1% to 10% frequency include maculopapular rash, pruritis, gout, nausea, vomiting, and renal failure syndrome.17 Even if the cost for rasburicase and allopurinol were similar, the lack of improved efficacy and the side-effect profiles of the two medications should make us question whether to prescribe rasburicase preferentially over allopurinol.

WHEN MIGHT URATE OXIDASE BE HELPFUL IN TUMOR LYSIS SYNDROME

While some experts recommend rasburicase prophylaxis in patients at high risk for developing TLS, such recommendations rely on low-quality evidence.2 When prescribing rasburicase, the hospitalist must ensure correct dosing. The FDA approved rasburicase for weight-based dosing at 0.2 mg/kg, though current evidence favors a single, fixed dose of 3 mg.16,17 Compared to weight-based dosing, which has an estimated cost-effectiveness ratio ranging from $27,982.77 to $119,643.59 per quality-adjusted life-year, single dosing has equivalent efficacy at approximately 50% lower cost per dose.11,17,18

WHAT YOU SHOULD DO INSTEAD

As a preventive treatment for TLS, clinicians should only consider prescribing rasburicase as a single fixed dose of 3 mg to high-risk patients.17 In the event of AKI secondary to TLS, clinicians should proceed with the mainstay treatment of resuscitation with aggressive fluid resuscitation, with a goal urine output of at least 2 mL/kg/h.1 Fluid resuscitation should be used cautiously in patients with oliguric or anuric AKI, pulmonary hypertension, congestive heart failure, and hemodynamically significant valvular disease. Clinicians should provide continuous cardiac monitoring during the initial presentation to monitor for electrocardiographic changes in the setting of hyperkalemia and hypocalcemia, and they should consult nephrology, oncology, and critical care services early in the disease course to maximize coordination of care.

RECOMMENDATIONS

Prevention

  • Identify patients at high-risk of TLS (Table 1) and consider a single 3-mg dose of rasburicase.
  • Manage low- and intermediate-risk patients with allopurinol and hydration.

Treatment

  • Identify patients with TLS using the clinical and laboratory findings outlined in the Cairo-Bishop classification system (Table 2).
  • Initiate aggressive fluid resuscitation and manage electrolyte abnormalities.
  • If urate-lowering therapy is part of local hospital guidelines for TLS management, consider a single dose regimen of rasburicase utilizing shared decision-making.

CONCLUSION

Tumor lysis syndrome remains a metabolic emergency that requires rapid diagnosis and management to prevent morbidity and mortality. Current data show rasburicase rapidly decreases PUA compared to allopurinol. However, the current literature does not provide compelling evidence that rapidly lowering uric acid with rasburicase to prevent TLS or to treat AKI secondary to TLS improves patient-oriented outcomes.

Do you think this is a low-value practice? Is this truly a “Thing We Do for No Reason™”? Share what you do in your practice and join in the conversation online by retweeting it on Twitter (#TWDFNR) and liking it on Facebook. We invite you to propose ideas for other “Things We Do for No Reason™” topics by emailing TWDFNR@hospitalmedicine.org

References

1. Howard SC, Jones DP, Pui CH. The tumor lysis syndrome. N Engl J Med.2011;364(19):1844-1854. https://doi.org/10.1056/nejmra0904569
2. Cairo MS, Coiffier B, Reiter A, Younes A; TLS Expert Panel. Recommendations for the evaluation of risk and prophylaxis of tumour lysis syndrome (TLS) in adults and children with malignant diseases: an expert TLS panel consensus. Br J Haematol. 2010;149(4):578-586. https://doi.org/10.1111/j.1365-2141.2010.08143.x
3. Cairo MS, Bishop M. Tumour lysis syndrome: new therapeutic strategies and classification. Br J Haematol.. 2004;127(1):3-11. https://doi.org/10.1111/j.1365-2141.2004.05094.x
4. Durani U, Shah ND, Go RS. In-hospital outcomes of tumor lysis syndrome: a population-based study using the National Inpatient Sample. Oncologist. 2017;22(12):1506-1509. https://doi.org/10.1634/theoncologist.2017-0147
5. Dinnel J, Moore BL, Skiver BM, Bose P. Rasburicase in the management of tumor lysis: an evidence-based review of its place in therapy. Core Evid.. 2015;10:23-38. https://doi.org/10.2147/ce.s54995
6. Goldman SC, Holcenberg JS, Finklestein JZ, et al. A randomized comparison between rasburicase and allopurinol in children with lymphoma or leukemia at high risk for tumor lysis. Blood. 2001;97(10):2998-3003. https://doi.org/10.1182/blood.v97.10.2998
7. Haslam A, Hey SP, Gill J, Prasad V. A systematic review of trial-level meta-analyses measuring the strength of association between surrogate end-points and overall survival in oncology. Eur J Cancer. 1990. 2019;106:196-211. https://doi.org/10.1016/j.ejca.2018.11.012
8. Lopez-Olivo MA, Pratt G, Palla SL, Salahudeen A. Rasburicase in tumor lysis syndrome of the adult: a systematic review and meta-analysis. Am J Kidney Dis. 2013;62(3):481-492. https://doi.org/10.1053/j.ajkd.2013.02.378
9. Cortes J, Moore JO, Maziarz RT, et al. Control of plasma uric acid in adults at risk for tumor lysis syndrome: efficacy and safety of rasburicase alone and rasburicase followed by allopurinol compared with allopurinol alone—results of a multicenter phase III study. J Clin Oncol. 2010;28(27):4207-4213. https://doi.org/10.1200/jco.2009.26.8896
10. Martens KL, Khalighi PR, Li S, et al. Comparative effectiveness of rasburicase versus allopurinol for cancer patients with renal dysfunction and hyperuricemia. Leuk Res. 2020;89:106298. https://doi.org/10.1016/j.leukres.2020.106298
11. Personett HA, Barreto EF, McCullough K, Dierkhising R, Leung N, Habermann TM. Impact of early rasburicase on incidence and outcomes of clinical tumor lysis syndrome in lymphoma. Blood. 2019;60(9)2271-2277. https://doi.org/10.1080/10428194.2019.1574000
12. Howard SC, Cockerham AR, Yvonne Barnes DN, Ryan M, Irish W, Gordan L. Real-world analysis of outpatient rasburicase to prevent and manage tumor lysis syndrome in newly diagnosed adults with leukemia or lymphoma. J Clin Pathways. 2020;6(2):46-51.
13. Abu-Hashyeh AM, Shenouda M, Al-Sharedi M. The efficacy of cost-effective fixed dose of rasburicase compared to weight-based dose in treatment and prevention of tumor lysis syndrome (TLS). J Natl Compr Canc Netw. 2020;18(3.5):QIM20-119. https://doi.org/10.6004/jnccn.2019.7516
14. Patel KK, Brown TJ, Gupta A, et al. Decreasing inappropriate use of rasburicase to promote cost-effective care. J Oncol Pract. 2019;15(2):e178-e186. https://doi.org/10.1200/jop.18.00528
15. Khalighi PR, Martens KL, White AA, et al. Utilization patterns and clinical outcomes of rasburicase administration according to tumor risk stratification. J Oncol Pharm Pract. 2020;26(3):529-535. https://doi.org/10.1177/1078155219851543
16. Elitek. Prescribing information. Sanofi-Aventis U.S., LLC; 2019. Accessed June 1, 2021. https://products.sanofi.us/elitek/Elitek.html  
17. Allopurinol. Drugs & Diseases. Medscape. Accessed June 1, 2021. https://reference.medscape.com/drug/zyloprim-aloprim-allopurinol-342811
18. Jones GL, Will A, Jackson GH, Webb NJA, Rule S; British Committee for Standards in Haematology. Guidelines for the management of tumour lysis syndrome in adults and children with haematological malignancies on behalf of the British Committee for Standards in Haematology. Br J Haematol. 2015;169(5):661‐671. https://doi.org/10.1111/bjh.13403
19. Boutin A, Blackman A, O’Sullivan DM, Forcello N. The value of fixed rasburicase dosing versus weight-based dosing in the treatment and prevention of tumor lysis syndrome. J Oncol Pharm Pract. 2019;25(3):577-583. https://doi.org/10.1177/1078155217752075

Article PDF
Author and Disclosure Information

1Department of Emergency Medicine, The Johns Hopkins University School of Medicine, Baltimore, Maryland; 2Department of Internal Medicine, University of Utah Hospital and Clinics, Salt Lake City, Utah; 3Department of Medicine, Brigham and Women’s Hospital/Harvard Medical School, Boston, Massachusetts; 4Department of Hematology/Oncology, University of Mississippi Medical Center, Jackson, Mississippi.

Disclosures
The authors have no conflicts to disclose.

Issue
Journal of Hospital Medicine 16(7)
Publications
Topics
Page Number
424-427. Published Online First June 16, 2021
Sections
Author and Disclosure Information

1Department of Emergency Medicine, The Johns Hopkins University School of Medicine, Baltimore, Maryland; 2Department of Internal Medicine, University of Utah Hospital and Clinics, Salt Lake City, Utah; 3Department of Medicine, Brigham and Women’s Hospital/Harvard Medical School, Boston, Massachusetts; 4Department of Hematology/Oncology, University of Mississippi Medical Center, Jackson, Mississippi.

Disclosures
The authors have no conflicts to disclose.

Author and Disclosure Information

1Department of Emergency Medicine, The Johns Hopkins University School of Medicine, Baltimore, Maryland; 2Department of Internal Medicine, University of Utah Hospital and Clinics, Salt Lake City, Utah; 3Department of Medicine, Brigham and Women’s Hospital/Harvard Medical School, Boston, Massachusetts; 4Department of Hematology/Oncology, University of Mississippi Medical Center, Jackson, Mississippi.

Disclosures
The authors have no conflicts to disclose.

Article PDF
Article PDF
Related Articles

Inspired by the ABIM Foundation’s Choosing Wisely ® campaign, the “Things We Do for No Reason™” (TWDFNR) series reviews practices that have become common parts of hospital care but may provide little value to our patients. Practices reviewed in the TWDFNR series do not represent clear-cut conclusions or clinical practice standards but are meant as a starting place for research and active discussions among hospitalists and patients. We invite you to be part of that discussion.

CLINICAL SCENARIO

A 35-year-old man with a history of diffuse large B-cell lymphoma (DLBCL), who most recently received treatment 12 months earlier, presents to the emergency department with abdominal pain and constipation. A computed tomography scan of the abdomen reveals retroperitoneal and mesenteric lymphadenopathy causing small bowel obstruction. The basic metabolic panel reveals a creatinine of 1.1 mg/dL, calcium of 8.5 mg/dL, phosphorus of 4 mg/dL, potassium of 4.5 mEq/L, and uric acid of 7.3 mg/dL. The admitting team contemplates using allopurinol or rasburicase for tumor lysis syndrome (TLS) prevention in the setting of recurrent DLBCL.

BACKGROUND

Tumor lysis syndrome is characterized by metabolic derangement and end-organ damage in the setting of cytotoxic chemotherapy, chemosensitive malignancy, and/or increased tumor burden.1 Risk stratification for TLS takes into account patient and disease characteristics (Table 1). Other risk factors include tumor bulk, elevated baseline serum lactate dehydrogenase, and certain types of chemotherapy (eg, cisplatin, cytarabine, etoposide, paclitaxel, cytotoxic therapies), immunotherapy, or targeted therapy.2 Elevated serum levels of uric acid, potassium, and phosphorus, as well as preexisting renal dysfunction, predispose patients to clinical TLS.3

niforatos1308-0621e_t1.png

The Cairo-Bishop classification system is most frequently used to diagnose TLS (Table 2).3 Laboratory features include hyperkalemia, hyperphosphatemia, hyperuricemia, and hypocalcemia secondary to lysis of proliferating tumor cells and their nuclei. Clinical features include arrhythmias, seizures, and acute kidney injury (AKI).1 Acute kidney injury, the most common clinical complication of TLS, results from crystallization of markedly elevated plasma uric acid, leading to tubular obstruction.1,4 The development of AKI can predict morbidity (namely, the need for renal replacement therapy [RRT]) and mortality in this patient population.1

niforatos1308-0621e_t2.png

Stratifying a patient’s baseline risk of developing TLS often dictates the prevention and management plan. Therapeutic prophylaxis and management strategies for TLS include aggressive fluid resuscitation, diuresis, plasma uric acid (PUA) levels, monitoring electrolyte levels, and, in certain life-threatening situations, dialysis. Oncologists presume reducing uric acid levels prevents and treats TLS.

Current methods to reduce PUA as a means of preventing or treating TLS include xanthine oxidase inhibitors (eg, allopurinol) or urate oxidase (eg, rasburicase). Before the US Food and Drug Administration’s (FDA) approval of rasburicase to manage TLS, providers combined allopurinol (a purine analog that inhibits the enzyme xanthine oxidase, decreasing uric acid level) with aggressive fluid resuscitation. Approved by the FDA in 2002, rasburicase offers an alternative treatment for hyperuricemia by directly decreasing levels of uric acid instead of merely preventing the increased formation of uric acid. As a urate oxidase, rasburicase converts uric acid to the non-nephrotoxic, water-soluble, and freely excreted allantoin.

WHY YOU MIGHT THINK YOU SHOULD USE URATE OXIDASE IN TUMOR LYSIS SYNDROME FOR THE PREVENTION AND MANAGEMENT OF ACUTE KIDNEY INJURY

Rasburicase is often considered the standard-of-care treatment for hyperuricemia due to its ability to reduce circulating uric acid levels rapidly. The primary goal of uric acid reduction is to prevent the occurrence of AKI.

Based upon bioplausible relevance to clinically meaningful endpoints, researchers selected PUA reduction as the primary outcome in randomized controlled trials (RCTs) and observational studies to justify treatment with rasburicase. In RCTs, compassionate trials, and systematic reviews and meta-analyses, rasburicase demonstrated a more rapid reduction in uric acid levels compared to allopurinol.5 Specifically, in one study by Goldman et al,6 rasburicase decreased baseline uric acid levels in pediatric oncology patients by 86% (statistically significant) 4 hours after administration, compared to allopurinol, which only reduced baseline uric acid by 12%. According to a study by Cairo et al, allopurinol may take up to 1 day to reduce PUA.3

WHY URATE OXIDASE MAY NOT IMPROVE CLINICAL OUTCOMES IN PATIENTS AT RISK FOR OR WITH TUMOR LYSIS SYNDROME

Randomized controlled trials examining the safety, efficacy, and cost-effectiveness of rasburicase in adult patients remain sparse. Both RCTs and systematic reviews and meta-analyses rely on PUA levels as a surrogate endpoint and fail to include clinically meaningful primary endpoints (eg, change in baseline creatinine or need for RRT), raising the question as to whether rasburicase improves patient-centered outcomes.5 Since previous studies in the oncology literature show low or modest correlations between PUA reduction and patient-oriented outcomes, we must question whether PUA reduction serves as a meaningful surrogate endpoint.

Treatment of Tumor Lysis Syndrome

Two meta-analyses focusing on the treatment of TLS by Dinnel et al5 and Lopez-Olivo et al8 each included only three unique RCTs (two of the three RCTs were referenced in both meta-analyses). Moreover, both studies included only one RCT comparing rasburicase directly to allopurinol (a 2010 RCT by Cortes et al9) while the other RCTs compared the impact of different rasburicase dosing regimens. Researchers powered the head-to-head RCT by Cortes et al9 to detect a difference in PUA levels across three different arms: rasburicase, rasburicase plus allopurinol, or allopurinol alone. All three treatment arms resulted in a statistically significant reduction in serum PUA levels (87%, 78%, 66%, respectively; P = .001) without a change in the secondary, underpowered clinical outcomes such as clinical TLS or reduced renal function (defined in this study as increased creatinine, renal failure/impairment, or acute renal failure).

More recently, retrospective analyses of patients with AKI secondary to TLS found no difference in creatinine improvement, renal recovery, or prevention of RRT based on whether the patients received either rasburicase or allopurinol.10,11 While rasburicase is associated with greater PUA reduction compared to allopurinol, according to meaningful RCT and observational data as discussed previously and described further in the following section, this does not translate to clinically important risk reduction.

Prevention of Tumor Lysis Syndrome

Furthermore, there exists little compelling evidence to support the use of rasburicase for preventing AKI secondary to TLS. Even among patients at high-risk for TLS (the only group for whom rasburicase is currently recommended),5 rasburicase does not definitively prevent AKI. Data suggest that despite lowering uric acid levels, rasburicase does not consistently prevent renal injury11 or decrease the total number of subsequent inpatient days.12 The only phase 3 trial that compared the efficacy of rasburicase to allopurinol for the prevention of TLS and included clinically meaningful endpoints (eg, renal failure) found that, while rasburicase reduced uric acid levels faster than allopurinol, it did not decrease rates of clinical TLS.9

The published literature offers limited efficacy data of rasburicase in preventing TLS in low-risk patients; however, the absence of benefit of rasburicase in preventing renal failure in high-risk patients warrants skepticism as to its potential efficacy in low-risk patients.8,10

Costs-Effectiveness and Other Ethical Considerations

Rasburicase is an expensive treatment. The estimated cost of the FDA-recommended dosing is around $37,500.13 Moreover, studies comparing the cost-effectiveness of rasburicase to allopurinol focus primarily on patients at high-risk for TLS, which overestimates the cost-effectiveness of rasburicase in patients at low-to-intermediate risk for TLS.14,15 Unfortunately, some providers inappropriately prescribe rasburicase regularly to patients at low or intermediate risk for TLS. Based on observational studies of rasburicase in various clinical scenarios, including inpatient and emergency department settings, inappropriate use of rasburicase (eg, in the setting of hyperuricemia without evidence of a high-risk TLS tumor, no prior trial of allopurinol, preserved renal function, no laboratory evaluation) ranges from 32% to 70%.14,15

Finally, while <1% of patients experience rasburicase-induced anaphylaxis, 20% to 50% of patients develop gastrointestinal symptoms and viral-syndrome-like symptoms.16 Meanwhile, major side effects from allopurinol that occur with 1% to 10% frequency include maculopapular rash, pruritis, gout, nausea, vomiting, and renal failure syndrome.17 Even if the cost for rasburicase and allopurinol were similar, the lack of improved efficacy and the side-effect profiles of the two medications should make us question whether to prescribe rasburicase preferentially over allopurinol.

WHEN MIGHT URATE OXIDASE BE HELPFUL IN TUMOR LYSIS SYNDROME

While some experts recommend rasburicase prophylaxis in patients at high risk for developing TLS, such recommendations rely on low-quality evidence.2 When prescribing rasburicase, the hospitalist must ensure correct dosing. The FDA approved rasburicase for weight-based dosing at 0.2 mg/kg, though current evidence favors a single, fixed dose of 3 mg.16,17 Compared to weight-based dosing, which has an estimated cost-effectiveness ratio ranging from $27,982.77 to $119,643.59 per quality-adjusted life-year, single dosing has equivalent efficacy at approximately 50% lower cost per dose.11,17,18

WHAT YOU SHOULD DO INSTEAD

As a preventive treatment for TLS, clinicians should only consider prescribing rasburicase as a single fixed dose of 3 mg to high-risk patients.17 In the event of AKI secondary to TLS, clinicians should proceed with the mainstay treatment of resuscitation with aggressive fluid resuscitation, with a goal urine output of at least 2 mL/kg/h.1 Fluid resuscitation should be used cautiously in patients with oliguric or anuric AKI, pulmonary hypertension, congestive heart failure, and hemodynamically significant valvular disease. Clinicians should provide continuous cardiac monitoring during the initial presentation to monitor for electrocardiographic changes in the setting of hyperkalemia and hypocalcemia, and they should consult nephrology, oncology, and critical care services early in the disease course to maximize coordination of care.

RECOMMENDATIONS

Prevention

  • Identify patients at high-risk of TLS (Table 1) and consider a single 3-mg dose of rasburicase.
  • Manage low- and intermediate-risk patients with allopurinol and hydration.

Treatment

  • Identify patients with TLS using the clinical and laboratory findings outlined in the Cairo-Bishop classification system (Table 2).
  • Initiate aggressive fluid resuscitation and manage electrolyte abnormalities.
  • If urate-lowering therapy is part of local hospital guidelines for TLS management, consider a single dose regimen of rasburicase utilizing shared decision-making.

CONCLUSION

Tumor lysis syndrome remains a metabolic emergency that requires rapid diagnosis and management to prevent morbidity and mortality. Current data show rasburicase rapidly decreases PUA compared to allopurinol. However, the current literature does not provide compelling evidence that rapidly lowering uric acid with rasburicase to prevent TLS or to treat AKI secondary to TLS improves patient-oriented outcomes.

Do you think this is a low-value practice? Is this truly a “Thing We Do for No Reason™”? Share what you do in your practice and join in the conversation online by retweeting it on Twitter (#TWDFNR) and liking it on Facebook. We invite you to propose ideas for other “Things We Do for No Reason™” topics by emailing TWDFNR@hospitalmedicine.org

Inspired by the ABIM Foundation’s Choosing Wisely ® campaign, the “Things We Do for No Reason™” (TWDFNR) series reviews practices that have become common parts of hospital care but may provide little value to our patients. Practices reviewed in the TWDFNR series do not represent clear-cut conclusions or clinical practice standards but are meant as a starting place for research and active discussions among hospitalists and patients. We invite you to be part of that discussion.

CLINICAL SCENARIO

A 35-year-old man with a history of diffuse large B-cell lymphoma (DLBCL), who most recently received treatment 12 months earlier, presents to the emergency department with abdominal pain and constipation. A computed tomography scan of the abdomen reveals retroperitoneal and mesenteric lymphadenopathy causing small bowel obstruction. The basic metabolic panel reveals a creatinine of 1.1 mg/dL, calcium of 8.5 mg/dL, phosphorus of 4 mg/dL, potassium of 4.5 mEq/L, and uric acid of 7.3 mg/dL. The admitting team contemplates using allopurinol or rasburicase for tumor lysis syndrome (TLS) prevention in the setting of recurrent DLBCL.

BACKGROUND

Tumor lysis syndrome is characterized by metabolic derangement and end-organ damage in the setting of cytotoxic chemotherapy, chemosensitive malignancy, and/or increased tumor burden.1 Risk stratification for TLS takes into account patient and disease characteristics (Table 1). Other risk factors include tumor bulk, elevated baseline serum lactate dehydrogenase, and certain types of chemotherapy (eg, cisplatin, cytarabine, etoposide, paclitaxel, cytotoxic therapies), immunotherapy, or targeted therapy.2 Elevated serum levels of uric acid, potassium, and phosphorus, as well as preexisting renal dysfunction, predispose patients to clinical TLS.3

niforatos1308-0621e_t1.png

The Cairo-Bishop classification system is most frequently used to diagnose TLS (Table 2).3 Laboratory features include hyperkalemia, hyperphosphatemia, hyperuricemia, and hypocalcemia secondary to lysis of proliferating tumor cells and their nuclei. Clinical features include arrhythmias, seizures, and acute kidney injury (AKI).1 Acute kidney injury, the most common clinical complication of TLS, results from crystallization of markedly elevated plasma uric acid, leading to tubular obstruction.1,4 The development of AKI can predict morbidity (namely, the need for renal replacement therapy [RRT]) and mortality in this patient population.1

niforatos1308-0621e_t2.png

Stratifying a patient’s baseline risk of developing TLS often dictates the prevention and management plan. Therapeutic prophylaxis and management strategies for TLS include aggressive fluid resuscitation, diuresis, plasma uric acid (PUA) levels, monitoring electrolyte levels, and, in certain life-threatening situations, dialysis. Oncologists presume reducing uric acid levels prevents and treats TLS.

Current methods to reduce PUA as a means of preventing or treating TLS include xanthine oxidase inhibitors (eg, allopurinol) or urate oxidase (eg, rasburicase). Before the US Food and Drug Administration’s (FDA) approval of rasburicase to manage TLS, providers combined allopurinol (a purine analog that inhibits the enzyme xanthine oxidase, decreasing uric acid level) with aggressive fluid resuscitation. Approved by the FDA in 2002, rasburicase offers an alternative treatment for hyperuricemia by directly decreasing levels of uric acid instead of merely preventing the increased formation of uric acid. As a urate oxidase, rasburicase converts uric acid to the non-nephrotoxic, water-soluble, and freely excreted allantoin.

WHY YOU MIGHT THINK YOU SHOULD USE URATE OXIDASE IN TUMOR LYSIS SYNDROME FOR THE PREVENTION AND MANAGEMENT OF ACUTE KIDNEY INJURY

Rasburicase is often considered the standard-of-care treatment for hyperuricemia due to its ability to reduce circulating uric acid levels rapidly. The primary goal of uric acid reduction is to prevent the occurrence of AKI.

Based upon bioplausible relevance to clinically meaningful endpoints, researchers selected PUA reduction as the primary outcome in randomized controlled trials (RCTs) and observational studies to justify treatment with rasburicase. In RCTs, compassionate trials, and systematic reviews and meta-analyses, rasburicase demonstrated a more rapid reduction in uric acid levels compared to allopurinol.5 Specifically, in one study by Goldman et al,6 rasburicase decreased baseline uric acid levels in pediatric oncology patients by 86% (statistically significant) 4 hours after administration, compared to allopurinol, which only reduced baseline uric acid by 12%. According to a study by Cairo et al, allopurinol may take up to 1 day to reduce PUA.3

WHY URATE OXIDASE MAY NOT IMPROVE CLINICAL OUTCOMES IN PATIENTS AT RISK FOR OR WITH TUMOR LYSIS SYNDROME

Randomized controlled trials examining the safety, efficacy, and cost-effectiveness of rasburicase in adult patients remain sparse. Both RCTs and systematic reviews and meta-analyses rely on PUA levels as a surrogate endpoint and fail to include clinically meaningful primary endpoints (eg, change in baseline creatinine or need for RRT), raising the question as to whether rasburicase improves patient-centered outcomes.5 Since previous studies in the oncology literature show low or modest correlations between PUA reduction and patient-oriented outcomes, we must question whether PUA reduction serves as a meaningful surrogate endpoint.

Treatment of Tumor Lysis Syndrome

Two meta-analyses focusing on the treatment of TLS by Dinnel et al5 and Lopez-Olivo et al8 each included only three unique RCTs (two of the three RCTs were referenced in both meta-analyses). Moreover, both studies included only one RCT comparing rasburicase directly to allopurinol (a 2010 RCT by Cortes et al9) while the other RCTs compared the impact of different rasburicase dosing regimens. Researchers powered the head-to-head RCT by Cortes et al9 to detect a difference in PUA levels across three different arms: rasburicase, rasburicase plus allopurinol, or allopurinol alone. All three treatment arms resulted in a statistically significant reduction in serum PUA levels (87%, 78%, 66%, respectively; P = .001) without a change in the secondary, underpowered clinical outcomes such as clinical TLS or reduced renal function (defined in this study as increased creatinine, renal failure/impairment, or acute renal failure).

More recently, retrospective analyses of patients with AKI secondary to TLS found no difference in creatinine improvement, renal recovery, or prevention of RRT based on whether the patients received either rasburicase or allopurinol.10,11 While rasburicase is associated with greater PUA reduction compared to allopurinol, according to meaningful RCT and observational data as discussed previously and described further in the following section, this does not translate to clinically important risk reduction.

Prevention of Tumor Lysis Syndrome

Furthermore, there exists little compelling evidence to support the use of rasburicase for preventing AKI secondary to TLS. Even among patients at high-risk for TLS (the only group for whom rasburicase is currently recommended),5 rasburicase does not definitively prevent AKI. Data suggest that despite lowering uric acid levels, rasburicase does not consistently prevent renal injury11 or decrease the total number of subsequent inpatient days.12 The only phase 3 trial that compared the efficacy of rasburicase to allopurinol for the prevention of TLS and included clinically meaningful endpoints (eg, renal failure) found that, while rasburicase reduced uric acid levels faster than allopurinol, it did not decrease rates of clinical TLS.9

The published literature offers limited efficacy data of rasburicase in preventing TLS in low-risk patients; however, the absence of benefit of rasburicase in preventing renal failure in high-risk patients warrants skepticism as to its potential efficacy in low-risk patients.8,10

Costs-Effectiveness and Other Ethical Considerations

Rasburicase is an expensive treatment. The estimated cost of the FDA-recommended dosing is around $37,500.13 Moreover, studies comparing the cost-effectiveness of rasburicase to allopurinol focus primarily on patients at high-risk for TLS, which overestimates the cost-effectiveness of rasburicase in patients at low-to-intermediate risk for TLS.14,15 Unfortunately, some providers inappropriately prescribe rasburicase regularly to patients at low or intermediate risk for TLS. Based on observational studies of rasburicase in various clinical scenarios, including inpatient and emergency department settings, inappropriate use of rasburicase (eg, in the setting of hyperuricemia without evidence of a high-risk TLS tumor, no prior trial of allopurinol, preserved renal function, no laboratory evaluation) ranges from 32% to 70%.14,15

Finally, while <1% of patients experience rasburicase-induced anaphylaxis, 20% to 50% of patients develop gastrointestinal symptoms and viral-syndrome-like symptoms.16 Meanwhile, major side effects from allopurinol that occur with 1% to 10% frequency include maculopapular rash, pruritis, gout, nausea, vomiting, and renal failure syndrome.17 Even if the cost for rasburicase and allopurinol were similar, the lack of improved efficacy and the side-effect profiles of the two medications should make us question whether to prescribe rasburicase preferentially over allopurinol.

WHEN MIGHT URATE OXIDASE BE HELPFUL IN TUMOR LYSIS SYNDROME

While some experts recommend rasburicase prophylaxis in patients at high risk for developing TLS, such recommendations rely on low-quality evidence.2 When prescribing rasburicase, the hospitalist must ensure correct dosing. The FDA approved rasburicase for weight-based dosing at 0.2 mg/kg, though current evidence favors a single, fixed dose of 3 mg.16,17 Compared to weight-based dosing, which has an estimated cost-effectiveness ratio ranging from $27,982.77 to $119,643.59 per quality-adjusted life-year, single dosing has equivalent efficacy at approximately 50% lower cost per dose.11,17,18

WHAT YOU SHOULD DO INSTEAD

As a preventive treatment for TLS, clinicians should only consider prescribing rasburicase as a single fixed dose of 3 mg to high-risk patients.17 In the event of AKI secondary to TLS, clinicians should proceed with the mainstay treatment of resuscitation with aggressive fluid resuscitation, with a goal urine output of at least 2 mL/kg/h.1 Fluid resuscitation should be used cautiously in patients with oliguric or anuric AKI, pulmonary hypertension, congestive heart failure, and hemodynamically significant valvular disease. Clinicians should provide continuous cardiac monitoring during the initial presentation to monitor for electrocardiographic changes in the setting of hyperkalemia and hypocalcemia, and they should consult nephrology, oncology, and critical care services early in the disease course to maximize coordination of care.

RECOMMENDATIONS

Prevention

  • Identify patients at high-risk of TLS (Table 1) and consider a single 3-mg dose of rasburicase.
  • Manage low- and intermediate-risk patients with allopurinol and hydration.

Treatment

  • Identify patients with TLS using the clinical and laboratory findings outlined in the Cairo-Bishop classification system (Table 2).
  • Initiate aggressive fluid resuscitation and manage electrolyte abnormalities.
  • If urate-lowering therapy is part of local hospital guidelines for TLS management, consider a single dose regimen of rasburicase utilizing shared decision-making.

CONCLUSION

Tumor lysis syndrome remains a metabolic emergency that requires rapid diagnosis and management to prevent morbidity and mortality. Current data show rasburicase rapidly decreases PUA compared to allopurinol. However, the current literature does not provide compelling evidence that rapidly lowering uric acid with rasburicase to prevent TLS or to treat AKI secondary to TLS improves patient-oriented outcomes.

Do you think this is a low-value practice? Is this truly a “Thing We Do for No Reason™”? Share what you do in your practice and join in the conversation online by retweeting it on Twitter (#TWDFNR) and liking it on Facebook. We invite you to propose ideas for other “Things We Do for No Reason™” topics by emailing TWDFNR@hospitalmedicine.org

References

1. Howard SC, Jones DP, Pui CH. The tumor lysis syndrome. N Engl J Med.2011;364(19):1844-1854. https://doi.org/10.1056/nejmra0904569
2. Cairo MS, Coiffier B, Reiter A, Younes A; TLS Expert Panel. Recommendations for the evaluation of risk and prophylaxis of tumour lysis syndrome (TLS) in adults and children with malignant diseases: an expert TLS panel consensus. Br J Haematol. 2010;149(4):578-586. https://doi.org/10.1111/j.1365-2141.2010.08143.x
3. Cairo MS, Bishop M. Tumour lysis syndrome: new therapeutic strategies and classification. Br J Haematol.. 2004;127(1):3-11. https://doi.org/10.1111/j.1365-2141.2004.05094.x
4. Durani U, Shah ND, Go RS. In-hospital outcomes of tumor lysis syndrome: a population-based study using the National Inpatient Sample. Oncologist. 2017;22(12):1506-1509. https://doi.org/10.1634/theoncologist.2017-0147
5. Dinnel J, Moore BL, Skiver BM, Bose P. Rasburicase in the management of tumor lysis: an evidence-based review of its place in therapy. Core Evid.. 2015;10:23-38. https://doi.org/10.2147/ce.s54995
6. Goldman SC, Holcenberg JS, Finklestein JZ, et al. A randomized comparison between rasburicase and allopurinol in children with lymphoma or leukemia at high risk for tumor lysis. Blood. 2001;97(10):2998-3003. https://doi.org/10.1182/blood.v97.10.2998
7. Haslam A, Hey SP, Gill J, Prasad V. A systematic review of trial-level meta-analyses measuring the strength of association between surrogate end-points and overall survival in oncology. Eur J Cancer. 1990. 2019;106:196-211. https://doi.org/10.1016/j.ejca.2018.11.012
8. Lopez-Olivo MA, Pratt G, Palla SL, Salahudeen A. Rasburicase in tumor lysis syndrome of the adult: a systematic review and meta-analysis. Am J Kidney Dis. 2013;62(3):481-492. https://doi.org/10.1053/j.ajkd.2013.02.378
9. Cortes J, Moore JO, Maziarz RT, et al. Control of plasma uric acid in adults at risk for tumor lysis syndrome: efficacy and safety of rasburicase alone and rasburicase followed by allopurinol compared with allopurinol alone—results of a multicenter phase III study. J Clin Oncol. 2010;28(27):4207-4213. https://doi.org/10.1200/jco.2009.26.8896
10. Martens KL, Khalighi PR, Li S, et al. Comparative effectiveness of rasburicase versus allopurinol for cancer patients with renal dysfunction and hyperuricemia. Leuk Res. 2020;89:106298. https://doi.org/10.1016/j.leukres.2020.106298
11. Personett HA, Barreto EF, McCullough K, Dierkhising R, Leung N, Habermann TM. Impact of early rasburicase on incidence and outcomes of clinical tumor lysis syndrome in lymphoma. Blood. 2019;60(9)2271-2277. https://doi.org/10.1080/10428194.2019.1574000
12. Howard SC, Cockerham AR, Yvonne Barnes DN, Ryan M, Irish W, Gordan L. Real-world analysis of outpatient rasburicase to prevent and manage tumor lysis syndrome in newly diagnosed adults with leukemia or lymphoma. J Clin Pathways. 2020;6(2):46-51.
13. Abu-Hashyeh AM, Shenouda M, Al-Sharedi M. The efficacy of cost-effective fixed dose of rasburicase compared to weight-based dose in treatment and prevention of tumor lysis syndrome (TLS). J Natl Compr Canc Netw. 2020;18(3.5):QIM20-119. https://doi.org/10.6004/jnccn.2019.7516
14. Patel KK, Brown TJ, Gupta A, et al. Decreasing inappropriate use of rasburicase to promote cost-effective care. J Oncol Pract. 2019;15(2):e178-e186. https://doi.org/10.1200/jop.18.00528
15. Khalighi PR, Martens KL, White AA, et al. Utilization patterns and clinical outcomes of rasburicase administration according to tumor risk stratification. J Oncol Pharm Pract. 2020;26(3):529-535. https://doi.org/10.1177/1078155219851543
16. Elitek. Prescribing information. Sanofi-Aventis U.S., LLC; 2019. Accessed June 1, 2021. https://products.sanofi.us/elitek/Elitek.html  
17. Allopurinol. Drugs & Diseases. Medscape. Accessed June 1, 2021. https://reference.medscape.com/drug/zyloprim-aloprim-allopurinol-342811
18. Jones GL, Will A, Jackson GH, Webb NJA, Rule S; British Committee for Standards in Haematology. Guidelines for the management of tumour lysis syndrome in adults and children with haematological malignancies on behalf of the British Committee for Standards in Haematology. Br J Haematol. 2015;169(5):661‐671. https://doi.org/10.1111/bjh.13403
19. Boutin A, Blackman A, O’Sullivan DM, Forcello N. The value of fixed rasburicase dosing versus weight-based dosing in the treatment and prevention of tumor lysis syndrome. J Oncol Pharm Pract. 2019;25(3):577-583. https://doi.org/10.1177/1078155217752075

References

1. Howard SC, Jones DP, Pui CH. The tumor lysis syndrome. N Engl J Med.2011;364(19):1844-1854. https://doi.org/10.1056/nejmra0904569
2. Cairo MS, Coiffier B, Reiter A, Younes A; TLS Expert Panel. Recommendations for the evaluation of risk and prophylaxis of tumour lysis syndrome (TLS) in adults and children with malignant diseases: an expert TLS panel consensus. Br J Haematol. 2010;149(4):578-586. https://doi.org/10.1111/j.1365-2141.2010.08143.x
3. Cairo MS, Bishop M. Tumour lysis syndrome: new therapeutic strategies and classification. Br J Haematol.. 2004;127(1):3-11. https://doi.org/10.1111/j.1365-2141.2004.05094.x
4. Durani U, Shah ND, Go RS. In-hospital outcomes of tumor lysis syndrome: a population-based study using the National Inpatient Sample. Oncologist. 2017;22(12):1506-1509. https://doi.org/10.1634/theoncologist.2017-0147
5. Dinnel J, Moore BL, Skiver BM, Bose P. Rasburicase in the management of tumor lysis: an evidence-based review of its place in therapy. Core Evid.. 2015;10:23-38. https://doi.org/10.2147/ce.s54995
6. Goldman SC, Holcenberg JS, Finklestein JZ, et al. A randomized comparison between rasburicase and allopurinol in children with lymphoma or leukemia at high risk for tumor lysis. Blood. 2001;97(10):2998-3003. https://doi.org/10.1182/blood.v97.10.2998
7. Haslam A, Hey SP, Gill J, Prasad V. A systematic review of trial-level meta-analyses measuring the strength of association between surrogate end-points and overall survival in oncology. Eur J Cancer. 1990. 2019;106:196-211. https://doi.org/10.1016/j.ejca.2018.11.012
8. Lopez-Olivo MA, Pratt G, Palla SL, Salahudeen A. Rasburicase in tumor lysis syndrome of the adult: a systematic review and meta-analysis. Am J Kidney Dis. 2013;62(3):481-492. https://doi.org/10.1053/j.ajkd.2013.02.378
9. Cortes J, Moore JO, Maziarz RT, et al. Control of plasma uric acid in adults at risk for tumor lysis syndrome: efficacy and safety of rasburicase alone and rasburicase followed by allopurinol compared with allopurinol alone—results of a multicenter phase III study. J Clin Oncol. 2010;28(27):4207-4213. https://doi.org/10.1200/jco.2009.26.8896
10. Martens KL, Khalighi PR, Li S, et al. Comparative effectiveness of rasburicase versus allopurinol for cancer patients with renal dysfunction and hyperuricemia. Leuk Res. 2020;89:106298. https://doi.org/10.1016/j.leukres.2020.106298
11. Personett HA, Barreto EF, McCullough K, Dierkhising R, Leung N, Habermann TM. Impact of early rasburicase on incidence and outcomes of clinical tumor lysis syndrome in lymphoma. Blood. 2019;60(9)2271-2277. https://doi.org/10.1080/10428194.2019.1574000
12. Howard SC, Cockerham AR, Yvonne Barnes DN, Ryan M, Irish W, Gordan L. Real-world analysis of outpatient rasburicase to prevent and manage tumor lysis syndrome in newly diagnosed adults with leukemia or lymphoma. J Clin Pathways. 2020;6(2):46-51.
13. Abu-Hashyeh AM, Shenouda M, Al-Sharedi M. The efficacy of cost-effective fixed dose of rasburicase compared to weight-based dose in treatment and prevention of tumor lysis syndrome (TLS). J Natl Compr Canc Netw. 2020;18(3.5):QIM20-119. https://doi.org/10.6004/jnccn.2019.7516
14. Patel KK, Brown TJ, Gupta A, et al. Decreasing inappropriate use of rasburicase to promote cost-effective care. J Oncol Pract. 2019;15(2):e178-e186. https://doi.org/10.1200/jop.18.00528
15. Khalighi PR, Martens KL, White AA, et al. Utilization patterns and clinical outcomes of rasburicase administration according to tumor risk stratification. J Oncol Pharm Pract. 2020;26(3):529-535. https://doi.org/10.1177/1078155219851543
16. Elitek. Prescribing information. Sanofi-Aventis U.S., LLC; 2019. Accessed June 1, 2021. https://products.sanofi.us/elitek/Elitek.html  
17. Allopurinol. Drugs & Diseases. Medscape. Accessed June 1, 2021. https://reference.medscape.com/drug/zyloprim-aloprim-allopurinol-342811
18. Jones GL, Will A, Jackson GH, Webb NJA, Rule S; British Committee for Standards in Haematology. Guidelines for the management of tumour lysis syndrome in adults and children with haematological malignancies on behalf of the British Committee for Standards in Haematology. Br J Haematol. 2015;169(5):661‐671. https://doi.org/10.1111/bjh.13403
19. Boutin A, Blackman A, O’Sullivan DM, Forcello N. The value of fixed rasburicase dosing versus weight-based dosing in the treatment and prevention of tumor lysis syndrome. J Oncol Pharm Pract. 2019;25(3):577-583. https://doi.org/10.1177/1078155217752075

Issue
Journal of Hospital Medicine 16(7)
Issue
Journal of Hospital Medicine 16(7)
Page Number
424-427. Published Online First June 16, 2021
Page Number
424-427. Published Online First June 16, 2021
Publications
Publications
Topics
Article Type
Display Headline
Things We Do For No Reason™: Rasburicase for Adult Patients With Tumor Lysis Syndrome
Display Headline
Things We Do For No Reason™: Rasburicase for Adult Patients With Tumor Lysis Syndrome
Sections
Article Source

© 2021 Society of Hospital Medicine

Disallow All Ads
Correspondence Location
Joshua D Niforatos, MD, MTS; Email: jnifora1@jhmi.edu; Telephone: 505-554-4644; Twitter: @ReverendofDoubt.
Content Gating
Gated (full article locked unless allowed per User)
Alternative CME
Disqus Comments
Default
Use ProPublica
Hide sidebar & use full width
render the right sidebar.
Conference Recap Checkbox
Not Conference Recap
Clinical Edge
Display the Slideshow in this Article
Gating Strategy
First Page Free
Medscape Article
Display survey writer
Reuters content
Disable Inline Native ads
WebMD Article
Article PDF Media
Image
Teambase ID
18001CEB.SIG
Disable zoom
Off

Things We Do for No Reason™: Calculating a “Corrected Calcium” Level

Article Type
Changed
Thu, 07/29/2021 - 11:03
Display Headline
Things We Do for No Reason™: Calculating a “Corrected Calcium” Level

Inspired by the ABIM Foundation’s Choosing Wisely® campaign, the “Things We Do for No Reason™” (TWDFNR) series reviews practices that have become common parts of hospital care but may provide little value to our patients. Practices reviewed in the TWDFNR series do not represent clear-cut conclusions or clinical practice standards but are meant as a starting place for research and active discussions among hospitalists and patients. We invite you to be part of that discussion.

CLINICAL SCENARIO

A hospitalist admits a 75-year-old man for evaluation of acute pyelonephritis; the patient’s medical history is significant for chronic kidney disease and nephrotic syndrome. The patient endorses moderate flank pain upon palpation. Initial serum laboratory studies reveal an albumin level of 1.5 g/dL and a calcium level of 10.0 mg/dL. A repeat serum calcium assessment produces similar results. The hospitalist corrects calcium for albumin concentration by applying the most common formula (Payne’s formula), which results in a corrected calcium value of 12 mg/dL. The hospitalist then starts the patient on intravenous (IV) fluids to treat hypercalcemia and obtains serum 25-hydroxyvitamin D and parathyroid hormone levels.

BACKGROUND

Our skeletons bind, with phosphate, nearly 99% of the body’s calcium, the most abundant mineral in our body. The remaining 1% of calcium (approximately 9-10.5 mg/dL) circulates in the blood. Approximately 40% of serum calcium is bound to albumin, with a smaller percentage bound to lactate and citrate. The remaining 4.5 to 5.5 mg/dL circulates unbound as free (ie, ionized) calcium (iCa).1 Calcium has many fundamental intra- and extracellular functions. Physiologic calcium homeostasis is maintained by parathyroid hormone and vitamin D.2 The amount of circulating iCa, rather than total plasma calcium, determines the many biologic effects of plasma calcium.

In the hospital setting, clinicians commonly encounter patients with derangements in calcium homeostasis.3 True hypercalcemia or hypocalcemia has significant clinical manifestations, including generalized fatigue, nephrolithiasis, cardiac arrhythmias, and, potentially, death. Thus, clinical practice requires correct and accurate assessment of serum calcium levels.1

WHY YOU MIGHT THINK CALCULATING A “CORRECTED CALCIUM” LEVEL IS HELPFUL

Although measuring biologically active calcium (ie, iCa) is the gold standard for assessing calcium levels, laboratories struggle to obtain a direct, accurate measurement of iCa due to the special handling and time constraints required to process samples.4 As a result, metabolic laboratory panels typically report the more easily measured total calcium, the sum of iCa and bound calcium.5 Changes in albumin levels, however, do not affect iCa levels. Since calcium has less available albumin for binding, hypoalbuminemia should theoretically decrease the amount of bound calcium and lead to a decreased reported total calcium. Therefore, a patient’s total calcium level may appear low even though their iCa is normal, which can lead to an incorrect diagnosis of hypocalcemia or overestimate of the extent of existing hypocalcemia. Moreover, these lower reported calcium levels can falsely report normocalcemia in patients with hypercalcemia or underestimate the extent of the patient’s hypercalcemia.

For years physicians have attempted to account for the underestimate in total calcium due to hypoalbuminemia by calculating a “corrected” calcium. The correction formulas use total calcium and serum albumin to estimate the expected iCa. Refinements to the original formula, developed by Payne et al in 1973, have resulted in the most commonly utilized formula today: corrected calcium = (0.8 x [normal albumin – patient’s albumin]) + serum calcium.6,7 Many commonly used clinical-decision resources recommend correcting serum calcium concentrations in patients with hypoalbuminemia.6

WHY CALCULATING A CORRECTED CALCIUM FOR ALBUMIN IS UNNECESSARY

While calculating corrected calcium should theoretically provide a more accurate estimate of physiologically active iCa in patients with hypoalbuminemia,4 the commonly used correction equations become less accurate as hypoalbuminemia worsens.8 Payne et al derived the original formula from 200 patients using a single laboratory; however, subsequent retrospective studies have not supported the use of albumin-corrected calcium calculations to estimate the iCa.4,9-11 For example, although Payne’s corrected calcium equations assume a constant relationship between albumin and calcium binding throughout all serum-albumin concentrations, studies have shown that as albumin falls, more calcium ions bind to each available gram of albumin. Payne’s assumption results in an overestimation of the total serum calcium after correction as compared to the iCa.8 In comparison, uncorrected total serum calcium assays more accurately reflect both the change in albumin binding that occurs with alterations in albumin concentration and the unchanged free calcium ions. Studies demonstrate superior correlation between iCa and uncorrected total calcium.4,9-11

Several large retrospective studies revealed the poor in vivo accuracy of equations used to correct calcium for albumin. In one study, Uppsala University Sweden researchers reviewed the laboratory records of more than 20,000 hospitalized patients from 2005 to 2013.9 This group compared seven corrected calcium formulas to direct measurements of iCa. All of the correction equations correlated poorly with iCa based on their intraclass correlation (ICC), a descriptive statistic for units that have been sorted into groups. (ICC describes how strongly the units in each group correlate or resemble each other—eg, the closer an ICC is to 1, the stronger the correlation is between each unit in the group.) ICC for the correcting equations ranged from 0.45-0.81. The formulas used to calculate corrected calcium levels performed especially poorly in patients with hypoalbuminemia. In this same patient population, the total serum calcium correlated well with directly assessed iCa, with an ICC of 0.85 (95% CI, 0.84-0.86). Moreover, the uncorrected total calcium classified the patient’s calcium level correctly in 82% of cases.

A second study of 5,500 patients in Australia comparing total and adjusted calcium with iCa similarly demonstrated that corrected calcium inaccurately predicts calcium status.10 Findings from this study showed that corrected calcium values correlated with iCa in only 55% to 65% of samples, but uncorrected total calcium correlated with iCa in 70% to 80% of samples. Notably, in patients with renal failure and/or serum albumin concentrations <3 g/dL, formulas used to correct calcium overestimated calcium levels when compared to directly assessed iCa. Correction formulas performed on serum albumin concentrations >3 g/dL correlated better with iCa (65%-77%), effectively negating the utility of the correction formulas.

Another large retrospective observational study from Norway reviewed laboratory data from more than 6,500 hospitalized and clinic patients.11 In this study, researchers calculated corrected calcium using several different albumin-adjusted formulas and compared results to laboratory-assessed iCa. As compared to corrected calcium, uncorrected total calcium more accurately determined clinically relevant free calcium.

Finally, a Canadian research group analyzed time-matched calcium, albumin, and iCa samples from 678 patients.4 They calculated each patient’s corrected calcium values using Payne’s formula. Results of this study showed that corrected calcium predicted iCa outcomes less reliably than uncorrected total calcium (ICC, 0.73 for corrected calcium vs 0.78 for uncorrected calcium).

Utilizing corrected calcium formulas in patients with hypoalbuminemia can overestimate serum calcium, resulting in false-positive findings and an incorrect diagnosis of hypercalcemia or normocalcemia.12 Incorrectly diagnosing hypercalcemia by using correction formulas prompts management that can lead to iatrogenic harm. Hypoalbuminemia is often associated with hepatic or renal disease. In this patient population, standard treatment of hypercalcemia with volume resuscitation (typically 2 to 4 L) and potentially IV loop diuretics will cause clinically significant volume overload and could worsen renal dysfunction.13 Notably, some of the correction formulas utilized in the studies discussed here performed well in hypercalcemic patients, particularly in those with preserved renal function (estimated glomerular filtration rate ≥60 mL/min/1.73 m2).

Importantly, correction formulas can mask true hypocalcemia or the true severity of hypocalcemia. Applying correction formulas in patients with clinically significant hypocalcemia and hypoalbuminemia can make hospitalists believe that the calcium levels are normal or not as clinically significant as they first seemed. This can lead to the withholding of appropriate treatment.12

WHAT YOU SHOULD DO INSTEAD

Based on the available literature, uncorrected total calcium values more accurately assess biologically active calcium. If a more certain calcium value will affect clinical outcomes, clinicians should obtain a direct measurement of iCa.4,9-11 Therefore, clinicians should assess iCa irrespective of the uncorrected serum calcium level in patients who are critically ill or who have known hypoparathyroidism or other derangements in iCa.14 Since iCa levels also fluctuate with pH, samples must be processed quickly and kept cool to slow blood cell metabolism, which alters pH levels.4 Using bedside point-of-care blood gas analyzers to obtain iCa removes a large logistical obstacle to obtaining an accurate iCa. Serum electrolyte interpretation with a properly calibrated point-of-care analyzer correlates well with a traditional laboratory analyzer.15

RECOMMENDATIONS

  • Use serum calcium testing routinely to evaluate calcium homeostasis.
  • Do not use corrected calcium equations to estimate total calcium.
  • If a more accurate measurement of calcium will change medical management, obtain a direct iCa.
  • Obtain a direct iCa measurement in critically ill patents and in patients with known hypoparathyroidism, hyperparathyroidism, or other derangements in calcium homeostasis.
  • Do not order a serum albumin test to assess calcium levels.

CONCLUSION

Returning to our clinical scenario, this patient did not have true hypercalcemia and experienced unnecessary evaluation and treatment. Multiple retrospective clinical trials do not support the practice of using corrected calcium equations to correct for serum albumin derangements.4,9-11 Hospitalists should therefore avoid the temptation to calculate a corrected calcium level in patients with hypoalbuminemia. For patients with clinically significant total serum hypocalcemia or hypercalcemia, they should consider obtaining an iCa assay to better determine the true physiologic impact.

Do you think this is a low-value practice? Is this truly a “Thing We Do for No Reason™”? Share what you do in your practice and join in the conversation online by retweeting it on Twitter (#TWDFNR) and liking it on Facebook. We invite you to propose ideas for other “Things We Do for No Reason™” topics by emailing TWDFNR@hospitalmedicine.org

References

1. Peacock M. Calcium metabolism in health and disease. Clin J Am Soc Nephrol. 2010;5 Suppl 1:S23-S30. https://doi.org/10.2215/cjn.05910809
2. Brown EM. Extracellular Ca2+ sensing, regulation of parathyroid cell function, and role of Ca2+ and other ions as extracellular (first) messengers. Physiol Rev. 1991;71(2):371-411. https://doi.org/10.1152/physrev.1991.71.2.371
3. Aishah AB, Foo YN. A retrospective study of serum calcium levels in a hospital population in Malaysia. Med J Malaysia. 1995;50(3):246-249.
4. Steen O, Clase C, Don-Wauchope A. Corrected calcium formula in routine clinical use does not accurately reflect ionized calcium in hospital patients. Can J Gen Int Med. 2016;11(3):14-21. https://doi.org/10.22374/cjgim.v11i3.150
5. Payne RB, Little AJ, Williams RB, Milner JR. Interpretation of serum calcium in patients with abnormal serum proteins. Br Med J. 1973;4(5893):643-646. https://doi.org/10.1136/bmj.4.5893.643
6. Shane E. Diagnostic approach to hypercalcemia. UpToDate website. Updated August 31, 2020. Accessed April 8, 2021. https://www.uptodate.com/contents/diagnostic-approach-to-hypercalcemia
7. Ladenson JH, Lewis JW, Boyd JC. Failure of total calcium corrected for protein, albumin, and pH to correctly assess free calcium status. J Clin Endocrinol Metab. 1978;46(6):986-993. https://doi.org/10.1210/jcem-46-6-986
8. Besarab A, Caro JF. Increased absolute calcium binding to albumin in hypoalbuminaemia. J Clin Pathol. 1981;34(12):1368-1374. https://doi.org/10.1136/jcp.34.12.1368
9. Ridefelt P, Helmersson-Karlqvist J. Albumin adjustment of total calcium does not improve the estimation of calcium status. Scand J Clin Lab Invest. 2017;77(6):442-447. https://doi.org/10.1080/00365513.2017.1336568
10. Smith JD, Wilson S, Schneider HG. Misclassification of calcium status based on albumin-adjusted calcium: studies in a tertiary hospital setting. Clin Chem. 2018;64(12):1713-1722. https://doi.org/10.1373/clinchem.2018.291377
11. Lian IA, Åsberg A. Should total calcium be adjusted for albumin? A retrospective observational study of laboratory data from central Norway. BMJ Open. 2018;8(4):e017703. https://doi.org/10.1136/bmjopen-2017-017703
12. Bowers GN Jr, Brassard C, Sena SF. Measurement of ionized calcium in serum with ion-selective electrodes: a mature technology that can meet the daily service needs. Clin Chem. 1986;32(8)1437-1447.
13. Myburgh JA. Fluid resuscitation in acute medicine: what is the current situation? J Intern Med. 2015;277(1):58-68. https://doi.org/10.1111/joim.12326
14. Aberegg SK. Ionized calcium in the ICU: should it be measured and corrected? Chest. 2016;149(3):846-855. https://doi.org/10.1016/j.chest.2015.12.001
15. Mirzazadeh M, Morovat A, James T, Smith I, Kirby J, Shine B. Point-of-care testing of electrolytes and calcium using blood gas analysers: it is time we trusted the results. Emerg Med J. 2016;33(3):181-186. https://doi.org/10.1136/emermed-2015-204669

Article PDF
Author and Disclosure Information

1Division of Internal Medicine and Department of Medicine, Tripler Army Medical Center, Honolulu, Hawaii; 2Division of Nephrology, Department of Medicine, Tripler Army Medical Center, Honolulu, Hawaii.

Disclosures
The authors have no conflicts to disclose.

Issue
Journal of Hospital Medicine 16(8)
Publications
Topics
Page Number
499-501. Published Online First June 16, 2021
Sections
Author and Disclosure Information

1Division of Internal Medicine and Department of Medicine, Tripler Army Medical Center, Honolulu, Hawaii; 2Division of Nephrology, Department of Medicine, Tripler Army Medical Center, Honolulu, Hawaii.

Disclosures
The authors have no conflicts to disclose.

Author and Disclosure Information

1Division of Internal Medicine and Department of Medicine, Tripler Army Medical Center, Honolulu, Hawaii; 2Division of Nephrology, Department of Medicine, Tripler Army Medical Center, Honolulu, Hawaii.

Disclosures
The authors have no conflicts to disclose.

Article PDF
Article PDF
Related Articles

Inspired by the ABIM Foundation’s Choosing Wisely® campaign, the “Things We Do for No Reason™” (TWDFNR) series reviews practices that have become common parts of hospital care but may provide little value to our patients. Practices reviewed in the TWDFNR series do not represent clear-cut conclusions or clinical practice standards but are meant as a starting place for research and active discussions among hospitalists and patients. We invite you to be part of that discussion.

CLINICAL SCENARIO

A hospitalist admits a 75-year-old man for evaluation of acute pyelonephritis; the patient’s medical history is significant for chronic kidney disease and nephrotic syndrome. The patient endorses moderate flank pain upon palpation. Initial serum laboratory studies reveal an albumin level of 1.5 g/dL and a calcium level of 10.0 mg/dL. A repeat serum calcium assessment produces similar results. The hospitalist corrects calcium for albumin concentration by applying the most common formula (Payne’s formula), which results in a corrected calcium value of 12 mg/dL. The hospitalist then starts the patient on intravenous (IV) fluids to treat hypercalcemia and obtains serum 25-hydroxyvitamin D and parathyroid hormone levels.

BACKGROUND

Our skeletons bind, with phosphate, nearly 99% of the body’s calcium, the most abundant mineral in our body. The remaining 1% of calcium (approximately 9-10.5 mg/dL) circulates in the blood. Approximately 40% of serum calcium is bound to albumin, with a smaller percentage bound to lactate and citrate. The remaining 4.5 to 5.5 mg/dL circulates unbound as free (ie, ionized) calcium (iCa).1 Calcium has many fundamental intra- and extracellular functions. Physiologic calcium homeostasis is maintained by parathyroid hormone and vitamin D.2 The amount of circulating iCa, rather than total plasma calcium, determines the many biologic effects of plasma calcium.

In the hospital setting, clinicians commonly encounter patients with derangements in calcium homeostasis.3 True hypercalcemia or hypocalcemia has significant clinical manifestations, including generalized fatigue, nephrolithiasis, cardiac arrhythmias, and, potentially, death. Thus, clinical practice requires correct and accurate assessment of serum calcium levels.1

WHY YOU MIGHT THINK CALCULATING A “CORRECTED CALCIUM” LEVEL IS HELPFUL

Although measuring biologically active calcium (ie, iCa) is the gold standard for assessing calcium levels, laboratories struggle to obtain a direct, accurate measurement of iCa due to the special handling and time constraints required to process samples.4 As a result, metabolic laboratory panels typically report the more easily measured total calcium, the sum of iCa and bound calcium.5 Changes in albumin levels, however, do not affect iCa levels. Since calcium has less available albumin for binding, hypoalbuminemia should theoretically decrease the amount of bound calcium and lead to a decreased reported total calcium. Therefore, a patient’s total calcium level may appear low even though their iCa is normal, which can lead to an incorrect diagnosis of hypocalcemia or overestimate of the extent of existing hypocalcemia. Moreover, these lower reported calcium levels can falsely report normocalcemia in patients with hypercalcemia or underestimate the extent of the patient’s hypercalcemia.

For years physicians have attempted to account for the underestimate in total calcium due to hypoalbuminemia by calculating a “corrected” calcium. The correction formulas use total calcium and serum albumin to estimate the expected iCa. Refinements to the original formula, developed by Payne et al in 1973, have resulted in the most commonly utilized formula today: corrected calcium = (0.8 x [normal albumin – patient’s albumin]) + serum calcium.6,7 Many commonly used clinical-decision resources recommend correcting serum calcium concentrations in patients with hypoalbuminemia.6

WHY CALCULATING A CORRECTED CALCIUM FOR ALBUMIN IS UNNECESSARY

While calculating corrected calcium should theoretically provide a more accurate estimate of physiologically active iCa in patients with hypoalbuminemia,4 the commonly used correction equations become less accurate as hypoalbuminemia worsens.8 Payne et al derived the original formula from 200 patients using a single laboratory; however, subsequent retrospective studies have not supported the use of albumin-corrected calcium calculations to estimate the iCa.4,9-11 For example, although Payne’s corrected calcium equations assume a constant relationship between albumin and calcium binding throughout all serum-albumin concentrations, studies have shown that as albumin falls, more calcium ions bind to each available gram of albumin. Payne’s assumption results in an overestimation of the total serum calcium after correction as compared to the iCa.8 In comparison, uncorrected total serum calcium assays more accurately reflect both the change in albumin binding that occurs with alterations in albumin concentration and the unchanged free calcium ions. Studies demonstrate superior correlation between iCa and uncorrected total calcium.4,9-11

Several large retrospective studies revealed the poor in vivo accuracy of equations used to correct calcium for albumin. In one study, Uppsala University Sweden researchers reviewed the laboratory records of more than 20,000 hospitalized patients from 2005 to 2013.9 This group compared seven corrected calcium formulas to direct measurements of iCa. All of the correction equations correlated poorly with iCa based on their intraclass correlation (ICC), a descriptive statistic for units that have been sorted into groups. (ICC describes how strongly the units in each group correlate or resemble each other—eg, the closer an ICC is to 1, the stronger the correlation is between each unit in the group.) ICC for the correcting equations ranged from 0.45-0.81. The formulas used to calculate corrected calcium levels performed especially poorly in patients with hypoalbuminemia. In this same patient population, the total serum calcium correlated well with directly assessed iCa, with an ICC of 0.85 (95% CI, 0.84-0.86). Moreover, the uncorrected total calcium classified the patient’s calcium level correctly in 82% of cases.

A second study of 5,500 patients in Australia comparing total and adjusted calcium with iCa similarly demonstrated that corrected calcium inaccurately predicts calcium status.10 Findings from this study showed that corrected calcium values correlated with iCa in only 55% to 65% of samples, but uncorrected total calcium correlated with iCa in 70% to 80% of samples. Notably, in patients with renal failure and/or serum albumin concentrations <3 g/dL, formulas used to correct calcium overestimated calcium levels when compared to directly assessed iCa. Correction formulas performed on serum albumin concentrations >3 g/dL correlated better with iCa (65%-77%), effectively negating the utility of the correction formulas.

Another large retrospective observational study from Norway reviewed laboratory data from more than 6,500 hospitalized and clinic patients.11 In this study, researchers calculated corrected calcium using several different albumin-adjusted formulas and compared results to laboratory-assessed iCa. As compared to corrected calcium, uncorrected total calcium more accurately determined clinically relevant free calcium.

Finally, a Canadian research group analyzed time-matched calcium, albumin, and iCa samples from 678 patients.4 They calculated each patient’s corrected calcium values using Payne’s formula. Results of this study showed that corrected calcium predicted iCa outcomes less reliably than uncorrected total calcium (ICC, 0.73 for corrected calcium vs 0.78 for uncorrected calcium).

Utilizing corrected calcium formulas in patients with hypoalbuminemia can overestimate serum calcium, resulting in false-positive findings and an incorrect diagnosis of hypercalcemia or normocalcemia.12 Incorrectly diagnosing hypercalcemia by using correction formulas prompts management that can lead to iatrogenic harm. Hypoalbuminemia is often associated with hepatic or renal disease. In this patient population, standard treatment of hypercalcemia with volume resuscitation (typically 2 to 4 L) and potentially IV loop diuretics will cause clinically significant volume overload and could worsen renal dysfunction.13 Notably, some of the correction formulas utilized in the studies discussed here performed well in hypercalcemic patients, particularly in those with preserved renal function (estimated glomerular filtration rate ≥60 mL/min/1.73 m2).

Importantly, correction formulas can mask true hypocalcemia or the true severity of hypocalcemia. Applying correction formulas in patients with clinically significant hypocalcemia and hypoalbuminemia can make hospitalists believe that the calcium levels are normal or not as clinically significant as they first seemed. This can lead to the withholding of appropriate treatment.12

WHAT YOU SHOULD DO INSTEAD

Based on the available literature, uncorrected total calcium values more accurately assess biologically active calcium. If a more certain calcium value will affect clinical outcomes, clinicians should obtain a direct measurement of iCa.4,9-11 Therefore, clinicians should assess iCa irrespective of the uncorrected serum calcium level in patients who are critically ill or who have known hypoparathyroidism or other derangements in iCa.14 Since iCa levels also fluctuate with pH, samples must be processed quickly and kept cool to slow blood cell metabolism, which alters pH levels.4 Using bedside point-of-care blood gas analyzers to obtain iCa removes a large logistical obstacle to obtaining an accurate iCa. Serum electrolyte interpretation with a properly calibrated point-of-care analyzer correlates well with a traditional laboratory analyzer.15

RECOMMENDATIONS

  • Use serum calcium testing routinely to evaluate calcium homeostasis.
  • Do not use corrected calcium equations to estimate total calcium.
  • If a more accurate measurement of calcium will change medical management, obtain a direct iCa.
  • Obtain a direct iCa measurement in critically ill patents and in patients with known hypoparathyroidism, hyperparathyroidism, or other derangements in calcium homeostasis.
  • Do not order a serum albumin test to assess calcium levels.

CONCLUSION

Returning to our clinical scenario, this patient did not have true hypercalcemia and experienced unnecessary evaluation and treatment. Multiple retrospective clinical trials do not support the practice of using corrected calcium equations to correct for serum albumin derangements.4,9-11 Hospitalists should therefore avoid the temptation to calculate a corrected calcium level in patients with hypoalbuminemia. For patients with clinically significant total serum hypocalcemia or hypercalcemia, they should consider obtaining an iCa assay to better determine the true physiologic impact.

Do you think this is a low-value practice? Is this truly a “Thing We Do for No Reason™”? Share what you do in your practice and join in the conversation online by retweeting it on Twitter (#TWDFNR) and liking it on Facebook. We invite you to propose ideas for other “Things We Do for No Reason™” topics by emailing TWDFNR@hospitalmedicine.org

Inspired by the ABIM Foundation’s Choosing Wisely® campaign, the “Things We Do for No Reason™” (TWDFNR) series reviews practices that have become common parts of hospital care but may provide little value to our patients. Practices reviewed in the TWDFNR series do not represent clear-cut conclusions or clinical practice standards but are meant as a starting place for research and active discussions among hospitalists and patients. We invite you to be part of that discussion.

CLINICAL SCENARIO

A hospitalist admits a 75-year-old man for evaluation of acute pyelonephritis; the patient’s medical history is significant for chronic kidney disease and nephrotic syndrome. The patient endorses moderate flank pain upon palpation. Initial serum laboratory studies reveal an albumin level of 1.5 g/dL and a calcium level of 10.0 mg/dL. A repeat serum calcium assessment produces similar results. The hospitalist corrects calcium for albumin concentration by applying the most common formula (Payne’s formula), which results in a corrected calcium value of 12 mg/dL. The hospitalist then starts the patient on intravenous (IV) fluids to treat hypercalcemia and obtains serum 25-hydroxyvitamin D and parathyroid hormone levels.

BACKGROUND

Our skeletons bind, with phosphate, nearly 99% of the body’s calcium, the most abundant mineral in our body. The remaining 1% of calcium (approximately 9-10.5 mg/dL) circulates in the blood. Approximately 40% of serum calcium is bound to albumin, with a smaller percentage bound to lactate and citrate. The remaining 4.5 to 5.5 mg/dL circulates unbound as free (ie, ionized) calcium (iCa).1 Calcium has many fundamental intra- and extracellular functions. Physiologic calcium homeostasis is maintained by parathyroid hormone and vitamin D.2 The amount of circulating iCa, rather than total plasma calcium, determines the many biologic effects of plasma calcium.

In the hospital setting, clinicians commonly encounter patients with derangements in calcium homeostasis.3 True hypercalcemia or hypocalcemia has significant clinical manifestations, including generalized fatigue, nephrolithiasis, cardiac arrhythmias, and, potentially, death. Thus, clinical practice requires correct and accurate assessment of serum calcium levels.1

WHY YOU MIGHT THINK CALCULATING A “CORRECTED CALCIUM” LEVEL IS HELPFUL

Although measuring biologically active calcium (ie, iCa) is the gold standard for assessing calcium levels, laboratories struggle to obtain a direct, accurate measurement of iCa due to the special handling and time constraints required to process samples.4 As a result, metabolic laboratory panels typically report the more easily measured total calcium, the sum of iCa and bound calcium.5 Changes in albumin levels, however, do not affect iCa levels. Since calcium has less available albumin for binding, hypoalbuminemia should theoretically decrease the amount of bound calcium and lead to a decreased reported total calcium. Therefore, a patient’s total calcium level may appear low even though their iCa is normal, which can lead to an incorrect diagnosis of hypocalcemia or overestimate of the extent of existing hypocalcemia. Moreover, these lower reported calcium levels can falsely report normocalcemia in patients with hypercalcemia or underestimate the extent of the patient’s hypercalcemia.

For years physicians have attempted to account for the underestimate in total calcium due to hypoalbuminemia by calculating a “corrected” calcium. The correction formulas use total calcium and serum albumin to estimate the expected iCa. Refinements to the original formula, developed by Payne et al in 1973, have resulted in the most commonly utilized formula today: corrected calcium = (0.8 x [normal albumin – patient’s albumin]) + serum calcium.6,7 Many commonly used clinical-decision resources recommend correcting serum calcium concentrations in patients with hypoalbuminemia.6

WHY CALCULATING A CORRECTED CALCIUM FOR ALBUMIN IS UNNECESSARY

While calculating corrected calcium should theoretically provide a more accurate estimate of physiologically active iCa in patients with hypoalbuminemia,4 the commonly used correction equations become less accurate as hypoalbuminemia worsens.8 Payne et al derived the original formula from 200 patients using a single laboratory; however, subsequent retrospective studies have not supported the use of albumin-corrected calcium calculations to estimate the iCa.4,9-11 For example, although Payne’s corrected calcium equations assume a constant relationship between albumin and calcium binding throughout all serum-albumin concentrations, studies have shown that as albumin falls, more calcium ions bind to each available gram of albumin. Payne’s assumption results in an overestimation of the total serum calcium after correction as compared to the iCa.8 In comparison, uncorrected total serum calcium assays more accurately reflect both the change in albumin binding that occurs with alterations in albumin concentration and the unchanged free calcium ions. Studies demonstrate superior correlation between iCa and uncorrected total calcium.4,9-11

Several large retrospective studies revealed the poor in vivo accuracy of equations used to correct calcium for albumin. In one study, Uppsala University Sweden researchers reviewed the laboratory records of more than 20,000 hospitalized patients from 2005 to 2013.9 This group compared seven corrected calcium formulas to direct measurements of iCa. All of the correction equations correlated poorly with iCa based on their intraclass correlation (ICC), a descriptive statistic for units that have been sorted into groups. (ICC describes how strongly the units in each group correlate or resemble each other—eg, the closer an ICC is to 1, the stronger the correlation is between each unit in the group.) ICC for the correcting equations ranged from 0.45-0.81. The formulas used to calculate corrected calcium levels performed especially poorly in patients with hypoalbuminemia. In this same patient population, the total serum calcium correlated well with directly assessed iCa, with an ICC of 0.85 (95% CI, 0.84-0.86). Moreover, the uncorrected total calcium classified the patient’s calcium level correctly in 82% of cases.

A second study of 5,500 patients in Australia comparing total and adjusted calcium with iCa similarly demonstrated that corrected calcium inaccurately predicts calcium status.10 Findings from this study showed that corrected calcium values correlated with iCa in only 55% to 65% of samples, but uncorrected total calcium correlated with iCa in 70% to 80% of samples. Notably, in patients with renal failure and/or serum albumin concentrations <3 g/dL, formulas used to correct calcium overestimated calcium levels when compared to directly assessed iCa. Correction formulas performed on serum albumin concentrations >3 g/dL correlated better with iCa (65%-77%), effectively negating the utility of the correction formulas.

Another large retrospective observational study from Norway reviewed laboratory data from more than 6,500 hospitalized and clinic patients.11 In this study, researchers calculated corrected calcium using several different albumin-adjusted formulas and compared results to laboratory-assessed iCa. As compared to corrected calcium, uncorrected total calcium more accurately determined clinically relevant free calcium.

Finally, a Canadian research group analyzed time-matched calcium, albumin, and iCa samples from 678 patients.4 They calculated each patient’s corrected calcium values using Payne’s formula. Results of this study showed that corrected calcium predicted iCa outcomes less reliably than uncorrected total calcium (ICC, 0.73 for corrected calcium vs 0.78 for uncorrected calcium).

Utilizing corrected calcium formulas in patients with hypoalbuminemia can overestimate serum calcium, resulting in false-positive findings and an incorrect diagnosis of hypercalcemia or normocalcemia.12 Incorrectly diagnosing hypercalcemia by using correction formulas prompts management that can lead to iatrogenic harm. Hypoalbuminemia is often associated with hepatic or renal disease. In this patient population, standard treatment of hypercalcemia with volume resuscitation (typically 2 to 4 L) and potentially IV loop diuretics will cause clinically significant volume overload and could worsen renal dysfunction.13 Notably, some of the correction formulas utilized in the studies discussed here performed well in hypercalcemic patients, particularly in those with preserved renal function (estimated glomerular filtration rate ≥60 mL/min/1.73 m2).

Importantly, correction formulas can mask true hypocalcemia or the true severity of hypocalcemia. Applying correction formulas in patients with clinically significant hypocalcemia and hypoalbuminemia can make hospitalists believe that the calcium levels are normal or not as clinically significant as they first seemed. This can lead to the withholding of appropriate treatment.12

WHAT YOU SHOULD DO INSTEAD

Based on the available literature, uncorrected total calcium values more accurately assess biologically active calcium. If a more certain calcium value will affect clinical outcomes, clinicians should obtain a direct measurement of iCa.4,9-11 Therefore, clinicians should assess iCa irrespective of the uncorrected serum calcium level in patients who are critically ill or who have known hypoparathyroidism or other derangements in iCa.14 Since iCa levels also fluctuate with pH, samples must be processed quickly and kept cool to slow blood cell metabolism, which alters pH levels.4 Using bedside point-of-care blood gas analyzers to obtain iCa removes a large logistical obstacle to obtaining an accurate iCa. Serum electrolyte interpretation with a properly calibrated point-of-care analyzer correlates well with a traditional laboratory analyzer.15

RECOMMENDATIONS

  • Use serum calcium testing routinely to evaluate calcium homeostasis.
  • Do not use corrected calcium equations to estimate total calcium.
  • If a more accurate measurement of calcium will change medical management, obtain a direct iCa.
  • Obtain a direct iCa measurement in critically ill patents and in patients with known hypoparathyroidism, hyperparathyroidism, or other derangements in calcium homeostasis.
  • Do not order a serum albumin test to assess calcium levels.

CONCLUSION

Returning to our clinical scenario, this patient did not have true hypercalcemia and experienced unnecessary evaluation and treatment. Multiple retrospective clinical trials do not support the practice of using corrected calcium equations to correct for serum albumin derangements.4,9-11 Hospitalists should therefore avoid the temptation to calculate a corrected calcium level in patients with hypoalbuminemia. For patients with clinically significant total serum hypocalcemia or hypercalcemia, they should consider obtaining an iCa assay to better determine the true physiologic impact.

Do you think this is a low-value practice? Is this truly a “Thing We Do for No Reason™”? Share what you do in your practice and join in the conversation online by retweeting it on Twitter (#TWDFNR) and liking it on Facebook. We invite you to propose ideas for other “Things We Do for No Reason™” topics by emailing TWDFNR@hospitalmedicine.org

References

1. Peacock M. Calcium metabolism in health and disease. Clin J Am Soc Nephrol. 2010;5 Suppl 1:S23-S30. https://doi.org/10.2215/cjn.05910809
2. Brown EM. Extracellular Ca2+ sensing, regulation of parathyroid cell function, and role of Ca2+ and other ions as extracellular (first) messengers. Physiol Rev. 1991;71(2):371-411. https://doi.org/10.1152/physrev.1991.71.2.371
3. Aishah AB, Foo YN. A retrospective study of serum calcium levels in a hospital population in Malaysia. Med J Malaysia. 1995;50(3):246-249.
4. Steen O, Clase C, Don-Wauchope A. Corrected calcium formula in routine clinical use does not accurately reflect ionized calcium in hospital patients. Can J Gen Int Med. 2016;11(3):14-21. https://doi.org/10.22374/cjgim.v11i3.150
5. Payne RB, Little AJ, Williams RB, Milner JR. Interpretation of serum calcium in patients with abnormal serum proteins. Br Med J. 1973;4(5893):643-646. https://doi.org/10.1136/bmj.4.5893.643
6. Shane E. Diagnostic approach to hypercalcemia. UpToDate website. Updated August 31, 2020. Accessed April 8, 2021. https://www.uptodate.com/contents/diagnostic-approach-to-hypercalcemia
7. Ladenson JH, Lewis JW, Boyd JC. Failure of total calcium corrected for protein, albumin, and pH to correctly assess free calcium status. J Clin Endocrinol Metab. 1978;46(6):986-993. https://doi.org/10.1210/jcem-46-6-986
8. Besarab A, Caro JF. Increased absolute calcium binding to albumin in hypoalbuminaemia. J Clin Pathol. 1981;34(12):1368-1374. https://doi.org/10.1136/jcp.34.12.1368
9. Ridefelt P, Helmersson-Karlqvist J. Albumin adjustment of total calcium does not improve the estimation of calcium status. Scand J Clin Lab Invest. 2017;77(6):442-447. https://doi.org/10.1080/00365513.2017.1336568
10. Smith JD, Wilson S, Schneider HG. Misclassification of calcium status based on albumin-adjusted calcium: studies in a tertiary hospital setting. Clin Chem. 2018;64(12):1713-1722. https://doi.org/10.1373/clinchem.2018.291377
11. Lian IA, Åsberg A. Should total calcium be adjusted for albumin? A retrospective observational study of laboratory data from central Norway. BMJ Open. 2018;8(4):e017703. https://doi.org/10.1136/bmjopen-2017-017703
12. Bowers GN Jr, Brassard C, Sena SF. Measurement of ionized calcium in serum with ion-selective electrodes: a mature technology that can meet the daily service needs. Clin Chem. 1986;32(8)1437-1447.
13. Myburgh JA. Fluid resuscitation in acute medicine: what is the current situation? J Intern Med. 2015;277(1):58-68. https://doi.org/10.1111/joim.12326
14. Aberegg SK. Ionized calcium in the ICU: should it be measured and corrected? Chest. 2016;149(3):846-855. https://doi.org/10.1016/j.chest.2015.12.001
15. Mirzazadeh M, Morovat A, James T, Smith I, Kirby J, Shine B. Point-of-care testing of electrolytes and calcium using blood gas analysers: it is time we trusted the results. Emerg Med J. 2016;33(3):181-186. https://doi.org/10.1136/emermed-2015-204669

References

1. Peacock M. Calcium metabolism in health and disease. Clin J Am Soc Nephrol. 2010;5 Suppl 1:S23-S30. https://doi.org/10.2215/cjn.05910809
2. Brown EM. Extracellular Ca2+ sensing, regulation of parathyroid cell function, and role of Ca2+ and other ions as extracellular (first) messengers. Physiol Rev. 1991;71(2):371-411. https://doi.org/10.1152/physrev.1991.71.2.371
3. Aishah AB, Foo YN. A retrospective study of serum calcium levels in a hospital population in Malaysia. Med J Malaysia. 1995;50(3):246-249.
4. Steen O, Clase C, Don-Wauchope A. Corrected calcium formula in routine clinical use does not accurately reflect ionized calcium in hospital patients. Can J Gen Int Med. 2016;11(3):14-21. https://doi.org/10.22374/cjgim.v11i3.150
5. Payne RB, Little AJ, Williams RB, Milner JR. Interpretation of serum calcium in patients with abnormal serum proteins. Br Med J. 1973;4(5893):643-646. https://doi.org/10.1136/bmj.4.5893.643
6. Shane E. Diagnostic approach to hypercalcemia. UpToDate website. Updated August 31, 2020. Accessed April 8, 2021. https://www.uptodate.com/contents/diagnostic-approach-to-hypercalcemia
7. Ladenson JH, Lewis JW, Boyd JC. Failure of total calcium corrected for protein, albumin, and pH to correctly assess free calcium status. J Clin Endocrinol Metab. 1978;46(6):986-993. https://doi.org/10.1210/jcem-46-6-986
8. Besarab A, Caro JF. Increased absolute calcium binding to albumin in hypoalbuminaemia. J Clin Pathol. 1981;34(12):1368-1374. https://doi.org/10.1136/jcp.34.12.1368
9. Ridefelt P, Helmersson-Karlqvist J. Albumin adjustment of total calcium does not improve the estimation of calcium status. Scand J Clin Lab Invest. 2017;77(6):442-447. https://doi.org/10.1080/00365513.2017.1336568
10. Smith JD, Wilson S, Schneider HG. Misclassification of calcium status based on albumin-adjusted calcium: studies in a tertiary hospital setting. Clin Chem. 2018;64(12):1713-1722. https://doi.org/10.1373/clinchem.2018.291377
11. Lian IA, Åsberg A. Should total calcium be adjusted for albumin? A retrospective observational study of laboratory data from central Norway. BMJ Open. 2018;8(4):e017703. https://doi.org/10.1136/bmjopen-2017-017703
12. Bowers GN Jr, Brassard C, Sena SF. Measurement of ionized calcium in serum with ion-selective electrodes: a mature technology that can meet the daily service needs. Clin Chem. 1986;32(8)1437-1447.
13. Myburgh JA. Fluid resuscitation in acute medicine: what is the current situation? J Intern Med. 2015;277(1):58-68. https://doi.org/10.1111/joim.12326
14. Aberegg SK. Ionized calcium in the ICU: should it be measured and corrected? Chest. 2016;149(3):846-855. https://doi.org/10.1016/j.chest.2015.12.001
15. Mirzazadeh M, Morovat A, James T, Smith I, Kirby J, Shine B. Point-of-care testing of electrolytes and calcium using blood gas analysers: it is time we trusted the results. Emerg Med J. 2016;33(3):181-186. https://doi.org/10.1136/emermed-2015-204669

Issue
Journal of Hospital Medicine 16(8)
Issue
Journal of Hospital Medicine 16(8)
Page Number
499-501. Published Online First June 16, 2021
Page Number
499-501. Published Online First June 16, 2021
Publications
Publications
Topics
Article Type
Display Headline
Things We Do for No Reason™: Calculating a “Corrected Calcium” Level
Display Headline
Things We Do for No Reason™: Calculating a “Corrected Calcium” Level
Sections
Article Source

© 2021 Society of Hospital Medicine

Disallow All Ads
Correspondence Location
Colin M Kenny, DO; Email: colin.kenny@gmail.com.
Content Gating
Gated (full article locked unless allowed per User)
Alternative CME
Disqus Comments
Default
Use ProPublica
Hide sidebar & use full width
render the right sidebar.
Conference Recap Checkbox
Not Conference Recap
Clinical Edge
Display the Slideshow in this Article
Gating Strategy
First Page Free
Medscape Article
Display survey writer
Reuters content
Disable Inline Native ads
WebMD Article
Article PDF Media
Image
Teambase ID
18001D78.SIG
Disable zoom
Off

Things We Do for No Reason™: NPO After Midnight

Article Type
Changed
Tue, 09/14/2021 - 14:16
Display Headline
Things We Do for No Reason™: NPO After Midnight

Inspired by the ABIM Foundation’s Choosing Wisel y ® campaign, the “Things We Do for No Reason ” (TWDFNR) series reviews practices that have become common parts of hospital care but may provide little value to our patients. Practices reviewed in the TWDFNR series do not represent clear-cut conclusions or clinical practice standards but are meant as a starting place for research and active discussions among hospitalists and patients. We invite you to be part of that discussion.

CLINICAL SCENARIO

The hospitalist admits an 18-year-old man for newly diagnosed granulomatosis with polyangiitis to receive expedited pulse-dose steroids and plasma exchange. After consulting interventional radiology for catheter placement the following day, the hospitalist places a “strict” nil per os (nothing by mouth, NPO) after midnight order. During rounds the following morning, the patient reports that he wants to eat. At 9 am, interventional radiology informs the nurse that the line placement will take place at 3 pm. Due to emergencies and other unplanned delays, the catheter placement occurs at 5 pm. The patient and family express their displeasure about the prolonged fasting and ask why this happened.

BACKGROUND

Hospitalists commonly order “NPO after midnight” diets in anticipation of procedures requiring sedation or general anesthesia. Typically, NPO refers to no food or drink, but in some instances, NPO includes no oral medications. Up to half of medical patients experience some time of fasting while hospitalized.1 However, NPO practices vary widely across institutions.2,3 A study from 2014 notes that, on average, patients fast preprocedure for approximately 13.5 hours for solids and 9.6 hours for liquids.2 Prolonged fasting times offer little benefit to patients and may lead to frequent patient dissatisfaction and complaints.

WHY YOU MIGHT THINK THAT MAKING PATIENTS NPO AFTER MIDNIGHT IS APPROPRIATE

In 1883, Sir Joseph Lister described 19th century NPO practices distinguishing solids from liquids, allowing patients “tea or beef tea” until 2 to 3 hours prior to surgery.4 However, in 1946, Mendelson published an influential account of 66 pregnant women who aspirated during delivery under general anesthesia.5 Two of the 66 patients, both of whom had eaten a full meal 6 to 8 hours prior to general anesthesia, died. The study not only increased awareness of the risk of aspiration with general anesthesia in pregnancy, but it influenced the care for the nonpregnant population of patients as well. By the 1960s, anesthesia texts recommended “NPO after midnight” for both liquids and solids in all patients, regardless of pregnancy status.4 To minimize the risk to patients, we have continued to pass down the practice of NPO after midnight to subsequent generations.

Additionally, medical centers and hospitals feel pressure to provide efficient, patient-centered, high-value care. Given the complexity of procedural scheduling and the penalties associated with delays, keeping patients NPO ensures their availability for the next open procedural slot. NPO after midnight orders aim to prevent potential delays in treatment that occur when inadvertent ingestion of food and drink leads to cancellation of procedures.

WHY THE INDISCRIMINATE USE OF NPO AFTER MIDNIGHT IS UNNECESSARY

Recent studies have led to a more sophisticated understanding of gastric emptying and the risks of aspiration during sedation and intubation. Gastric emptying studies routinely show that transit of clear liquids out of the stomach is virtually complete within two hours of drinking.6 Age, body mass index, and alcohol have no effect on gastric emptying time, and almost all patients return to preingestion gastric residual volumes within 2 hours of clear liquid consumption.6,7 While morbidly obese patients tend to have higher gastric fluid volumes after 9 hours of fasting, their stomachs empty at rates similar to nonobese individuals.6 Note that, regardless of fasting times, morbid obesity predisposes patients to a higher overall gastric volume and lower pH of gastric contents, which may increase risk of aspiration.8 A Cochrane review found no statistical difference in gastric volumes or stomach pH in patients on a standard fast vs shortened (<180 minutes) liquid fast.9 The review included nine studies that found patients who consumed a clear liquid beverage had reduced gastric volumes, compared with patients in a fasting state (P < .001).9

In a pediatric retrospective study of pulmonary aspiration events, the researchers demonstrated that clinically significant aspiration (presence of bilious secretions in the tracheobronchial airways) occurred at a rate of 0.04% with emergency surgery.10 Bowel obstruction or ileus accounted for approximately 54% of those cases. Importantly, the reported aspiration rate approximates the rate of pregnant patients from the 1946 Mendelson study of 0.14% (66 out of 44,016), which originally prompted the use of the prolonged NPO status. Based on the Cochrane review of perioperative fasting recommendations for those older than 18 years, consuming fluids more than 90 minutes preoperatively confers a negligible (0 adverse events reported in 9 studies) risk for aspiration or regurgitation events.9

In 1998, as a result of these and other similar studies, the American Society of Anesthesiologists (ASA) along with global anesthesia partners adopted guidelines that allowed clear liquids up until 2 hours prior to anesthesia or sedation in low-aspiration-risk patients undergoing elective cases.11 The guidelines allowed for other beverages and food based on their standard transit times (Table). The ASA guidelines do not define low-aspiration-risk patients. Anesthesiologists generally exclude from the low-risk category patients who may have delayed gastric emptying from medical or iatrogenic causes. The updated 2017 ASA guidelines remain unchanged regarding fasting guidelines.12 Studies suggest that approximately 10% to 20% of NPO after midnight orders are avoidable.1,3 For those instances, procedures are often deemed not necessary or do not require NPO status.1

black0238_0521e_t1.png

In a study evaluating the reasons that necessary procedures are canceled, only 0.5% of inpatient procedures are cancelled due to the inappropriate ingestion of food or drink.3 In addition, NPO status creates risk. Patients with prolonged NPO status report greater hunger, thirst, tiredness, and weakness prior to surgery when compared with patients receiving a carbohydrate-rich drink 2 hours prior to procedures.9,13,14 In fact, multiple studies have suggested that preoperative carbohydrate-rich drinks 2 hours before surgery can be associated with decreased insulin resistance in the perioperative period, decreased length of stay, and improvement in perioperative metabolic, cardiac, and psychosomatic status.9,13-15 These types of studies have informed the enhanced recovery after surgery program, which recommends a carbohydrate beverage 2 to 3 hours prior to surgery.

WHEN TO ORDER LONGER PREPROCEDURAL NPO TIMES

Prescribe the minimum recommended fasting times only for low-aspiration-risk patients undergoing elective procedures. Risk for regurgitation or aspiration increases for patients with conditions resulting in decreased gastric emptying, gastric or bowel obstruction, or lower esophageal sphincter incompetence. Those patients may require longer NPO time periods.8 Higher-risk diagnoses and clinical conditions include gastroparesis, trauma, and pregnancy.5,8,16 Specific risk factors for aspiration in children may include trauma, bowel obstruction, depressed consciousness, shock, or ileus.10 For surgical emergencies, balance the risk of surgical delay vs perceived aspiration risk.

WHAT WE SHOULD DO INSTEAD OF ROUTINELY ORDERING NPO AFTER MIDNIGHT

Use evidence-based guidelines to assess periprocedural aspiration risk. The ASA guidelines suggest that healthy, nonpregnant patients should fast for 8 hours after heavy meals, 6 hours after a light, nonfatty meal, and 2 hours after clear liquids (eg, water, fruit juices without pulp, carbonated beverages, black coffee).12 Focus on the type of food or drink rather than the volume ingested.12 Additionally, patients should ingest, with small amounts of clear fluids, appropriate home medications for acute and chronic conditions regardless of NPO status.

While procedure delays or cancellations for any reason upset patients and families and can disrupt the flow of the operating room and procedural suite, we can achieve the delicate balance between efficiency and patient safety and comfort. Since complex inpatient procedural scheduling may not allow for liberalization of solids requiring 6 to 8 hours of fasting time, focus on liberalizing liquids 2 hours prior to anesthesia. This allows staff to minimize the time low-risk patients fast while still maintaining flexibility for operating room case scheduling. We must promote communication between operating room and floor staff to anticipate timing of procedures each day. Healthcare facilities should aim to achieve time-based preprocedural NPO status as opposed to an arbitrary starting time like midnight.4

RECOMMENDATIONS

  • Risk stratify patients for anesthesia-related aspiration with the aim of identifying those at low aspiration risk.
  • For low-risk patients, adhere to recommended fasting times: 2 hours for a clear carbohydrate beverage, 4 hours for breast milk, 6 hours for a light meal or formula, and 8 hours for a fatty meal.
  • For patients not deemed low risk, determine the appropriate length of preprocedural fasting by consulting with the anesthesia and surgical teams.

CONCLUSION

NPO after midnight represents a low-value and arbitrary practice that leaves patients fasting longer than necessary.2,3,12 In addition to the 2017 ASA guidelines, newer studies and protocols are improving patient satisfaction, minimizing patient dehydration and electrolyte disturbances, and incorporating enhanced recovery after surgery factors into a better patient experience. Returning to the clinical scenario, the hospitalist team can increase patient satisfaction by focusing on liberalizing clear fluids with a carbohydrate beverage up to 2 hours prior to elective surgery while still allowing for schedule flexibility. For this patient, a 3 pm procedure time would have allowed him to have a light breakfast and carbohydrate beverages until 2 hours prior to anesthesia. Dispose of the antiquated practice of NPO after midnight by maximizing clear fluid intake in accordance with current guidelines prior to sedation and general anesthesia. This change in practice will help to achieve normophysiology and increase patient satisfaction.

Do you think this is a low-value practice? Is this truly a “Thing We Do for No Reason™”? Share what you do in your practice and join in the conversation online by retweeting it on Twitter (#TWDFNR) and liking it on Facebook. We invite you to propose ideas for other “Things We Do for No Reason™” topics by emailing TWDFNR@hospitalmedicine.org.

Disclaimer: The opinions expressed in this article are those of the authors alone and do not reflect the views of the Department of Veterans Affairs. The Veterans Affairs Quality Scholars Program is supported by the Veterans Affairs Office of Academic Affiliations, Washington, DC.

References

1. Sorita A, Thongprayoon C, Ahmed A, et al. Frequency and appropriateness of fasting orders in the hospital. Mayo Clin Proc. 2015;90(9):1225-1232. https://doi.org/10.1016/j.mayocp.2015.07.013
2. Falconer R, Skouras C, Carter T, Greenway L, Paisley AM. Preoperative fasting: current practice and areas for improvement. Updates Surg. 2014;66(1):31-39. https://doi.org/10.1007/s13304-013-0242-z
3. Sorita A, Thongprayoon C, Ratelle JT, et al. Characteristics and outcomes of fasting orders among medical inpatients. J Hosp Med. 2017;12(1):36-39. https://doi.org/10.1002/jhm.2674
4. Maltby JR. Fasting from midnight–the history behind the dogma. Best Pract Res Clin Anaesthesiol. 2006;20(3):363-378. https://doi.org/10.1016/j.bpa.2006.02.001
5. Mendelson CL. The aspiration of stomach contents into the lungs during obstetric anesthesia. Am J Obstet Gynecol. 1946;52:191-205. https://doi.org/10.1016/s0002-9378(16)39829-5
6. Shiraishi T, Kurosaki D, Nakamura M, et al. Gastric fluid volume change after oral rehydration solution intake in morbidly obese and normal controls: a magnetic resonance imaging-based analysis. Anesth Analg. 2017;124(4):1174-1178. https://doi.org/10.1213/ane.0000000000001886
7. Vasavid P, Chaiwatanarat T, Pusuwan P, et al. Normal solid gastric emptying values measured by scintigraphy using Asian-style meal: a multicenter study in healthy volunteers. J Neurogastroenterol Motil. 2014;20(3):371-378. https://doi.org/10.5056/jnm13114
8. Mahajan V, Hashmi J, Singh R, Samra T, Aneja S. Comparative evaluation of gastric pH and volume in morbidly obese and lean patients undergoing elective surgery and effect of aspiration prophylaxis. J Clin Anesth. 2015;27(5):396-400. https://doi.org/10.1016/j.jclinane.2015.03.004
9. Brady MC, Kinn S, Stuart P, Ness V. Preoperative fasting for adults to prevent perioperative complications. Cochrane Database Syst Rev. 2003;(4):CD004423. https://doi.org/10.1002/14651858.cd004423
10. Warner MA, Warner ME, Warner DO, Warner LO, Warner EJ. Perioperative pulmonary aspiration in infants and children. Anesthesiology. 1999;90(1):66-71. https://doi.org/10.1097/00000542-199901000-00011
11. Practice guidelines for preoperative fasting and the use of pharmacologic agents to reduce the risk of pulmonary aspiration: application to healthy patients undergoing elective procedures: a report by the American Society of Anesthesiologist Task Force on Preoperative Fasting. Anesthesiology. 1999;90(3):896-905. https://doi.org/10.1097/00000542-199903000-00034
12. Practice guidelines for preoperative fasting and the use of pharmacologic agents to reduce the risk of pulmonary aspiration: application to healthy patients undergoing elective procedures: an updated report by the American Society of Anesthesiologists task force on preoperative fasting and the use of pharmacologic agents to reduce the risk of pulmonary aspiration. Anesthesiology. 2017;126(3):376-393. https://doi.org/10.1097/aln.0000000000001452
13. Hausel J, Nygren J, Lagerkranser M, et al. A carbohydrate-rich drink reduces preoperative discomfort in elective surgery patients. Anesth Analg. 2001;93(5):1344-1350. https://doi.org/10.1097/00000539-200111000-00063
14. Awad S, Varadhan KK, Ljungqvist O, Lobo DN. A meta-analysis of randomised controlled trials on preoperative oral carbohydrate treatment in elective surgery. Clin Nutr. 2013;32(1):34-44. https://doi.org/10.1016/j.clnu.2012.10.011
15. Kaška M, Grosmanová T, Havel E, et al. The impact and safety of preoperative oral or intravenous carbohydrate administration versus fasting in colorectal surgery–a randomized controlled trial. Wien Klin Wochenschr. 2010;122(1-2):23-30. https://doi.org/10.1007/s00508-009-1291-7
16. Tokumine J, Sugahara K, Fuchigami T, Teruya K, Nitta K, Satou K. Unanticipated full stomach at anesthesia induction in a type I diabetic patient with asymptomatic gastroparesis. J Anesth. 2005;19(3):247-248. https://doi.org/10.1007/s00540-005-0321-5

Article PDF
Author and Disclosure Information

1Division of General Internal Medicine, Department of Medicine, University of Alabama at Birmingham, Birmingham, Alabama; 2Section of General Internal Medicine, Birmingham VA Medical Center, Birmingham, Alabama; 3Departments of Anesthesiology and Pediatrics, University of North Carolina, Chapel Hill, North Carolina; 4Department of Pediatrics, North Carolina Children’s Hospital, UNC Health Care, Chapel Hill, North Carolina; 5Division of General and Acute Care Surgery, Department of Surgery, University of North Carolina, Chapel Hill, North Carolina; 6Department of Internal Medicine, UNC Health Care, Chapel Hill, North Carolina; 7Department of Medicine, University of Mississippi Medical Center, Jackson, Mississippi; 8Division of Hospital Medicine, St. Dominic’s Hospital, Jackson, Mississippi.

Disclosures
The authors have nothing to disclose.

Issue
Journal of Hospital Medicine 16(6)
Publications
Topics
Page Number
368-370. Published Online First May 19, 2021
Sections
Author and Disclosure Information

1Division of General Internal Medicine, Department of Medicine, University of Alabama at Birmingham, Birmingham, Alabama; 2Section of General Internal Medicine, Birmingham VA Medical Center, Birmingham, Alabama; 3Departments of Anesthesiology and Pediatrics, University of North Carolina, Chapel Hill, North Carolina; 4Department of Pediatrics, North Carolina Children’s Hospital, UNC Health Care, Chapel Hill, North Carolina; 5Division of General and Acute Care Surgery, Department of Surgery, University of North Carolina, Chapel Hill, North Carolina; 6Department of Internal Medicine, UNC Health Care, Chapel Hill, North Carolina; 7Department of Medicine, University of Mississippi Medical Center, Jackson, Mississippi; 8Division of Hospital Medicine, St. Dominic’s Hospital, Jackson, Mississippi.

Disclosures
The authors have nothing to disclose.

Author and Disclosure Information

1Division of General Internal Medicine, Department of Medicine, University of Alabama at Birmingham, Birmingham, Alabama; 2Section of General Internal Medicine, Birmingham VA Medical Center, Birmingham, Alabama; 3Departments of Anesthesiology and Pediatrics, University of North Carolina, Chapel Hill, North Carolina; 4Department of Pediatrics, North Carolina Children’s Hospital, UNC Health Care, Chapel Hill, North Carolina; 5Division of General and Acute Care Surgery, Department of Surgery, University of North Carolina, Chapel Hill, North Carolina; 6Department of Internal Medicine, UNC Health Care, Chapel Hill, North Carolina; 7Department of Medicine, University of Mississippi Medical Center, Jackson, Mississippi; 8Division of Hospital Medicine, St. Dominic’s Hospital, Jackson, Mississippi.

Disclosures
The authors have nothing to disclose.

Article PDF
Article PDF
Related Articles

Inspired by the ABIM Foundation’s Choosing Wisel y ® campaign, the “Things We Do for No Reason ” (TWDFNR) series reviews practices that have become common parts of hospital care but may provide little value to our patients. Practices reviewed in the TWDFNR series do not represent clear-cut conclusions or clinical practice standards but are meant as a starting place for research and active discussions among hospitalists and patients. We invite you to be part of that discussion.

CLINICAL SCENARIO

The hospitalist admits an 18-year-old man for newly diagnosed granulomatosis with polyangiitis to receive expedited pulse-dose steroids and plasma exchange. After consulting interventional radiology for catheter placement the following day, the hospitalist places a “strict” nil per os (nothing by mouth, NPO) after midnight order. During rounds the following morning, the patient reports that he wants to eat. At 9 am, interventional radiology informs the nurse that the line placement will take place at 3 pm. Due to emergencies and other unplanned delays, the catheter placement occurs at 5 pm. The patient and family express their displeasure about the prolonged fasting and ask why this happened.

BACKGROUND

Hospitalists commonly order “NPO after midnight” diets in anticipation of procedures requiring sedation or general anesthesia. Typically, NPO refers to no food or drink, but in some instances, NPO includes no oral medications. Up to half of medical patients experience some time of fasting while hospitalized.1 However, NPO practices vary widely across institutions.2,3 A study from 2014 notes that, on average, patients fast preprocedure for approximately 13.5 hours for solids and 9.6 hours for liquids.2 Prolonged fasting times offer little benefit to patients and may lead to frequent patient dissatisfaction and complaints.

WHY YOU MIGHT THINK THAT MAKING PATIENTS NPO AFTER MIDNIGHT IS APPROPRIATE

In 1883, Sir Joseph Lister described 19th century NPO practices distinguishing solids from liquids, allowing patients “tea or beef tea” until 2 to 3 hours prior to surgery.4 However, in 1946, Mendelson published an influential account of 66 pregnant women who aspirated during delivery under general anesthesia.5 Two of the 66 patients, both of whom had eaten a full meal 6 to 8 hours prior to general anesthesia, died. The study not only increased awareness of the risk of aspiration with general anesthesia in pregnancy, but it influenced the care for the nonpregnant population of patients as well. By the 1960s, anesthesia texts recommended “NPO after midnight” for both liquids and solids in all patients, regardless of pregnancy status.4 To minimize the risk to patients, we have continued to pass down the practice of NPO after midnight to subsequent generations.

Additionally, medical centers and hospitals feel pressure to provide efficient, patient-centered, high-value care. Given the complexity of procedural scheduling and the penalties associated with delays, keeping patients NPO ensures their availability for the next open procedural slot. NPO after midnight orders aim to prevent potential delays in treatment that occur when inadvertent ingestion of food and drink leads to cancellation of procedures.

WHY THE INDISCRIMINATE USE OF NPO AFTER MIDNIGHT IS UNNECESSARY

Recent studies have led to a more sophisticated understanding of gastric emptying and the risks of aspiration during sedation and intubation. Gastric emptying studies routinely show that transit of clear liquids out of the stomach is virtually complete within two hours of drinking.6 Age, body mass index, and alcohol have no effect on gastric emptying time, and almost all patients return to preingestion gastric residual volumes within 2 hours of clear liquid consumption.6,7 While morbidly obese patients tend to have higher gastric fluid volumes after 9 hours of fasting, their stomachs empty at rates similar to nonobese individuals.6 Note that, regardless of fasting times, morbid obesity predisposes patients to a higher overall gastric volume and lower pH of gastric contents, which may increase risk of aspiration.8 A Cochrane review found no statistical difference in gastric volumes or stomach pH in patients on a standard fast vs shortened (<180 minutes) liquid fast.9 The review included nine studies that found patients who consumed a clear liquid beverage had reduced gastric volumes, compared with patients in a fasting state (P < .001).9

In a pediatric retrospective study of pulmonary aspiration events, the researchers demonstrated that clinically significant aspiration (presence of bilious secretions in the tracheobronchial airways) occurred at a rate of 0.04% with emergency surgery.10 Bowel obstruction or ileus accounted for approximately 54% of those cases. Importantly, the reported aspiration rate approximates the rate of pregnant patients from the 1946 Mendelson study of 0.14% (66 out of 44,016), which originally prompted the use of the prolonged NPO status. Based on the Cochrane review of perioperative fasting recommendations for those older than 18 years, consuming fluids more than 90 minutes preoperatively confers a negligible (0 adverse events reported in 9 studies) risk for aspiration or regurgitation events.9

In 1998, as a result of these and other similar studies, the American Society of Anesthesiologists (ASA) along with global anesthesia partners adopted guidelines that allowed clear liquids up until 2 hours prior to anesthesia or sedation in low-aspiration-risk patients undergoing elective cases.11 The guidelines allowed for other beverages and food based on their standard transit times (Table). The ASA guidelines do not define low-aspiration-risk patients. Anesthesiologists generally exclude from the low-risk category patients who may have delayed gastric emptying from medical or iatrogenic causes. The updated 2017 ASA guidelines remain unchanged regarding fasting guidelines.12 Studies suggest that approximately 10% to 20% of NPO after midnight orders are avoidable.1,3 For those instances, procedures are often deemed not necessary or do not require NPO status.1

black0238_0521e_t1.png

In a study evaluating the reasons that necessary procedures are canceled, only 0.5% of inpatient procedures are cancelled due to the inappropriate ingestion of food or drink.3 In addition, NPO status creates risk. Patients with prolonged NPO status report greater hunger, thirst, tiredness, and weakness prior to surgery when compared with patients receiving a carbohydrate-rich drink 2 hours prior to procedures.9,13,14 In fact, multiple studies have suggested that preoperative carbohydrate-rich drinks 2 hours before surgery can be associated with decreased insulin resistance in the perioperative period, decreased length of stay, and improvement in perioperative metabolic, cardiac, and psychosomatic status.9,13-15 These types of studies have informed the enhanced recovery after surgery program, which recommends a carbohydrate beverage 2 to 3 hours prior to surgery.

WHEN TO ORDER LONGER PREPROCEDURAL NPO TIMES

Prescribe the minimum recommended fasting times only for low-aspiration-risk patients undergoing elective procedures. Risk for regurgitation or aspiration increases for patients with conditions resulting in decreased gastric emptying, gastric or bowel obstruction, or lower esophageal sphincter incompetence. Those patients may require longer NPO time periods.8 Higher-risk diagnoses and clinical conditions include gastroparesis, trauma, and pregnancy.5,8,16 Specific risk factors for aspiration in children may include trauma, bowel obstruction, depressed consciousness, shock, or ileus.10 For surgical emergencies, balance the risk of surgical delay vs perceived aspiration risk.

WHAT WE SHOULD DO INSTEAD OF ROUTINELY ORDERING NPO AFTER MIDNIGHT

Use evidence-based guidelines to assess periprocedural aspiration risk. The ASA guidelines suggest that healthy, nonpregnant patients should fast for 8 hours after heavy meals, 6 hours after a light, nonfatty meal, and 2 hours after clear liquids (eg, water, fruit juices without pulp, carbonated beverages, black coffee).12 Focus on the type of food or drink rather than the volume ingested.12 Additionally, patients should ingest, with small amounts of clear fluids, appropriate home medications for acute and chronic conditions regardless of NPO status.

While procedure delays or cancellations for any reason upset patients and families and can disrupt the flow of the operating room and procedural suite, we can achieve the delicate balance between efficiency and patient safety and comfort. Since complex inpatient procedural scheduling may not allow for liberalization of solids requiring 6 to 8 hours of fasting time, focus on liberalizing liquids 2 hours prior to anesthesia. This allows staff to minimize the time low-risk patients fast while still maintaining flexibility for operating room case scheduling. We must promote communication between operating room and floor staff to anticipate timing of procedures each day. Healthcare facilities should aim to achieve time-based preprocedural NPO status as opposed to an arbitrary starting time like midnight.4

RECOMMENDATIONS

  • Risk stratify patients for anesthesia-related aspiration with the aim of identifying those at low aspiration risk.
  • For low-risk patients, adhere to recommended fasting times: 2 hours for a clear carbohydrate beverage, 4 hours for breast milk, 6 hours for a light meal or formula, and 8 hours for a fatty meal.
  • For patients not deemed low risk, determine the appropriate length of preprocedural fasting by consulting with the anesthesia and surgical teams.

CONCLUSION

NPO after midnight represents a low-value and arbitrary practice that leaves patients fasting longer than necessary.2,3,12 In addition to the 2017 ASA guidelines, newer studies and protocols are improving patient satisfaction, minimizing patient dehydration and electrolyte disturbances, and incorporating enhanced recovery after surgery factors into a better patient experience. Returning to the clinical scenario, the hospitalist team can increase patient satisfaction by focusing on liberalizing clear fluids with a carbohydrate beverage up to 2 hours prior to elective surgery while still allowing for schedule flexibility. For this patient, a 3 pm procedure time would have allowed him to have a light breakfast and carbohydrate beverages until 2 hours prior to anesthesia. Dispose of the antiquated practice of NPO after midnight by maximizing clear fluid intake in accordance with current guidelines prior to sedation and general anesthesia. This change in practice will help to achieve normophysiology and increase patient satisfaction.

Do you think this is a low-value practice? Is this truly a “Thing We Do for No Reason™”? Share what you do in your practice and join in the conversation online by retweeting it on Twitter (#TWDFNR) and liking it on Facebook. We invite you to propose ideas for other “Things We Do for No Reason™” topics by emailing TWDFNR@hospitalmedicine.org.

Disclaimer: The opinions expressed in this article are those of the authors alone and do not reflect the views of the Department of Veterans Affairs. The Veterans Affairs Quality Scholars Program is supported by the Veterans Affairs Office of Academic Affiliations, Washington, DC.

Inspired by the ABIM Foundation’s Choosing Wisel y ® campaign, the “Things We Do for No Reason ” (TWDFNR) series reviews practices that have become common parts of hospital care but may provide little value to our patients. Practices reviewed in the TWDFNR series do not represent clear-cut conclusions or clinical practice standards but are meant as a starting place for research and active discussions among hospitalists and patients. We invite you to be part of that discussion.

CLINICAL SCENARIO

The hospitalist admits an 18-year-old man for newly diagnosed granulomatosis with polyangiitis to receive expedited pulse-dose steroids and plasma exchange. After consulting interventional radiology for catheter placement the following day, the hospitalist places a “strict” nil per os (nothing by mouth, NPO) after midnight order. During rounds the following morning, the patient reports that he wants to eat. At 9 am, interventional radiology informs the nurse that the line placement will take place at 3 pm. Due to emergencies and other unplanned delays, the catheter placement occurs at 5 pm. The patient and family express their displeasure about the prolonged fasting and ask why this happened.

BACKGROUND

Hospitalists commonly order “NPO after midnight” diets in anticipation of procedures requiring sedation or general anesthesia. Typically, NPO refers to no food or drink, but in some instances, NPO includes no oral medications. Up to half of medical patients experience some time of fasting while hospitalized.1 However, NPO practices vary widely across institutions.2,3 A study from 2014 notes that, on average, patients fast preprocedure for approximately 13.5 hours for solids and 9.6 hours for liquids.2 Prolonged fasting times offer little benefit to patients and may lead to frequent patient dissatisfaction and complaints.

WHY YOU MIGHT THINK THAT MAKING PATIENTS NPO AFTER MIDNIGHT IS APPROPRIATE

In 1883, Sir Joseph Lister described 19th century NPO practices distinguishing solids from liquids, allowing patients “tea or beef tea” until 2 to 3 hours prior to surgery.4 However, in 1946, Mendelson published an influential account of 66 pregnant women who aspirated during delivery under general anesthesia.5 Two of the 66 patients, both of whom had eaten a full meal 6 to 8 hours prior to general anesthesia, died. The study not only increased awareness of the risk of aspiration with general anesthesia in pregnancy, but it influenced the care for the nonpregnant population of patients as well. By the 1960s, anesthesia texts recommended “NPO after midnight” for both liquids and solids in all patients, regardless of pregnancy status.4 To minimize the risk to patients, we have continued to pass down the practice of NPO after midnight to subsequent generations.

Additionally, medical centers and hospitals feel pressure to provide efficient, patient-centered, high-value care. Given the complexity of procedural scheduling and the penalties associated with delays, keeping patients NPO ensures their availability for the next open procedural slot. NPO after midnight orders aim to prevent potential delays in treatment that occur when inadvertent ingestion of food and drink leads to cancellation of procedures.

WHY THE INDISCRIMINATE USE OF NPO AFTER MIDNIGHT IS UNNECESSARY

Recent studies have led to a more sophisticated understanding of gastric emptying and the risks of aspiration during sedation and intubation. Gastric emptying studies routinely show that transit of clear liquids out of the stomach is virtually complete within two hours of drinking.6 Age, body mass index, and alcohol have no effect on gastric emptying time, and almost all patients return to preingestion gastric residual volumes within 2 hours of clear liquid consumption.6,7 While morbidly obese patients tend to have higher gastric fluid volumes after 9 hours of fasting, their stomachs empty at rates similar to nonobese individuals.6 Note that, regardless of fasting times, morbid obesity predisposes patients to a higher overall gastric volume and lower pH of gastric contents, which may increase risk of aspiration.8 A Cochrane review found no statistical difference in gastric volumes or stomach pH in patients on a standard fast vs shortened (<180 minutes) liquid fast.9 The review included nine studies that found patients who consumed a clear liquid beverage had reduced gastric volumes, compared with patients in a fasting state (P < .001).9

In a pediatric retrospective study of pulmonary aspiration events, the researchers demonstrated that clinically significant aspiration (presence of bilious secretions in the tracheobronchial airways) occurred at a rate of 0.04% with emergency surgery.10 Bowel obstruction or ileus accounted for approximately 54% of those cases. Importantly, the reported aspiration rate approximates the rate of pregnant patients from the 1946 Mendelson study of 0.14% (66 out of 44,016), which originally prompted the use of the prolonged NPO status. Based on the Cochrane review of perioperative fasting recommendations for those older than 18 years, consuming fluids more than 90 minutes preoperatively confers a negligible (0 adverse events reported in 9 studies) risk for aspiration or regurgitation events.9

In 1998, as a result of these and other similar studies, the American Society of Anesthesiologists (ASA) along with global anesthesia partners adopted guidelines that allowed clear liquids up until 2 hours prior to anesthesia or sedation in low-aspiration-risk patients undergoing elective cases.11 The guidelines allowed for other beverages and food based on their standard transit times (Table). The ASA guidelines do not define low-aspiration-risk patients. Anesthesiologists generally exclude from the low-risk category patients who may have delayed gastric emptying from medical or iatrogenic causes. The updated 2017 ASA guidelines remain unchanged regarding fasting guidelines.12 Studies suggest that approximately 10% to 20% of NPO after midnight orders are avoidable.1,3 For those instances, procedures are often deemed not necessary or do not require NPO status.1

black0238_0521e_t1.png

In a study evaluating the reasons that necessary procedures are canceled, only 0.5% of inpatient procedures are cancelled due to the inappropriate ingestion of food or drink.3 In addition, NPO status creates risk. Patients with prolonged NPO status report greater hunger, thirst, tiredness, and weakness prior to surgery when compared with patients receiving a carbohydrate-rich drink 2 hours prior to procedures.9,13,14 In fact, multiple studies have suggested that preoperative carbohydrate-rich drinks 2 hours before surgery can be associated with decreased insulin resistance in the perioperative period, decreased length of stay, and improvement in perioperative metabolic, cardiac, and psychosomatic status.9,13-15 These types of studies have informed the enhanced recovery after surgery program, which recommends a carbohydrate beverage 2 to 3 hours prior to surgery.

WHEN TO ORDER LONGER PREPROCEDURAL NPO TIMES

Prescribe the minimum recommended fasting times only for low-aspiration-risk patients undergoing elective procedures. Risk for regurgitation or aspiration increases for patients with conditions resulting in decreased gastric emptying, gastric or bowel obstruction, or lower esophageal sphincter incompetence. Those patients may require longer NPO time periods.8 Higher-risk diagnoses and clinical conditions include gastroparesis, trauma, and pregnancy.5,8,16 Specific risk factors for aspiration in children may include trauma, bowel obstruction, depressed consciousness, shock, or ileus.10 For surgical emergencies, balance the risk of surgical delay vs perceived aspiration risk.

WHAT WE SHOULD DO INSTEAD OF ROUTINELY ORDERING NPO AFTER MIDNIGHT

Use evidence-based guidelines to assess periprocedural aspiration risk. The ASA guidelines suggest that healthy, nonpregnant patients should fast for 8 hours after heavy meals, 6 hours after a light, nonfatty meal, and 2 hours after clear liquids (eg, water, fruit juices without pulp, carbonated beverages, black coffee).12 Focus on the type of food or drink rather than the volume ingested.12 Additionally, patients should ingest, with small amounts of clear fluids, appropriate home medications for acute and chronic conditions regardless of NPO status.

While procedure delays or cancellations for any reason upset patients and families and can disrupt the flow of the operating room and procedural suite, we can achieve the delicate balance between efficiency and patient safety and comfort. Since complex inpatient procedural scheduling may not allow for liberalization of solids requiring 6 to 8 hours of fasting time, focus on liberalizing liquids 2 hours prior to anesthesia. This allows staff to minimize the time low-risk patients fast while still maintaining flexibility for operating room case scheduling. We must promote communication between operating room and floor staff to anticipate timing of procedures each day. Healthcare facilities should aim to achieve time-based preprocedural NPO status as opposed to an arbitrary starting time like midnight.4

RECOMMENDATIONS

  • Risk stratify patients for anesthesia-related aspiration with the aim of identifying those at low aspiration risk.
  • For low-risk patients, adhere to recommended fasting times: 2 hours for a clear carbohydrate beverage, 4 hours for breast milk, 6 hours for a light meal or formula, and 8 hours for a fatty meal.
  • For patients not deemed low risk, determine the appropriate length of preprocedural fasting by consulting with the anesthesia and surgical teams.

CONCLUSION

NPO after midnight represents a low-value and arbitrary practice that leaves patients fasting longer than necessary.2,3,12 In addition to the 2017 ASA guidelines, newer studies and protocols are improving patient satisfaction, minimizing patient dehydration and electrolyte disturbances, and incorporating enhanced recovery after surgery factors into a better patient experience. Returning to the clinical scenario, the hospitalist team can increase patient satisfaction by focusing on liberalizing clear fluids with a carbohydrate beverage up to 2 hours prior to elective surgery while still allowing for schedule flexibility. For this patient, a 3 pm procedure time would have allowed him to have a light breakfast and carbohydrate beverages until 2 hours prior to anesthesia. Dispose of the antiquated practice of NPO after midnight by maximizing clear fluid intake in accordance with current guidelines prior to sedation and general anesthesia. This change in practice will help to achieve normophysiology and increase patient satisfaction.

Do you think this is a low-value practice? Is this truly a “Thing We Do for No Reason™”? Share what you do in your practice and join in the conversation online by retweeting it on Twitter (#TWDFNR) and liking it on Facebook. We invite you to propose ideas for other “Things We Do for No Reason™” topics by emailing TWDFNR@hospitalmedicine.org.

Disclaimer: The opinions expressed in this article are those of the authors alone and do not reflect the views of the Department of Veterans Affairs. The Veterans Affairs Quality Scholars Program is supported by the Veterans Affairs Office of Academic Affiliations, Washington, DC.

References

1. Sorita A, Thongprayoon C, Ahmed A, et al. Frequency and appropriateness of fasting orders in the hospital. Mayo Clin Proc. 2015;90(9):1225-1232. https://doi.org/10.1016/j.mayocp.2015.07.013
2. Falconer R, Skouras C, Carter T, Greenway L, Paisley AM. Preoperative fasting: current practice and areas for improvement. Updates Surg. 2014;66(1):31-39. https://doi.org/10.1007/s13304-013-0242-z
3. Sorita A, Thongprayoon C, Ratelle JT, et al. Characteristics and outcomes of fasting orders among medical inpatients. J Hosp Med. 2017;12(1):36-39. https://doi.org/10.1002/jhm.2674
4. Maltby JR. Fasting from midnight–the history behind the dogma. Best Pract Res Clin Anaesthesiol. 2006;20(3):363-378. https://doi.org/10.1016/j.bpa.2006.02.001
5. Mendelson CL. The aspiration of stomach contents into the lungs during obstetric anesthesia. Am J Obstet Gynecol. 1946;52:191-205. https://doi.org/10.1016/s0002-9378(16)39829-5
6. Shiraishi T, Kurosaki D, Nakamura M, et al. Gastric fluid volume change after oral rehydration solution intake in morbidly obese and normal controls: a magnetic resonance imaging-based analysis. Anesth Analg. 2017;124(4):1174-1178. https://doi.org/10.1213/ane.0000000000001886
7. Vasavid P, Chaiwatanarat T, Pusuwan P, et al. Normal solid gastric emptying values measured by scintigraphy using Asian-style meal: a multicenter study in healthy volunteers. J Neurogastroenterol Motil. 2014;20(3):371-378. https://doi.org/10.5056/jnm13114
8. Mahajan V, Hashmi J, Singh R, Samra T, Aneja S. Comparative evaluation of gastric pH and volume in morbidly obese and lean patients undergoing elective surgery and effect of aspiration prophylaxis. J Clin Anesth. 2015;27(5):396-400. https://doi.org/10.1016/j.jclinane.2015.03.004
9. Brady MC, Kinn S, Stuart P, Ness V. Preoperative fasting for adults to prevent perioperative complications. Cochrane Database Syst Rev. 2003;(4):CD004423. https://doi.org/10.1002/14651858.cd004423
10. Warner MA, Warner ME, Warner DO, Warner LO, Warner EJ. Perioperative pulmonary aspiration in infants and children. Anesthesiology. 1999;90(1):66-71. https://doi.org/10.1097/00000542-199901000-00011
11. Practice guidelines for preoperative fasting and the use of pharmacologic agents to reduce the risk of pulmonary aspiration: application to healthy patients undergoing elective procedures: a report by the American Society of Anesthesiologist Task Force on Preoperative Fasting. Anesthesiology. 1999;90(3):896-905. https://doi.org/10.1097/00000542-199903000-00034
12. Practice guidelines for preoperative fasting and the use of pharmacologic agents to reduce the risk of pulmonary aspiration: application to healthy patients undergoing elective procedures: an updated report by the American Society of Anesthesiologists task force on preoperative fasting and the use of pharmacologic agents to reduce the risk of pulmonary aspiration. Anesthesiology. 2017;126(3):376-393. https://doi.org/10.1097/aln.0000000000001452
13. Hausel J, Nygren J, Lagerkranser M, et al. A carbohydrate-rich drink reduces preoperative discomfort in elective surgery patients. Anesth Analg. 2001;93(5):1344-1350. https://doi.org/10.1097/00000539-200111000-00063
14. Awad S, Varadhan KK, Ljungqvist O, Lobo DN. A meta-analysis of randomised controlled trials on preoperative oral carbohydrate treatment in elective surgery. Clin Nutr. 2013;32(1):34-44. https://doi.org/10.1016/j.clnu.2012.10.011
15. Kaška M, Grosmanová T, Havel E, et al. The impact and safety of preoperative oral or intravenous carbohydrate administration versus fasting in colorectal surgery–a randomized controlled trial. Wien Klin Wochenschr. 2010;122(1-2):23-30. https://doi.org/10.1007/s00508-009-1291-7
16. Tokumine J, Sugahara K, Fuchigami T, Teruya K, Nitta K, Satou K. Unanticipated full stomach at anesthesia induction in a type I diabetic patient with asymptomatic gastroparesis. J Anesth. 2005;19(3):247-248. https://doi.org/10.1007/s00540-005-0321-5

References

1. Sorita A, Thongprayoon C, Ahmed A, et al. Frequency and appropriateness of fasting orders in the hospital. Mayo Clin Proc. 2015;90(9):1225-1232. https://doi.org/10.1016/j.mayocp.2015.07.013
2. Falconer R, Skouras C, Carter T, Greenway L, Paisley AM. Preoperative fasting: current practice and areas for improvement. Updates Surg. 2014;66(1):31-39. https://doi.org/10.1007/s13304-013-0242-z
3. Sorita A, Thongprayoon C, Ratelle JT, et al. Characteristics and outcomes of fasting orders among medical inpatients. J Hosp Med. 2017;12(1):36-39. https://doi.org/10.1002/jhm.2674
4. Maltby JR. Fasting from midnight–the history behind the dogma. Best Pract Res Clin Anaesthesiol. 2006;20(3):363-378. https://doi.org/10.1016/j.bpa.2006.02.001
5. Mendelson CL. The aspiration of stomach contents into the lungs during obstetric anesthesia. Am J Obstet Gynecol. 1946;52:191-205. https://doi.org/10.1016/s0002-9378(16)39829-5
6. Shiraishi T, Kurosaki D, Nakamura M, et al. Gastric fluid volume change after oral rehydration solution intake in morbidly obese and normal controls: a magnetic resonance imaging-based analysis. Anesth Analg. 2017;124(4):1174-1178. https://doi.org/10.1213/ane.0000000000001886
7. Vasavid P, Chaiwatanarat T, Pusuwan P, et al. Normal solid gastric emptying values measured by scintigraphy using Asian-style meal: a multicenter study in healthy volunteers. J Neurogastroenterol Motil. 2014;20(3):371-378. https://doi.org/10.5056/jnm13114
8. Mahajan V, Hashmi J, Singh R, Samra T, Aneja S. Comparative evaluation of gastric pH and volume in morbidly obese and lean patients undergoing elective surgery and effect of aspiration prophylaxis. J Clin Anesth. 2015;27(5):396-400. https://doi.org/10.1016/j.jclinane.2015.03.004
9. Brady MC, Kinn S, Stuart P, Ness V. Preoperative fasting for adults to prevent perioperative complications. Cochrane Database Syst Rev. 2003;(4):CD004423. https://doi.org/10.1002/14651858.cd004423
10. Warner MA, Warner ME, Warner DO, Warner LO, Warner EJ. Perioperative pulmonary aspiration in infants and children. Anesthesiology. 1999;90(1):66-71. https://doi.org/10.1097/00000542-199901000-00011
11. Practice guidelines for preoperative fasting and the use of pharmacologic agents to reduce the risk of pulmonary aspiration: application to healthy patients undergoing elective procedures: a report by the American Society of Anesthesiologist Task Force on Preoperative Fasting. Anesthesiology. 1999;90(3):896-905. https://doi.org/10.1097/00000542-199903000-00034
12. Practice guidelines for preoperative fasting and the use of pharmacologic agents to reduce the risk of pulmonary aspiration: application to healthy patients undergoing elective procedures: an updated report by the American Society of Anesthesiologists task force on preoperative fasting and the use of pharmacologic agents to reduce the risk of pulmonary aspiration. Anesthesiology. 2017;126(3):376-393. https://doi.org/10.1097/aln.0000000000001452
13. Hausel J, Nygren J, Lagerkranser M, et al. A carbohydrate-rich drink reduces preoperative discomfort in elective surgery patients. Anesth Analg. 2001;93(5):1344-1350. https://doi.org/10.1097/00000539-200111000-00063
14. Awad S, Varadhan KK, Ljungqvist O, Lobo DN. A meta-analysis of randomised controlled trials on preoperative oral carbohydrate treatment in elective surgery. Clin Nutr. 2013;32(1):34-44. https://doi.org/10.1016/j.clnu.2012.10.011
15. Kaška M, Grosmanová T, Havel E, et al. The impact and safety of preoperative oral or intravenous carbohydrate administration versus fasting in colorectal surgery–a randomized controlled trial. Wien Klin Wochenschr. 2010;122(1-2):23-30. https://doi.org/10.1007/s00508-009-1291-7
16. Tokumine J, Sugahara K, Fuchigami T, Teruya K, Nitta K, Satou K. Unanticipated full stomach at anesthesia induction in a type I diabetic patient with asymptomatic gastroparesis. J Anesth. 2005;19(3):247-248. https://doi.org/10.1007/s00540-005-0321-5

Issue
Journal of Hospital Medicine 16(6)
Issue
Journal of Hospital Medicine 16(6)
Page Number
368-370. Published Online First May 19, 2021
Page Number
368-370. Published Online First May 19, 2021
Publications
Publications
Topics
Article Type
Display Headline
Things We Do for No Reason™: NPO After Midnight
Display Headline
Things We Do for No Reason™: NPO After Midnight
Sections
Article Source

© 2021 Society of Hospital Medicine

Disallow All Ads
Correspondence Location
Richard M Wardrop III, MD, PhD, FAAP, FACP; Email: Richard.wardrop@stdom.com; Telephone: 601-200-4644.
Content Gating
Gated (full article locked unless allowed per User)
Alternative CME
Disqus Comments
Default
Use ProPublica
Hide sidebar & use full width
render the right sidebar.
Conference Recap Checkbox
Not Conference Recap
Clinical Edge
Display the Slideshow in this Article
Gating Strategy
First Page Free
Medscape Article
Display survey writer
Reuters content
Disable Inline Native ads
WebMD Article
Article PDF Media
Image
Teambase ID
18001E35.SIG
Disable zoom
Off

Things We Do for No Reason™: Ova and Parasite Testing in Patients With Acute Diarrhea Arising During Hospitalization

Article Type
Changed
Tue, 03/30/2021 - 14:11
Display Headline
Things We Do for No Reason™: Ova and Parasite Testing in Patients With Acute Diarrhea Arising During Hospitalization

Inspired by the ABIM Foundation’s Choosing Wisely® campaign, the “Things We Do for No Reason” (TWDFNR) series reviews practices that have become common parts of hospital care but may provide little value to our patients. Practices reviewed in the TWDFNR series do not represent clear-cut conclusions or clinical practice standards but are meant as a starting place for research and active discussions among hospitalists and patients. We invite you to be part of that discussion.

CLINICAL SCENARIO

A 54-year-old immunocompetent man admitted to the hospital for non–ST-segment elevation myocardial infarction develops profuse watery diarrhea after his third day of admission. He denies prior episodes of diarrhea. He does not have any fevers, blood in the stool, recent travel, or antibiotic use. Vital signs include a blood pressure of 128/82 mm Hg, heart rate of 120 beats per minute, respiratory rate of 16 breaths per min, oxygen saturation of 100% on room air, and temperature of 36.9 °C. His physical examination is normal, without signs of abdominal tenderness, rebound, or guarding. Complete blood count is normal, without eosinophilia. The comprehensive metabolic panel shows mild hypokalemia of 3.3 mmol/L. The hospitalist resuscitates him with normal saline, provides oral potassium repletion, and orders a stool culture, Clostridioides difficile test, and an ova and parasite (O&P) test. Loperamide and time resolve his symptoms in 2 days. Results of his stool culture, C difficile, and O&P tests return negative in 3 days.

BACKGROUND

Acute diarrhea is a common complaint in both inpatient and outpatient settings. It is defined as the passage of three or more liquid or poorly formed stools in a 24-hour period lasting less than 14 days. Persistent diarrhea lasts from 14 to 29 days, while chronic diarrhea lasts longer than 30 days. There are 47.8 million cases of acute diarrhea per year in the United States, costing $150 million in US health expenditures.1 Viral pathogens remain the most common cause of acute diarrhea in the United States.1,2 Standard O&P testing consists of applying a stool sample to a slide with either saline or iodine (wet mount) and evaluating the specimen with a microscope.

WHY YOU MIGHT THINK O&P TESTING IS HELPFUL

Giardia and Cryptosporidium remain the most commonly implicated parasitic pathogens in acute diarrheal episodes in the United States.3Cryptosporidium cases in the United States range from 2.2 to 3.9 per 100,000 persons,4 and Giardia cases in the United States range from 5.8 to 6.4 per 100,000 persons.5 To avoid missing potentially treatable causes, providers often order O&P tests reflexively as part of a standard workup for acute diarrhea. From 2001 to 2007, Associated Regional and University Pathologists Laboratories experienced a 379% increase in O&P testing.6 Many providers ordering these tests assume that standard O&P testing covers most, if not all, parasites and that a negative test will rule out a parasitic cause of disease. Furthermore, providers are unaware that more sensitive tests to detect certain parasites have replaced standard O&P microscopy.3

WHY O&P TESTING IS USUALLY UNNECESSARY

Most hospitalized patients do not have a parasitic infection

In a review of 5,681 completed O&P tests from a tertiary care medical center in Canada over a 5-year period, only 1.4% of tests were positive.7 In a 3-year retrospective analysis of stool samples obtained after 3 days of hospitalization, positive results were found in only 1 of 191 stool cultures and in 0 of 90 O&P samples.8 Current practice guidelines suggest not testing patients with stool studies in cases of acute diarrhea lasting less than 7 days in the absence of significant risk factors for parasitic disease because it has been shown to be a rare event and most cases will self-resolve with supportive care only.1,9

The stool O&P test has low sensitivity

Classically ordered stool O&P tests have low sensitivity for the detection of Giardia and Cryptosporidium, the two most common parasites in the United States.6,10,11 O&P studies detect Giardia in only 66% to 79% of specimen samples and Cryptosporidium in less than 5% of specimens. Diagnostic yields can be improved with the use of special stains such as modified acid-fast stain (MAF).6 Despite use of MAF staining, though, sensitivity for Cryptosporidium detection has remained at only 55%.12 Additionally, several studies have shown that physicians are generally unaware of the test characteristics of stool O&P tests and they do not know to order the newer more sensitive enzyme immunoassays (EIA) or direct fluorescent antibody (DFA) tests even in situations when testing for a parasitic infection is appropriate.10,11,13,14 As stated earlier, a parasitic infection without significant risk factors is a rare event. A negative test with low or moderate sensitivity is not additive to such a low clinical suspicion because it does not significantly change posttest probability.

Testing can have adverse consequences

In addition to the low yield, O&P testing is technically complex, is time intensive, and requires an experienced technician’s interpretation. Inappropriate testing increases the cost of care and staff workload without much benefit.6 As such, some institutions have opted to send the O&P tests to labs with experienced technicians. Other institutions have adopted a restrictive stool O&P testing approach that reduces healthcare time and costs and improves the rate of positive tests.13,15 A study at a single tertiary care medical center demonstrated an estimated cost savings of $21,931 annually by implementing a computer-based algorithm to restrict testing for stool cultures and O&P tests to patients with higher probabilities of infection.15 The algorithm directed clinicians to provide further information when attempting to order stool culture, O&P, or other specific stool tests. For patients hospitalized for more than 3 days, the system did not allow certain testing. For patients with worrisome features like severe symptoms or an immunocompromised state, the algorithm directed the clinician to place an infectious disease or gastroenterology consult rather than order stool tests. Decreased laboratory costs of all stool studies (including O&P) in adult inpatient locations led to the cost savings. Additionally, the study authors felt that they likely underestimated the cost savings because they did not account for other expenses in the analysis, such as nursing workload and supplies.15

WHAT YOU SHOULD DO INSTEAD

Clinicians should evaluate patients on a case-by-case basis and determine the need to test based on the presence of high-risk features (Table).

golfeyz02830317e_t1.jpg
Perform O&P testing only in patients with a high pretest probability of having a parasitic disease that will not resolve on its own.1,16 For example, if a patient recently returned from South America with acute diarrhea, EIA testing should be performed for Entamoeba histolytica. If you order O&P tests, you should order at least three spaced over 10 days to increase sensitivity. The yield with one test is 50% to 60%, but with three tests, it is >95%.17 Additionally, it is important to send a fresh stool sample that has not been contaminated with water or urine, both of which may lead to false positives. Most cases of acute diarrheal illnesses, however, do not require O&P evaluation and resolve with supportive treatment alone.

When performing parasitic testing in patients without a recent travel history but with other high-risk features, test for the most prevalent parasites in the United States (ie, Giardia, Cryptosporidium, and Entamoeba histolytica) with DFA or EIA tests.3 DFA testing for Giardia is 99% sensitive.12 In patients with symptoms lasting more than 7 days and recent travel, in addition to the above DFA/EIA tests, perform O&P testing with wet mount, modified acid-fast bacilli stain to detect rare parasites such as helminths, Strongyloides, Cyclospora, and Cystoisospora.3 In patients who live or travel to endemic areas (about 10% of traveler’s diarrhea is caused by parasitic infections), have unexplained eosinophilia, or are part of a community outbreak (eg, childcare institutions or drinking water/food outbreaks), test for Giardia, Cryptosporidium, Cyclospora, Cystoisospora, Entamoeba histolytica, and Isospora belli.9 In addition, among patients with AIDS or immunosuppression, testing should include assays for Microsporidia, Strongyloides, and Mycobacterium avium complex (Figure).9,16 Newer tests, such as the multiplex real-time polymerase chain reaction assay, can also simultaneously detect Entamoeba histolytica, Giardia lamblia, and Cryptosporidium parvum. For more information on parasitic testing, we suggest reading the review article “Beyond O&P times three.”3 It is important to familiarize yourself with the parasitic tests available at your respective clinics/hospital so the optimal test can be used.

golfeyz02830317e_f1.jpg

RECOMMENDATIONS

  • Prescribe a trial of “wait and see” for patients without high-risk features for parasitic disease.
  • Test patients with high-risk features for parasitic disease by utilizing targeted testing.
  • For patients with high-risk features but no travel history, first perform DFA, EIA, or multiplex real-time polymerase chain reaction testing to evaluate for Giardia, Cryptosporidium, and Entamoeba histolytica.
  • If DFA/EIA testing is negative, obtain O&P tests with and without stains, such as acid-fast bacilli, for detection of other rare parasites.

CONCLUSION

Hospitalists should risk-stratify patients to determine when O&P testing is appropriate. Employ more targeted testing, especially use of DFA/EIA tests when evaluating for parasites. Avoid parasitic testing if symptoms have lasted less than 7 days and the patient has no other high-risk features. Become familiar with the tests available at your institution and their sensitivities. As in our clinical scenario, most acute cases of diarrhea resolve without intervention and should be managed and treated conservatively.

Do you think this is a low-value practice? Is this truly a “Thing We Do for No Reason”? Share what you do in your practice and join in the conversation online by retweeting it on Twitter (#TWDFNR) and liking it on Facebook. We invite you to propose ideas for other “Things We Do for No Reason ” topics by emailing TWDFNR@hospitalmedicine.org .

Acknowledgments

The authors thank Dr Lenny Feldman for his assistance with editing the manuscript.

References

1. Riddle MS, DuPont HL, Connor BA. ACG clinical guideline: diagnosis, treatment, and prevention of acute diarrheal infections in adults. Am J Gastroenterol. 2016;111(5):602-622. https://doi.org/10.1038/ajg.2016.126
2. DuPont HL. Acute infectious diarrhea in immunocompetent adults. N Engl J Med. 2014;370(16):1532-1540. https://doi.org/10.1056/nejmra1301069
3. Mohapatra S, Singh DP, Alcid D, Pitchumoni CS. Beyond O&P times three. Am J Gastroenterol. 2018;113(6):805-818. https://doi.org/10.1038/s41395-018-0083-y
4. Painter JE, Hlavsa MC, Collier SA, Xiao L, Yoder JS. Cryptosporidiosis surveillance -- United States, 2011-2012. MMWR Suppl. 2015;64(3):1-14.
5. Painter JE, Gargano JW, Collier SA, Yoder JS. Giardiasis surveillance -- United States, 2011-2012. MMWR Suppl. 2015;64(3):15-25.
6. Polage CR, Stoddard GJ, Rolfs RT, Petti CA. Physician use of parasite tests in the United States from 1997 to 2006 and in a Utah Cryptosporidium outbreak in 2007. J Clin Microbiol. 2011;49(2):591-596. https://doi.org/10.1128/jcm.01806-10
7. Mosli M, Gregor J, Chande N, Lannigan R. Nonutility of routine testing of stool for ova and parasites in a tertiary care Canadian centre. Can J Microbiol. 2012;58(5):653-659. https://doi.org/10.1139/w2012-039
8. Siegel DL, Edelstein PH, Nachamkin I. Inappropriate testing for diarrheal diseases in the hospital. JAMA. 1990;263(7):979-982.
9. Shane AL, Mody RK, Crump JA, et al. 2017 Infectious Diseases Society of America clinical practice guidelines for the diagnosis and management of infectious diarrhea. Clin Infect Dis. 2017;65(12):e45-e80. https://doi.org/10.1093/cid/cix669
10. Hennessy TW, Marcus R, Deneen V, et al. Survey of physician diagnostic practices for patients with acute diarrhea: clinical and public health implications. Clin Infect Dis. 2004;38 (Suppl 3):S203-S211. https://doi.org/10.1086/381588
11. Morin CA, Roberts CL, Mshar PA, Addiss DG, Hadler JL. What do physicians know about cryptosporidiosis? a survey of Connecticut physicians. Arch Intern Med. 1997;157(9):1017-1022.
12. McHardy IH, Wu M, Shimizu-Cohen R, Couturier MR, Humphries RM. Detection of intestinal protozoa in the clinical laboratory. J Clin Microbiol. 2014;52(3):712-720. https://doi.org/10.1128/jcm.02877-13
13. Valenstein P, Pfaller M, Yungbluth M. The use and abuse of routine stool microbiology: a College of American Pathologists Q-probes study of 601 institutions. Arch Pathol Lab Med. 1996;120(2):206-211.
14. Jones JL, Lopez A, Wahlquist SP, Nadle J, Wilson M; Emerging Infections Program FoodNet Working Group. Survey of clinical laboratory practices for parasitic diseases. Clin Infect Dis. 2004;38(Suppl 3):S198-S202. https://doi.org/10.1086/381587
15. Tewell CE, Talbot TR, Nelson GE, et al. Reducing inappropriate testing for the evaluation of diarrhea among hospitalized patients. Am J Med. 2018;131(2):193-199.e1. https://doi.org/10.1016/j.amjmed.2017.10.006
16. Thielman NM, Guerrant RL. Clinical practice. acute infectious diarrhea. N Engl J Med. 2004;350(1):38-47. https://doi.org/10.1056/nejmcp031534
17. Marti H, Koella JC. Multiple stool examinations for ova and parasites and rate of false-negative results. J Clin Microbiol. 1993;31(11):3044-3045. https://doi.org/10.1128/jcm.31.11.3044-3045.1993

Article PDF
Author and Disclosure Information

1Division of Gastroenterology and Hepatology, Maimonides Medical Center, Brooklyn, New York; 2Division of Infectious Diseases, Montefiore Hospital and Medical Center, Bronx, New York; 3Division of Hospital Medicine, Mount Sinai Beth Israel, New York, New York; 4Department of Medicine, Icahn School of Medicine at Mount Sinai, New York, New York.

Disclosures

The authors have nothing to disclose.

Issue
Journal of Hospital Medicine 16(4)
Publications
Topics
Page Number
236-238. Published Online First March 17, 2021
Sections
Author and Disclosure Information

1Division of Gastroenterology and Hepatology, Maimonides Medical Center, Brooklyn, New York; 2Division of Infectious Diseases, Montefiore Hospital and Medical Center, Bronx, New York; 3Division of Hospital Medicine, Mount Sinai Beth Israel, New York, New York; 4Department of Medicine, Icahn School of Medicine at Mount Sinai, New York, New York.

Disclosures

The authors have nothing to disclose.

Author and Disclosure Information

1Division of Gastroenterology and Hepatology, Maimonides Medical Center, Brooklyn, New York; 2Division of Infectious Diseases, Montefiore Hospital and Medical Center, Bronx, New York; 3Division of Hospital Medicine, Mount Sinai Beth Israel, New York, New York; 4Department of Medicine, Icahn School of Medicine at Mount Sinai, New York, New York.

Disclosures

The authors have nothing to disclose.

Article PDF
Article PDF
Related Articles

Inspired by the ABIM Foundation’s Choosing Wisely® campaign, the “Things We Do for No Reason” (TWDFNR) series reviews practices that have become common parts of hospital care but may provide little value to our patients. Practices reviewed in the TWDFNR series do not represent clear-cut conclusions or clinical practice standards but are meant as a starting place for research and active discussions among hospitalists and patients. We invite you to be part of that discussion.

CLINICAL SCENARIO

A 54-year-old immunocompetent man admitted to the hospital for non–ST-segment elevation myocardial infarction develops profuse watery diarrhea after his third day of admission. He denies prior episodes of diarrhea. He does not have any fevers, blood in the stool, recent travel, or antibiotic use. Vital signs include a blood pressure of 128/82 mm Hg, heart rate of 120 beats per minute, respiratory rate of 16 breaths per min, oxygen saturation of 100% on room air, and temperature of 36.9 °C. His physical examination is normal, without signs of abdominal tenderness, rebound, or guarding. Complete blood count is normal, without eosinophilia. The comprehensive metabolic panel shows mild hypokalemia of 3.3 mmol/L. The hospitalist resuscitates him with normal saline, provides oral potassium repletion, and orders a stool culture, Clostridioides difficile test, and an ova and parasite (O&P) test. Loperamide and time resolve his symptoms in 2 days. Results of his stool culture, C difficile, and O&P tests return negative in 3 days.

BACKGROUND

Acute diarrhea is a common complaint in both inpatient and outpatient settings. It is defined as the passage of three or more liquid or poorly formed stools in a 24-hour period lasting less than 14 days. Persistent diarrhea lasts from 14 to 29 days, while chronic diarrhea lasts longer than 30 days. There are 47.8 million cases of acute diarrhea per year in the United States, costing $150 million in US health expenditures.1 Viral pathogens remain the most common cause of acute diarrhea in the United States.1,2 Standard O&P testing consists of applying a stool sample to a slide with either saline or iodine (wet mount) and evaluating the specimen with a microscope.

WHY YOU MIGHT THINK O&P TESTING IS HELPFUL

Giardia and Cryptosporidium remain the most commonly implicated parasitic pathogens in acute diarrheal episodes in the United States.3Cryptosporidium cases in the United States range from 2.2 to 3.9 per 100,000 persons,4 and Giardia cases in the United States range from 5.8 to 6.4 per 100,000 persons.5 To avoid missing potentially treatable causes, providers often order O&P tests reflexively as part of a standard workup for acute diarrhea. From 2001 to 2007, Associated Regional and University Pathologists Laboratories experienced a 379% increase in O&P testing.6 Many providers ordering these tests assume that standard O&P testing covers most, if not all, parasites and that a negative test will rule out a parasitic cause of disease. Furthermore, providers are unaware that more sensitive tests to detect certain parasites have replaced standard O&P microscopy.3

WHY O&P TESTING IS USUALLY UNNECESSARY

Most hospitalized patients do not have a parasitic infection

In a review of 5,681 completed O&P tests from a tertiary care medical center in Canada over a 5-year period, only 1.4% of tests were positive.7 In a 3-year retrospective analysis of stool samples obtained after 3 days of hospitalization, positive results were found in only 1 of 191 stool cultures and in 0 of 90 O&P samples.8 Current practice guidelines suggest not testing patients with stool studies in cases of acute diarrhea lasting less than 7 days in the absence of significant risk factors for parasitic disease because it has been shown to be a rare event and most cases will self-resolve with supportive care only.1,9

The stool O&P test has low sensitivity

Classically ordered stool O&P tests have low sensitivity for the detection of Giardia and Cryptosporidium, the two most common parasites in the United States.6,10,11 O&P studies detect Giardia in only 66% to 79% of specimen samples and Cryptosporidium in less than 5% of specimens. Diagnostic yields can be improved with the use of special stains such as modified acid-fast stain (MAF).6 Despite use of MAF staining, though, sensitivity for Cryptosporidium detection has remained at only 55%.12 Additionally, several studies have shown that physicians are generally unaware of the test characteristics of stool O&P tests and they do not know to order the newer more sensitive enzyme immunoassays (EIA) or direct fluorescent antibody (DFA) tests even in situations when testing for a parasitic infection is appropriate.10,11,13,14 As stated earlier, a parasitic infection without significant risk factors is a rare event. A negative test with low or moderate sensitivity is not additive to such a low clinical suspicion because it does not significantly change posttest probability.

Testing can have adverse consequences

In addition to the low yield, O&P testing is technically complex, is time intensive, and requires an experienced technician’s interpretation. Inappropriate testing increases the cost of care and staff workload without much benefit.6 As such, some institutions have opted to send the O&P tests to labs with experienced technicians. Other institutions have adopted a restrictive stool O&P testing approach that reduces healthcare time and costs and improves the rate of positive tests.13,15 A study at a single tertiary care medical center demonstrated an estimated cost savings of $21,931 annually by implementing a computer-based algorithm to restrict testing for stool cultures and O&P tests to patients with higher probabilities of infection.15 The algorithm directed clinicians to provide further information when attempting to order stool culture, O&P, or other specific stool tests. For patients hospitalized for more than 3 days, the system did not allow certain testing. For patients with worrisome features like severe symptoms or an immunocompromised state, the algorithm directed the clinician to place an infectious disease or gastroenterology consult rather than order stool tests. Decreased laboratory costs of all stool studies (including O&P) in adult inpatient locations led to the cost savings. Additionally, the study authors felt that they likely underestimated the cost savings because they did not account for other expenses in the analysis, such as nursing workload and supplies.15

WHAT YOU SHOULD DO INSTEAD

Clinicians should evaluate patients on a case-by-case basis and determine the need to test based on the presence of high-risk features (Table).

golfeyz02830317e_t1.jpg
Perform O&P testing only in patients with a high pretest probability of having a parasitic disease that will not resolve on its own.1,16 For example, if a patient recently returned from South America with acute diarrhea, EIA testing should be performed for Entamoeba histolytica. If you order O&P tests, you should order at least three spaced over 10 days to increase sensitivity. The yield with one test is 50% to 60%, but with three tests, it is >95%.17 Additionally, it is important to send a fresh stool sample that has not been contaminated with water or urine, both of which may lead to false positives. Most cases of acute diarrheal illnesses, however, do not require O&P evaluation and resolve with supportive treatment alone.

When performing parasitic testing in patients without a recent travel history but with other high-risk features, test for the most prevalent parasites in the United States (ie, Giardia, Cryptosporidium, and Entamoeba histolytica) with DFA or EIA tests.3 DFA testing for Giardia is 99% sensitive.12 In patients with symptoms lasting more than 7 days and recent travel, in addition to the above DFA/EIA tests, perform O&P testing with wet mount, modified acid-fast bacilli stain to detect rare parasites such as helminths, Strongyloides, Cyclospora, and Cystoisospora.3 In patients who live or travel to endemic areas (about 10% of traveler’s diarrhea is caused by parasitic infections), have unexplained eosinophilia, or are part of a community outbreak (eg, childcare institutions or drinking water/food outbreaks), test for Giardia, Cryptosporidium, Cyclospora, Cystoisospora, Entamoeba histolytica, and Isospora belli.9 In addition, among patients with AIDS or immunosuppression, testing should include assays for Microsporidia, Strongyloides, and Mycobacterium avium complex (Figure).9,16 Newer tests, such as the multiplex real-time polymerase chain reaction assay, can also simultaneously detect Entamoeba histolytica, Giardia lamblia, and Cryptosporidium parvum. For more information on parasitic testing, we suggest reading the review article “Beyond O&P times three.”3 It is important to familiarize yourself with the parasitic tests available at your respective clinics/hospital so the optimal test can be used.

golfeyz02830317e_f1.jpg

RECOMMENDATIONS

  • Prescribe a trial of “wait and see” for patients without high-risk features for parasitic disease.
  • Test patients with high-risk features for parasitic disease by utilizing targeted testing.
  • For patients with high-risk features but no travel history, first perform DFA, EIA, or multiplex real-time polymerase chain reaction testing to evaluate for Giardia, Cryptosporidium, and Entamoeba histolytica.
  • If DFA/EIA testing is negative, obtain O&P tests with and without stains, such as acid-fast bacilli, for detection of other rare parasites.

CONCLUSION

Hospitalists should risk-stratify patients to determine when O&P testing is appropriate. Employ more targeted testing, especially use of DFA/EIA tests when evaluating for parasites. Avoid parasitic testing if symptoms have lasted less than 7 days and the patient has no other high-risk features. Become familiar with the tests available at your institution and their sensitivities. As in our clinical scenario, most acute cases of diarrhea resolve without intervention and should be managed and treated conservatively.

Do you think this is a low-value practice? Is this truly a “Thing We Do for No Reason”? Share what you do in your practice and join in the conversation online by retweeting it on Twitter (#TWDFNR) and liking it on Facebook. We invite you to propose ideas for other “Things We Do for No Reason ” topics by emailing TWDFNR@hospitalmedicine.org .

Acknowledgments

The authors thank Dr Lenny Feldman for his assistance with editing the manuscript.

Inspired by the ABIM Foundation’s Choosing Wisely® campaign, the “Things We Do for No Reason” (TWDFNR) series reviews practices that have become common parts of hospital care but may provide little value to our patients. Practices reviewed in the TWDFNR series do not represent clear-cut conclusions or clinical practice standards but are meant as a starting place for research and active discussions among hospitalists and patients. We invite you to be part of that discussion.

CLINICAL SCENARIO

A 54-year-old immunocompetent man admitted to the hospital for non–ST-segment elevation myocardial infarction develops profuse watery diarrhea after his third day of admission. He denies prior episodes of diarrhea. He does not have any fevers, blood in the stool, recent travel, or antibiotic use. Vital signs include a blood pressure of 128/82 mm Hg, heart rate of 120 beats per minute, respiratory rate of 16 breaths per min, oxygen saturation of 100% on room air, and temperature of 36.9 °C. His physical examination is normal, without signs of abdominal tenderness, rebound, or guarding. Complete blood count is normal, without eosinophilia. The comprehensive metabolic panel shows mild hypokalemia of 3.3 mmol/L. The hospitalist resuscitates him with normal saline, provides oral potassium repletion, and orders a stool culture, Clostridioides difficile test, and an ova and parasite (O&P) test. Loperamide and time resolve his symptoms in 2 days. Results of his stool culture, C difficile, and O&P tests return negative in 3 days.

BACKGROUND

Acute diarrhea is a common complaint in both inpatient and outpatient settings. It is defined as the passage of three or more liquid or poorly formed stools in a 24-hour period lasting less than 14 days. Persistent diarrhea lasts from 14 to 29 days, while chronic diarrhea lasts longer than 30 days. There are 47.8 million cases of acute diarrhea per year in the United States, costing $150 million in US health expenditures.1 Viral pathogens remain the most common cause of acute diarrhea in the United States.1,2 Standard O&P testing consists of applying a stool sample to a slide with either saline or iodine (wet mount) and evaluating the specimen with a microscope.

WHY YOU MIGHT THINK O&P TESTING IS HELPFUL

Giardia and Cryptosporidium remain the most commonly implicated parasitic pathogens in acute diarrheal episodes in the United States.3Cryptosporidium cases in the United States range from 2.2 to 3.9 per 100,000 persons,4 and Giardia cases in the United States range from 5.8 to 6.4 per 100,000 persons.5 To avoid missing potentially treatable causes, providers often order O&P tests reflexively as part of a standard workup for acute diarrhea. From 2001 to 2007, Associated Regional and University Pathologists Laboratories experienced a 379% increase in O&P testing.6 Many providers ordering these tests assume that standard O&P testing covers most, if not all, parasites and that a negative test will rule out a parasitic cause of disease. Furthermore, providers are unaware that more sensitive tests to detect certain parasites have replaced standard O&P microscopy.3

WHY O&P TESTING IS USUALLY UNNECESSARY

Most hospitalized patients do not have a parasitic infection

In a review of 5,681 completed O&P tests from a tertiary care medical center in Canada over a 5-year period, only 1.4% of tests were positive.7 In a 3-year retrospective analysis of stool samples obtained after 3 days of hospitalization, positive results were found in only 1 of 191 stool cultures and in 0 of 90 O&P samples.8 Current practice guidelines suggest not testing patients with stool studies in cases of acute diarrhea lasting less than 7 days in the absence of significant risk factors for parasitic disease because it has been shown to be a rare event and most cases will self-resolve with supportive care only.1,9

The stool O&P test has low sensitivity

Classically ordered stool O&P tests have low sensitivity for the detection of Giardia and Cryptosporidium, the two most common parasites in the United States.6,10,11 O&P studies detect Giardia in only 66% to 79% of specimen samples and Cryptosporidium in less than 5% of specimens. Diagnostic yields can be improved with the use of special stains such as modified acid-fast stain (MAF).6 Despite use of MAF staining, though, sensitivity for Cryptosporidium detection has remained at only 55%.12 Additionally, several studies have shown that physicians are generally unaware of the test characteristics of stool O&P tests and they do not know to order the newer more sensitive enzyme immunoassays (EIA) or direct fluorescent antibody (DFA) tests even in situations when testing for a parasitic infection is appropriate.10,11,13,14 As stated earlier, a parasitic infection without significant risk factors is a rare event. A negative test with low or moderate sensitivity is not additive to such a low clinical suspicion because it does not significantly change posttest probability.

Testing can have adverse consequences

In addition to the low yield, O&P testing is technically complex, is time intensive, and requires an experienced technician’s interpretation. Inappropriate testing increases the cost of care and staff workload without much benefit.6 As such, some institutions have opted to send the O&P tests to labs with experienced technicians. Other institutions have adopted a restrictive stool O&P testing approach that reduces healthcare time and costs and improves the rate of positive tests.13,15 A study at a single tertiary care medical center demonstrated an estimated cost savings of $21,931 annually by implementing a computer-based algorithm to restrict testing for stool cultures and O&P tests to patients with higher probabilities of infection.15 The algorithm directed clinicians to provide further information when attempting to order stool culture, O&P, or other specific stool tests. For patients hospitalized for more than 3 days, the system did not allow certain testing. For patients with worrisome features like severe symptoms or an immunocompromised state, the algorithm directed the clinician to place an infectious disease or gastroenterology consult rather than order stool tests. Decreased laboratory costs of all stool studies (including O&P) in adult inpatient locations led to the cost savings. Additionally, the study authors felt that they likely underestimated the cost savings because they did not account for other expenses in the analysis, such as nursing workload and supplies.15

WHAT YOU SHOULD DO INSTEAD

Clinicians should evaluate patients on a case-by-case basis and determine the need to test based on the presence of high-risk features (Table).

golfeyz02830317e_t1.jpg
Perform O&P testing only in patients with a high pretest probability of having a parasitic disease that will not resolve on its own.1,16 For example, if a patient recently returned from South America with acute diarrhea, EIA testing should be performed for Entamoeba histolytica. If you order O&P tests, you should order at least three spaced over 10 days to increase sensitivity. The yield with one test is 50% to 60%, but with three tests, it is >95%.17 Additionally, it is important to send a fresh stool sample that has not been contaminated with water or urine, both of which may lead to false positives. Most cases of acute diarrheal illnesses, however, do not require O&P evaluation and resolve with supportive treatment alone.

When performing parasitic testing in patients without a recent travel history but with other high-risk features, test for the most prevalent parasites in the United States (ie, Giardia, Cryptosporidium, and Entamoeba histolytica) with DFA or EIA tests.3 DFA testing for Giardia is 99% sensitive.12 In patients with symptoms lasting more than 7 days and recent travel, in addition to the above DFA/EIA tests, perform O&P testing with wet mount, modified acid-fast bacilli stain to detect rare parasites such as helminths, Strongyloides, Cyclospora, and Cystoisospora.3 In patients who live or travel to endemic areas (about 10% of traveler’s diarrhea is caused by parasitic infections), have unexplained eosinophilia, or are part of a community outbreak (eg, childcare institutions or drinking water/food outbreaks), test for Giardia, Cryptosporidium, Cyclospora, Cystoisospora, Entamoeba histolytica, and Isospora belli.9 In addition, among patients with AIDS or immunosuppression, testing should include assays for Microsporidia, Strongyloides, and Mycobacterium avium complex (Figure).9,16 Newer tests, such as the multiplex real-time polymerase chain reaction assay, can also simultaneously detect Entamoeba histolytica, Giardia lamblia, and Cryptosporidium parvum. For more information on parasitic testing, we suggest reading the review article “Beyond O&P times three.”3 It is important to familiarize yourself with the parasitic tests available at your respective clinics/hospital so the optimal test can be used.

golfeyz02830317e_f1.jpg

RECOMMENDATIONS

  • Prescribe a trial of “wait and see” for patients without high-risk features for parasitic disease.
  • Test patients with high-risk features for parasitic disease by utilizing targeted testing.
  • For patients with high-risk features but no travel history, first perform DFA, EIA, or multiplex real-time polymerase chain reaction testing to evaluate for Giardia, Cryptosporidium, and Entamoeba histolytica.
  • If DFA/EIA testing is negative, obtain O&P tests with and without stains, such as acid-fast bacilli, for detection of other rare parasites.

CONCLUSION

Hospitalists should risk-stratify patients to determine when O&P testing is appropriate. Employ more targeted testing, especially use of DFA/EIA tests when evaluating for parasites. Avoid parasitic testing if symptoms have lasted less than 7 days and the patient has no other high-risk features. Become familiar with the tests available at your institution and their sensitivities. As in our clinical scenario, most acute cases of diarrhea resolve without intervention and should be managed and treated conservatively.

Do you think this is a low-value practice? Is this truly a “Thing We Do for No Reason”? Share what you do in your practice and join in the conversation online by retweeting it on Twitter (#TWDFNR) and liking it on Facebook. We invite you to propose ideas for other “Things We Do for No Reason ” topics by emailing TWDFNR@hospitalmedicine.org .

Acknowledgments

The authors thank Dr Lenny Feldman for his assistance with editing the manuscript.

References

1. Riddle MS, DuPont HL, Connor BA. ACG clinical guideline: diagnosis, treatment, and prevention of acute diarrheal infections in adults. Am J Gastroenterol. 2016;111(5):602-622. https://doi.org/10.1038/ajg.2016.126
2. DuPont HL. Acute infectious diarrhea in immunocompetent adults. N Engl J Med. 2014;370(16):1532-1540. https://doi.org/10.1056/nejmra1301069
3. Mohapatra S, Singh DP, Alcid D, Pitchumoni CS. Beyond O&P times three. Am J Gastroenterol. 2018;113(6):805-818. https://doi.org/10.1038/s41395-018-0083-y
4. Painter JE, Hlavsa MC, Collier SA, Xiao L, Yoder JS. Cryptosporidiosis surveillance -- United States, 2011-2012. MMWR Suppl. 2015;64(3):1-14.
5. Painter JE, Gargano JW, Collier SA, Yoder JS. Giardiasis surveillance -- United States, 2011-2012. MMWR Suppl. 2015;64(3):15-25.
6. Polage CR, Stoddard GJ, Rolfs RT, Petti CA. Physician use of parasite tests in the United States from 1997 to 2006 and in a Utah Cryptosporidium outbreak in 2007. J Clin Microbiol. 2011;49(2):591-596. https://doi.org/10.1128/jcm.01806-10
7. Mosli M, Gregor J, Chande N, Lannigan R. Nonutility of routine testing of stool for ova and parasites in a tertiary care Canadian centre. Can J Microbiol. 2012;58(5):653-659. https://doi.org/10.1139/w2012-039
8. Siegel DL, Edelstein PH, Nachamkin I. Inappropriate testing for diarrheal diseases in the hospital. JAMA. 1990;263(7):979-982.
9. Shane AL, Mody RK, Crump JA, et al. 2017 Infectious Diseases Society of America clinical practice guidelines for the diagnosis and management of infectious diarrhea. Clin Infect Dis. 2017;65(12):e45-e80. https://doi.org/10.1093/cid/cix669
10. Hennessy TW, Marcus R, Deneen V, et al. Survey of physician diagnostic practices for patients with acute diarrhea: clinical and public health implications. Clin Infect Dis. 2004;38 (Suppl 3):S203-S211. https://doi.org/10.1086/381588
11. Morin CA, Roberts CL, Mshar PA, Addiss DG, Hadler JL. What do physicians know about cryptosporidiosis? a survey of Connecticut physicians. Arch Intern Med. 1997;157(9):1017-1022.
12. McHardy IH, Wu M, Shimizu-Cohen R, Couturier MR, Humphries RM. Detection of intestinal protozoa in the clinical laboratory. J Clin Microbiol. 2014;52(3):712-720. https://doi.org/10.1128/jcm.02877-13
13. Valenstein P, Pfaller M, Yungbluth M. The use and abuse of routine stool microbiology: a College of American Pathologists Q-probes study of 601 institutions. Arch Pathol Lab Med. 1996;120(2):206-211.
14. Jones JL, Lopez A, Wahlquist SP, Nadle J, Wilson M; Emerging Infections Program FoodNet Working Group. Survey of clinical laboratory practices for parasitic diseases. Clin Infect Dis. 2004;38(Suppl 3):S198-S202. https://doi.org/10.1086/381587
15. Tewell CE, Talbot TR, Nelson GE, et al. Reducing inappropriate testing for the evaluation of diarrhea among hospitalized patients. Am J Med. 2018;131(2):193-199.e1. https://doi.org/10.1016/j.amjmed.2017.10.006
16. Thielman NM, Guerrant RL. Clinical practice. acute infectious diarrhea. N Engl J Med. 2004;350(1):38-47. https://doi.org/10.1056/nejmcp031534
17. Marti H, Koella JC. Multiple stool examinations for ova and parasites and rate of false-negative results. J Clin Microbiol. 1993;31(11):3044-3045. https://doi.org/10.1128/jcm.31.11.3044-3045.1993

References

1. Riddle MS, DuPont HL, Connor BA. ACG clinical guideline: diagnosis, treatment, and prevention of acute diarrheal infections in adults. Am J Gastroenterol. 2016;111(5):602-622. https://doi.org/10.1038/ajg.2016.126
2. DuPont HL. Acute infectious diarrhea in immunocompetent adults. N Engl J Med. 2014;370(16):1532-1540. https://doi.org/10.1056/nejmra1301069
3. Mohapatra S, Singh DP, Alcid D, Pitchumoni CS. Beyond O&P times three. Am J Gastroenterol. 2018;113(6):805-818. https://doi.org/10.1038/s41395-018-0083-y
4. Painter JE, Hlavsa MC, Collier SA, Xiao L, Yoder JS. Cryptosporidiosis surveillance -- United States, 2011-2012. MMWR Suppl. 2015;64(3):1-14.
5. Painter JE, Gargano JW, Collier SA, Yoder JS. Giardiasis surveillance -- United States, 2011-2012. MMWR Suppl. 2015;64(3):15-25.
6. Polage CR, Stoddard GJ, Rolfs RT, Petti CA. Physician use of parasite tests in the United States from 1997 to 2006 and in a Utah Cryptosporidium outbreak in 2007. J Clin Microbiol. 2011;49(2):591-596. https://doi.org/10.1128/jcm.01806-10
7. Mosli M, Gregor J, Chande N, Lannigan R. Nonutility of routine testing of stool for ova and parasites in a tertiary care Canadian centre. Can J Microbiol. 2012;58(5):653-659. https://doi.org/10.1139/w2012-039
8. Siegel DL, Edelstein PH, Nachamkin I. Inappropriate testing for diarrheal diseases in the hospital. JAMA. 1990;263(7):979-982.
9. Shane AL, Mody RK, Crump JA, et al. 2017 Infectious Diseases Society of America clinical practice guidelines for the diagnosis and management of infectious diarrhea. Clin Infect Dis. 2017;65(12):e45-e80. https://doi.org/10.1093/cid/cix669
10. Hennessy TW, Marcus R, Deneen V, et al. Survey of physician diagnostic practices for patients with acute diarrhea: clinical and public health implications. Clin Infect Dis. 2004;38 (Suppl 3):S203-S211. https://doi.org/10.1086/381588
11. Morin CA, Roberts CL, Mshar PA, Addiss DG, Hadler JL. What do physicians know about cryptosporidiosis? a survey of Connecticut physicians. Arch Intern Med. 1997;157(9):1017-1022.
12. McHardy IH, Wu M, Shimizu-Cohen R, Couturier MR, Humphries RM. Detection of intestinal protozoa in the clinical laboratory. J Clin Microbiol. 2014;52(3):712-720. https://doi.org/10.1128/jcm.02877-13
13. Valenstein P, Pfaller M, Yungbluth M. The use and abuse of routine stool microbiology: a College of American Pathologists Q-probes study of 601 institutions. Arch Pathol Lab Med. 1996;120(2):206-211.
14. Jones JL, Lopez A, Wahlquist SP, Nadle J, Wilson M; Emerging Infections Program FoodNet Working Group. Survey of clinical laboratory practices for parasitic diseases. Clin Infect Dis. 2004;38(Suppl 3):S198-S202. https://doi.org/10.1086/381587
15. Tewell CE, Talbot TR, Nelson GE, et al. Reducing inappropriate testing for the evaluation of diarrhea among hospitalized patients. Am J Med. 2018;131(2):193-199.e1. https://doi.org/10.1016/j.amjmed.2017.10.006
16. Thielman NM, Guerrant RL. Clinical practice. acute infectious diarrhea. N Engl J Med. 2004;350(1):38-47. https://doi.org/10.1056/nejmcp031534
17. Marti H, Koella JC. Multiple stool examinations for ova and parasites and rate of false-negative results. J Clin Microbiol. 1993;31(11):3044-3045. https://doi.org/10.1128/jcm.31.11.3044-3045.1993

Issue
Journal of Hospital Medicine 16(4)
Issue
Journal of Hospital Medicine 16(4)
Page Number
236-238. Published Online First March 17, 2021
Page Number
236-238. Published Online First March 17, 2021
Publications
Publications
Topics
Article Type
Display Headline
Things We Do for No Reason™: Ova and Parasite Testing in Patients With Acute Diarrhea Arising During Hospitalization
Display Headline
Things We Do for No Reason™: Ova and Parasite Testing in Patients With Acute Diarrhea Arising During Hospitalization
Sections
Article Source

© 2021 Society of Hospital Medicine

Disallow All Ads
Correspondence Location
Shmuel Golfeyz; Email: sgolfeyz@gmail.com.
Content Gating
Gated (full article locked unless allowed per User)
Alternative CME
Disqus Comments
Default
Use ProPublica
Hide sidebar & use full width
render the right sidebar.
Conference Recap Checkbox
Not Conference Recap
Clinical Edge
Display the Slideshow in this Article
Gating Strategy
First Page Free
Medscape Article
Display survey writer
Reuters content
Article PDF Media